Which statement is true regarding the recording of data from the history and physical examination

ANS: C) skin color

Show

General appearance includes items such as level of consciousness, skin color, nutritional status, posture, mobility, facial expression, mood and affect, speech, hearing, and personal hygiene. Height, weight, and vital signs are considered measurements.
Page: 764

Earl W. Campbell, JR and Christopher K. Lynn.

Table of Contents

  • Which definition correctly describes a person’s functional ability?a.Functional ability is the measure of the expected changes of aging that a person is experiencing.b.Functional ability refers to an individual’s motivation to live independently.c.Functional ability refers to the level of cognition present in an older person.d.Functional ability refers to a person’s ability to perform activities necessary to live in modern society.
  • The nurse is preparing to perform a functional assessment of an older patient, and knows that a good approach would be to:a.observe the patient’s ability to perform tasks.b.ask the patient’s wife how well he performs tasks.c.review the medical record for information about the patient’s abilities.d.ask the patient’s physician for information about the patient’s abilities.
  • The nurse will choose which of the following tools to assess a patient’s ability to perform activities of daily living?a.Direct Assessment of Functional Abilities (DAFA)b.Lawton and Brody IADLc.Katz Indexd.Older Americans Resources and Services Multidimensional Functional Assessment Questionnaire-IADL (OARS-IADL)
  • Which of the following statements about the Lawton IADL instrument is true?a.The nurse uses direct observation to implement this tool.b.It is designed as a self-report measure of performance, rather than ability.c.It is not useful in the acute hospital setting.d.It is best used for those residing in an institutional setting.
  • The nurse is assessing an older adult’s advanced activities of daily living, which would include:a.recreational activities.b.meal preparation.c.balancing the chequebook.d.self-grooming activities.
  • When using the various instruments to assess an older person’s activities of daily living, the nurse needs to remember that a disadvantage of these instruments includes:a.the reliability of the tools.b.the self or proxy report of functional activities.c.the lack of confidentiality during the assessment.d.insufficient detail about the deficiencies identified.
  • The nurse is administering a test that is timed over 15 minutes, and assesses a person’s upper body fine and coarse motor activities, balance, mobility, coordination, and endurance. During this test, activities such as dressing and stair climbing are timed. Which test is described by these activities?a.The Up and Go Testb.The Performance Activities of Daily Livingc.The Physical Performance Testd.Tinetti Gait and Balance Evaluation
  • A patient will be ready to be discharged from the hospital soon, and the patient’s family members are concerned about whether he is able to go outside alone safely. The nurse will perform which test to assess this ability?a.The Up and Go Testb.The Performance Activities of Daily Livingc.The Physical Performance Testd.Tinetti Gait and Balance Evaluation
  • The nurse is assessing the forms of support an older patient has before she is discharged. Which of the following illustrates an informal source of support?a.The local senior centreb.Her cleaning ladyc.Her Meals on Wheels meal delivery serviced.Her neighbour, who visits with her daily
  • An 85-year-old man has been hospitalized after a fall at home, and his 86-year-old wife is at his bedside. She tells the nurse that she is his primary caregiver. The nurse should assess the caregiver for signs of possible caregiver burnout, such as:a.depression.b.weight gain.c.hypertension.d.social phobias.
  • During a morning assessment, the nurse notices that an older patient is less attentive and is unable to recall yesterday’s events. The nurse administers the Mini Mental State Examination, which will screen for:a.dementia.b.depression.c.delirium.d.psychosis.
  • During an assessment of a newly admitted 92-year-old woman, the nurse notes that her son does not want to leave the room. The woman has signs of old bruises and healed cuts that happened “last week,” according to the son. Which of the following actions by the nurse is appropriate?a.Ask the son for details about the nature of the patient’s injuries.b.Recognize that older people are often unsteady on their feet, and that falls do occur.c.Notify the authorities of a potential abusive situation.d.Recognize that these findings do not necessarily indicate that abuse has occurred, but are signs that further assessment is needed.
  • Which of the following statements regarding common environmental hazards is most appropriate for the nurse to make during a functional assessment of an older person’s home environment?a.“These low toilet seats are safe because they are nearer to the ground in case of falls.”b.“Ask a relative or friend to help you to install grab bars in your shower.”c.“These small rugs are ideal for preventing you from slipping on the hard floor.”d.“It would be safer to keep the lighting low in this room to avoid glare in your eyes.”
  • Which of the following questions would be most appropriate for the nurse to ask when beginning to assess a person’s spirituality?a.“Do you believe in God?”b.“Do you consider yourself to be a spiritual person?”c.“What religious faith do you follow?”d.“Do you believe in the power of prayer?”
  • The nurse is preparing to assess an older adult, and discovers that he is in severe pain. Which of the following statements about pain and the older adult is true?a.Pain is inevitable with aging.b.Older adults with cognitive impairment feel less pain.c.Alleviating pain should take priority over other aspects of the assessment.d.The assessment should take priority so that care decisions can be made.
  • The nurse is assessing the abilities of an older adult. Which of the following activities are considered instrumental activities of daily living? Select all that apply.a.Feeding oneselfb.Preparing a mealc.Balancing a chequebookd.Walkinge.Toiletingf.Grocery shopping
  • Which of the following best describes the action of the hormone progesterone during pregnancy?a.It produces the hormone human chorionic gonadotropin.b.It stimulates duct formation in the breast.c.It promotes sloughing of the endometrial wall.d.It maintains the endometrium around the fetus.
  • A female patient is experiencing nausea, breast tenderness, fatigue, and amenorrhea. Her last menstrual period was 6 weeks ago. The nurse recognizes that this patient is experiencing:a.positive signs of pregnancy.b.possible signs of pregnancy.c.probable signs of pregnancy.d.presumptive signs of pregnancy.
  • When performing the examination of a woman who is 8 weeks pregnant, the nurse notes that her cervix is a bluish colour. The nurse would document this finding as:a.Hegar’s sign.b.Homan’s sign.c.Chadwick’s sign.d.Goodell’s sign.
  • A woman who is 8 weeks pregnant is visiting the clinic for a checkup. Her systolic blood pressure is 30 mm Hg higher than her pre-pregnancy blood pressure. The nurse would:a.consider this a normal finding.b.expect the blood pressure to decrease as the estrogen levels increase throughout the pregnancy.c.consider this an abnormal finding, because blood pressure is typically lower at this point in the pregnancy.d.recommend that she decrease her salt intake in an attempt to decrease her peripheral vascular resistance.
  • A patient is being seen at the clinic for her 10-week prenatal visit. She asks when she will be able to hear the baby’s heartbeat. The nurse should reply:a.“The baby’s heartbeat is not usually heard until the second trimester.”b.“The baby’s heartbeat may be heard anywhere from the ninth to the twelfth week.”c.“It is often difficult to hear the heartbeat at this point, but we can try.”d.“It is normal to hear the heartbeat at 6 weeks. We may be able to hear it today.”
  • A patient who is in her first trimester of pregnancy tells the nurse that she is experiencing significant nausea and vomiting, and asks when it will improve. The nurse should reply:a.“Did your mother have significant nausea and vomiting?”b.“Many women experience nausea and vomiting until the third trimester.”c.“Usually, by the beginning of the second trimester, the nausea and vomiting improve.”d.“At about the time you begin to feel the baby move, the nausea and vomiting will subside.”
  • During the examination of a woman in her second trimester of pregnancy, the nurse notes the presence of a small amount of yellow drainage from the nipples. The nurse knows that this is:a.an indication that the woman’s milk is coming in.b.a sign of possible breast cancer in a pregnant woman.c.most likely colostrum, which is considered a normal finding at this stage of the pregnancy.d.an early stage in the pregnancy for lactation to begin. The woman should be referred to a specialist.
  • A woman in her second trimester of pregnancy complains of heartburn and indigestion. The nurse should offer which of the following explanations for these problems?a.Tone and motility of the gastrointestinal tract increase during the second trimester.b.Sluggish emptying of the gallbladder, resulting from the effects of progesterone, often causes heartburn.c.Lower blood pressure at this time decreases blood flow to the stomach and gastrointestinal tract.d.The enlarging uterus and altered esophageal sphincter tone predispose the woman to have heartburn.
  • A patient who is 20 weeks pregnant tells the nurse that she feels more short of breath as her pregnancy progresses. The nurse recognizes that which of the following is true?a.High levels of estrogen cause shortness of breath.b.Feelings of shortness of breath are abnormal during pregnancy.c.The hormones of pregnancy cause an increased respiratory effort.d.The patient should get more exercise in an attempt to increase her respiratory reserve.
  • The nurse auscultates a functional systolic murmur, grade ii/iv, in a woman in week 30 of her pregnancy. The remainder of her physical assessment is within normal limits. The nurse would:a.consider this an abnormal finding, and refer her for additional consultation.b.ask the woman to run in place, and then assess for an increase in the intensity of the murmur.c.know that this is a normal finding, resulting from the increase in blood volume during pregnancy.d.ask the woman to restrict her activities, and return to the clinic in 1 week for re-evaluation.
  • A woman who is 28 weeks pregnant has edema in her lower legs bilaterally after working 8 hours a day as a cashier at a local grocery store. What should the nurse tell her?a.“You will be at risk for development of varicose veins when your legs are edematous.”b.“I would like to listen to your heart sounds. Edema can indicate a problem with your heart.”c.“Edema is usually the result of too much salt and fluids in your diet. You may need to try to cut down on salty foods.”d.“As your baby grows, it slows blood return from your legs, causing the swelling. This often occurs with prolonged standing.”
  • The nurse knows that classic symptoms associated with preeclampsia include:a.proteinuria, headaches, and seizures.b.elevated blood pressure and proteinuria.c.elevated liver enzymes and high platelets.d.neurological signs, elevated blood pressure, and edema.
  • The nurse knows that the best time to assess a woman’s blood pressure during an initial prenatal visit is:a.at the end of the examination, when she will be the most relaxed.b.at the beginning of the interview, as a nonthreatening method of establishing rapport.c.during the middle of the physical examination, when she is the most comfortable.d.before beginning the pelvic examination, because her blood pressure will be higher after the pelvic examination.
  • When examining the face of a 28-week pregnant woman, the nurse notes the presence of a butterfly-shaped increase in pigmentation on her face. When documenting, the correct term for this finding is:a.striae.b.chloasma.c.linea nigra.d.the mask of pregnancy.
  • Which of the following findings is considered normal and expected when the nurse is performing a physical examination on a pregnant woman?a.A palpable, full thyroidb.Spontaneously bleeding gingivac.Significant, diffuse enlargement of the thyroidd.Pale, hypertrophied mucous membranes of the mouth
  • When auscultating the anterior thorax of a pregnant woman, the nurse notes the presence of a murmur over the second, third, and fourth intercostal spaces. It is continuous, but can be obliterated by pressure with the stethoscope or finger on the thorax just lateral to the murmur. The nurse knows that this is:a.the murmur of aortic stenosis.b.most likely a mammary souffle.c.associated with aortic insufficiency.d.an indication of a patent ductus arteriosis.
  • When the nurse is assessing the deep tendon reflexes (DTRs) of a 32-week pregnant woman, which of the following findings would be considered normal, on a 0–4+ scale?a.Absent DTRsb.2+c.4+d.Brisk reflexes and the presence of clonus
  • When performing an examination of a 34-week pregnant woman, the nurse notes that as the woman raises her head and shoulders off of the bed, there is a midline linear protrusion in the abdomen over the area of the rectus abdominis muscles. The nurse would:a.document the presence of diastasis rectus abdominis.b.discuss this condition with the physician, because it will most likely need to be surgically repaired.c.suspect that the woman has a hernia from the increased pressure within the abdomen from pregnancy.d.tell the woman that she may have a difficult time with delivery because of the weakness in her abdominal muscles.
  • When palpating the fundus, the nurse knows that:a.it should be hard and slightly tender to palpation during the first trimester.b.fetal movement should be felt by the examiner at the beginning of the second trimester.c.after 20 weeks’ gestation, the number of centimetres should approximate the number of weeks of gestation.d.fundal height is usually less than the number of weeks of gestation, unless there is an abnormal condition such as the presence of too much amniotic fluid.
  • The nurse is palpating the abdomen of a woman who is 35 weeks pregnant, and notes that the fetal head is facing downward toward the pelvis. The nurse would document this as:a.fetal lie.b.fetal variety.c.fetal attitude.d.fetal presentation.
  • During the health history of a woman who is pregnant with her first child, the woman states, “I just cannot stop crunching on ice! What’s wrong with me?” The nurse recognizes that:a.she is experiencing a common food intolerance.b.the woman is experiencing pica, or craving for nonfood items, which is sometimes associated with anemia.c.she may be experiencing gastrointestinal changes associated with pregnancy.d.she probably craved ice before her pregnancy, and this is nothing new.
  • Which of the following findings would be most consistent with an 8-week pregnant uterus?a.The uterus seems slightly enlarged and softened.b.It reaches to the pelvic brim, and is about the size of a grapefruit.c.It rises above the pelvic brim, and is about the size of a cantaloupe.d.It is approximately 8 cm across the fundus, and is about the size of an avocado.
  • A woman in week 25 of her pregnancy has come to the clinic with a complaint of 3 weeks of bouts of severe vomiting. The nurse notes that she is showing signs of dehydration. Her blood pressure is lower than usual, and she is extremely fatigued. The nurse recognizes that this patient is experiencing:a.preeclampsia.b.polyhydramnios.c.proteinuria.d.hyperemesis.
  • Which of the following time periods correctly describes the average length of pregnancy?a.38 weeksb.9 lunar monthsc.280 days from the last day of the last menstrual periodd.280 days from the first day of the last menstrual period
  • A patient’s pregnancy test is positive, and she wants to know when the baby is due. The first day of her last menstrual period was June 14, and that period ended June 20. What is her expected date of delivery, using Nägele’s rule?a.March 7b.March 14c.March 21d.March 27
  • During the assessment of a woman in week 22 of her pregnancy, the nurse is unable to hear fetal heart tones with the Doppler device. The nurse should:a.wait 10 minutes and try again.b.notify the physician immediately.c.use a fetoscope to identify fetal heart tones.d.use an ultrasound to verify cardiac activity.
  • A patient, who is 24 weeks pregnant, asks about wearing a seatbelt while driving. The nurse should reply:a.“Seatbelts should not be worn during pregnancy.”b.“Place the lap belt below the uterus, and use the shoulder strap at the same time.”c.“Place the lap belt below the uterus, but omit the shoulder strap during pregnancy.”d.“Place the lap belt at your waist above the uterus, and use the shoulder strap at the same time.”
  • Which of the following statements is true regarding pregnancy after 35 years of age?a.Women over 35 years of age deliver most often by vaginal delivery.b.The occurrence of having a child with Down syndrome is much more frequent after 35 years of age.c.Genetic counselling and prenatal screening are not routine until after 40 years of age.d.Women over 35 years of age who are pregnant have the same rate of pregnancy-related complications as those who are under 35 years of age.
  • A 25-year-old woman is visiting the clinic for her first prenatal visit. Which laboratory screening is appropriate at this time?a.Human chorionic gonadotropinb.Complete blood cell countc.Alpha-fetoproteind.Carrier screening for cystic fibrosis
  • A woman, who is in week 25 of her pregnancy, comes to the clinic for her prenatal visit. The nurse notes that her face is swollen and that her blood pressure is 144/94 mm Hg. She states that she has had headaches and blurry vision, but thought she was just tired. What should the nurse suspect?a.Eclampsiab.Preeclampsiac.Diabetes type 1d.Premature labour
  • During auscultation of fetal heart tones (FHTs), the nurse determines that the rate is 136 beats per minute. The nurse’s next action should be to:a.document the results, which are within normal range.b.take the maternal pulse to verify these findings as the uterine souffle.c.have the patient change positions, and count the FHTs again.d.notify the physician immediately for possible fetal distress.
  • In week 34 of pregnancy, a woman she is told that she has preeclampsia. The nurse knows that which of the following statements about preeclampsia is true?a.Preeclampsia has little effect on the fetus.b.Edema is one of the main indications of preeclampsia.c.Eclampsia only occurs before delivery of the baby.d.Untreated preeclampsia may progress to eclampsia, which is manifested by generalized tonic-clonic seizures
  • During an internal examination of a woman during her first prenatal visit, the nurse notes that the cervix is soft. This is known as:a.Hegar’s sign.b.Chadwick’s sign.c.Homan’s sign.d.Goodell’s sign.
  • During a group prenatal teaching session, the nurse teaches Kegel exercises. Which of the following statements would be appropriate for this teaching session? Select all that apply.a.“Kegel exercises help to keep your uterus strong during the pregnancy.”b.“Kegel exercises should be performed twice a day.”c.“Kegel exercises should be performed 50 to 100 times a day.”d.“To perform Kegel exercises, squeeze slowly to a peak at the count of eight, then release slowly to a count of eight.”e.“To perform Kegel exercises, perform rapidly alternating squeeze-release exercises up to the count of eight.”
  • At the beginning of rounds, when the nurse enters the room, what should be done first?a.Check the intravenous infusion site for swelling or redness.b.Check the infusion pump settings for accuracy.c.Make eye contact with the patient and introduce himself or herself as the patient’s nurse.d.Offer the patient something to drink.
  • During an assessment, the nurse is unable to palpate pulses in the left lower leg. The nurse should:a.document that the pulses are not palpable.b.reassess the pulses in 1 hour.c.have the patient turn to the side, and then palpate for the pulses again.d.use a Doppler device to assess the pulses.
  • During a morning assessment, the nurse notes that a patient’s urine output is below the expected amount. What should be done next?a.An order for a Foley catheter should be obtained.b.An order for a straight catheter should be obtained.c.Perform a bladder scan test.d.Refer the patient to a urologist.
  • What should the nurse assess before entering the patient’s room on morning rounds?a.Posted conditions, such as isolation precautionsb.The patient’s input and output chart from the previous shiftc.The patient’s general appearanced.The presence of any visitors in the room
  • The nurse has administered a pain medication to a patient by intravenous infusion. The nurse should reassess the patient’s response to the pain medication within:a.5 minutes.b.15 minutes.c.30 minutes.d.60 minutes.
  • During an assessment of a hospitalized patient, the nurse pinches a fold of skin under the clavicle or on the forearm to note:a.mobility and turgor.b.the patient’s response to pain.c.the percentage of the patient’s fat-to-muscle ratio.d.the presence of edema.
  • When assessing the neurological system of a hospitalized patient during morning rounds, the nurse will include which of the following assessments?a.Blood pressureb.The patient’s rating of pain on a 1 to 10 scalec.The patient’s ability to communicated.The patient’s personal hygiene level
  • When assessing a patient’s general appearance, the nurse will include which of the following assessments?a.Is the patient’s muscle strength equal in both arms?b.Is ptosis or facial droop present?c.Does the patient respond appropriately to questions?d.Are the pupils equal in reaction and size?
  • Which statement reflects the assessment of a patient in the hospital setting?a.The patient will need a brief assessment at least every 4 hours.b.The patient will need a consistent, specialized exam every 8 hours that focuses on certain parameters.c.The patient will need a complete head-to-toe physical examination every 24 hours.d.Most patients require a minimal examination during each shift, unless they are in critical condition.
  • The nurse is assessing the intravenous (IV) infusion at the beginning of the shift. Which of the following should be included in the assessment of the infusion? Select all that apply.a.The proper IV solution is infusing, according to physician’s orders.b.Infusion is occurring at the proper rate, according to physician’s orders.c.The infusion solution and rate is proper, according to the nurse’s own assessment of the patient’s needs.
  • An 85-year-old man has come into the clinic for a physical examination, and the nurse notes that he uses a cane. When documenting general appearance, the nurse will document this information under the section that covers:a.posture.b.mobility.c.mood and affect.d.physical deformity.
  • After the health history has been obtained, and before beginning the physical examination, the nurse should ask the patient to first:a.empty the bladder.b.completely disrobe.c.lie on the examination table.d.walk around the room.
  • While the nurse palpates the maxillary sinuses, the patient tells the nurse that he has some tenderness in that area. The nurse should proceed by:a.tapping on the sinus area.b.auscultating the sinus area.c.asking him to blow his nose.d.transilluminating the sinuses.
  • A patient states “whenever I open my mouth real wide, I feel this popping sensation in front of my ears.” To further examine this, the nurse would:a.place the stethoscope over the temporomandibular joint, and listen for bruits.b.place the hands over his ears, and ask him to open his mouth “really wide.”c.place one hand on his forehead and the other on his jaw, and ask him to try to open his mouth.d.place a finger on his temporomandibular joint, and ask him to open and close his mouth.
  • The nurse has just completed an examination of a patient’s extraocular muscles. When documenting the findings, the nurse would note the assessment of which cranial nerves?a.II, III, VIb.II, IV, Vc.III, IV, Vd.III, IV, VI
  • A patient’s uvula rises midline when she says “ahh,” and she has a positive gag reflex. The nurse has just tested which cranial nerves?a.IX, Xb.IX, XIIc.X, XIId.XI, XII
  • During an examination, the nurse notices that a patient is unable to stick out his tongue. Which cranial nerve is involved with the successful performance of this action?a.Ib.Vc.XId.XII
  • A patient is unable to shrug her shoulders against the nurse’s resistant hands. What cranial nerve is involved with successful shoulder shrugging?a.VIIb.IXc.XId.XII
  • During an examination, a patient has successfully completed the finger-to-nose and the rapid-alternating-movements tests, and is able to run each heel down the opposite shin. The nurse will conclude that the patient’s:a.occipital function is intact.b.cerebral function is intact.c.temporal function is intact.d.cerebellar function is intact.
  • A 5-year-old child is in the clinic for a checkup. The nurse would expect him to:a.have to be held on his mother’s lap.b.be able to sit on the examination table.c.be able to stand on the floor for the examination.d.be able to remain alone in the examination room.
  • When the nurse performs the confrontation test, the nurse has assessed:a.EOMs.b.PERRLA.c.near vision.d.visual fields.
  • Which of the following statements is true regarding the recording of data from the health history and physical examination?a.Use long, descriptive sentences to document findings.b.Record the data as soon as possible after the interview and physical examination.c.If the information is not documented, it can be assumed that it was done as a standard of care.d.The examiner should avoid taking any notes during the history and examination because of the possibility of decreasing rapport with the patient.
  • Which of the following is included in assessment of general appearance?a.Heightb.Weightc.Skin colourd.Vital signs
  • Gloves should be worn for which of the following examinations?a.Measurement of vital signsb.Palpation of the sinusesc.Palpation of the mouth and tongued.Inspection of the eye with an ophthalmoscope
  • Which of the following is an appropriate location for eliciting deep tendon reflexes?a.Achillesb.Femoralc.Scapulard.Abdominal
  • During inspection of a patient’s face, the nurse notes that the facial features are symmetrical. This finding indicates that which cranial nerve is intact?a.VIIb.IXc.XId.XII
  • During inspection of the posterior chest, the nurse should assess for:a.symmetrical expansion.b.symmetry of the shoulders and muscles.c.tactile fremitus.d.diaphragmatic excursion.
  • When assessing the neonate, the nurse should use which of the following methods to test for hip stability?a.Elicit the Moro reflexb.Perform Romberg’s testc.Check for Ortolani’s signd.Assess the stepping reflex
  • During an examination, the patient tells the nurse that she sometimes feels as if objects are spinning around her. The nurse would note that she occasionally experiences:a.vertigo.b.tinnitus.c.syncope.d.dizziness.
  • A patient tells the nurse that “sometimes I wake up at night and I have real trouble breathing. I have to sit up in bed to get a good breath.” When documenting this information, the nurse would note:a.orthopnea.b.acute emphysema.c.paroxysmal nocturnal dyspnea.d.acute shortness of breath episode.
  • During the examination of a patient, the nurse notes that the patient has several small, flat macules on her posterior thorax. These macules are less than 1 cm wide. Another name for these macules is:a.warts.b.bullas.c.freckles.d.papules.
  • During an examination, the nurse notes that a patient’s legs turn white when they are raised above her head. The nurse would suspect:a.lymphedema.b.Raynaud’s disease.c.chronic arterial insufficiency.d.chronic venous insufficiency.
  • The nurse documents that a patient has coarse, thickened skin and brown discoloration over the lower legs. Pulses are present. This finding is likely the result of:a.lymphedema.b.Raynaud’s disease.c.chronic arterial insufficiency.d.chronic venous insufficiency.
  • The nurse notes that a patient has ulcerations on the tips of the toes and on the lateral ankles. This finding would indicate:a.lymphedema.b.Raynaud’s disease.c.arterial insufficiency.d.venous insufficiency.
  • When the nurse flexes the patient’s knee and gently compresses the gastrocnemius muscle anteriorly against the tibia, the patient indicates that he is having calf pain. The nurse would document:a.positive Allen’s sign.b.negative Allen’s sign.c.positive Homan’s sign.d.negative Homan’s sign.
  • The nurse has just recorded a positive obturator test on a patient who has abdominal pain. This test is used to confirm a(n):a.inflamed liver.b.perforated spleen.c.perforated appendix.d.enlarged gallbladder.
  • The nurse is documenting the assessment of an infant. During the abdominal assessment, the nurse notes a very loud splash auscultating over the upper abdomen when the nurse rocks her from side to side. This finding would indicate:a.epigastric hernia.b.pyloric obstruction.c.hypoactive bowel sounds.d.hyperactive bowel sounds.
  • The nurse will use which of the following tools to measure a patient’s near vision?a.A Snellen eye chart with lettersb.A Snellen “E” chartc.A Jaeger cardd.An ophthalmoscope
  • If the nurse records the results to the Hirschberg test, the nurse has:a.tested the patellar reflex.b.assessed for appendicitis.c.tested the corneal light reflex.d.assessed for thrombophlebitis.
  • During the examination of a patient’s mouth, the nurse observes a nodular, bony ridge down the middle of the hard palate. The nurse would chart this finding as:a.cheilosis.b.leukoplakia.c.ankyloglossia.d.torus palatinus.
  • During an examination, the nurse finds that a patient is unable to distinguish objects placed in his hand. The nurse would document:a.stereognosis.b.astereognosis.c.graphesthesia.d.agraphesthesia.
  • After the examination of an infant, the nurse notes opisthotonos. The nurse recognizes that this finding often occurs with:a.cerebral palsy.b.meningeal irritation.c.lower motor neuron lesion.d.upper motor neuron lesion.
  • After assessing a female patient, the nurse notes flesh-coloured, soft, pointed, moist papules in a cauliflower-like patch around her introitus. This finding is most likely:a.a urethral caruncle.b.a syphilitic chancre.c.herpes.d.human papillomavirus.
  • While recording in a patient’s chart, the nurse notes that a patient’s hematest results have been positive. This means that:a.there are crystals in his urine.b.there are parasites in his stool.c.there is occult blood in his stool.d.there are bacteria in his sputum.
  • While examining a 48-year-old patient’s eyes, the nurse notes that he has to move the hand-held vision screener farther away from his face. The nurse would suspect:a.myopia.b.omniopia.c.hyperopia.d.presbyopia.
  • Which of the following assessments is most appropriate to perform on a 9-month-old well child?a.Assessment for Ortolani’s signb.Assessment for stereognosisc.Assessment of blood pressured.Assessment for the presence of the startle reflex
  • The female structure that corresponds with the male penis is called the:a.labia.b.clitoris.c.prepuce.d.frenulum.
  • When observing the vestibule, the nurse should be able to see the:a.urethral meatus and vaginal orifice.b.vaginal orifice and vestibular (Bartholin’s) glands.c.urethral meatus and paraurethral (Skene’s) glands.d.paraurethral (Skene’s) and vestibular (Bartholin’s) glands.
  • During an inspection of the vagina, the nurse would expect to see what at the end of the vagina?a.Cervixb.Uterusc.Ovariesd.Fallopian tubes
  • The uterus is usually positioned tilting forward and superior to the bladder. This position is known as:a.anteverted and anteflexed.b.retroverted and anteflexed.c.retroverted and retroflexed.d.superior verted and anteflexed.
  • An 11-year-old girl is in the clinic for a sports physical. The nurse notes that she has begun to develop breasts, and during the conversation the girl reveals that she is unsure about the progression of development. The nurse should use which of the following to best assist the young girl in understanding the expected sequence for development?a.The nurse should use Tanner’s table on the five stages of sexual development.b.The nurse should describe her development and compare it with that of other girls her age.c.Jacobsen’s table on expected development on the basis of height and weight data should be used.d.The nurse should reassure her that her development is within normal limits, and tell her not to worry about the next step.
  • A woman who is 8 weeks pregnant is in the clinic for a check. The nurse reads on her chart that her cervix is softened and looks cyanotic. The nurse knows that the woman is exhibiting which of the following signs?a.Tanner’s sign and Hegar’s signb.Hegar’s sign and Goodell’s signc.Chadwick’s sign and Hegar’s signd.Goodell’s sign and Chadwick’s sign
  • A woman who is 22 weeks pregnant has a vaginal infection. She tells the nurse that she is afraid that this infection will “hurt my baby.” The nurse knows that which of the following statements is true?a.If intercourse is avoided, the risk for infection is minimal.b.A thick mucus plug forms that protects the fetus from infection.c.The acidic pH of vaginal secretions promotes the growth of pathogenic bacteria.d.The mucus plug that forms in the cervical canal is a good medium for bacterial growth.
  • The changes normally associated with menopause occur generally because the cells in the reproductive tract are:a.aging.b.becoming fibrous.c.estrogen dependent.d.able to respond to estrogen.
  • Which of the following are changes associated with menopause?a.Uterine and ovarian atrophy, along with thinning vaginal epitheliumb.Ovarian atrophy, increased vaginal secretions, and increasing clitoral sizec.Cervical hypertrophy, ovarian atrophy, and increased acidity of vaginal secretionsd.Vaginal mucosa fragility, increased acidity of vaginal secretions, and uterine hypertrophy
  • A 54-year-old woman who has just completed menopause is visiting the clinic for a yearly physical examination. Which of the following should the nurse include in patient education?a.A postmenopausal woman is not at any greater risk for heart disease than a younger woman is.b.A postmenopausal woman should be aware that she is at increased risk for dyspareunia because of decreased vaginal secretions.c.A postmenopausal woman has only stopped menstruating; there really are no other significant changes that she should be concerned with.d.A postmenopausal woman is likely to have difficulty with sexual pleasure as a result of drastic changes in the female sexual response cycle.
  • A patient has had three pregnancies and two live births. How would the nurse record this information?a.Gravida 2, para 2, AB 1b.Gravida 3, para 2, AB 0c.Gravida 3, para 2, AB 1d.Gravida 3, para 3, AB 1 (adsbygoogle = window.adsbygoogle || []).push({});
  • During the interview with a female patient, the nurse gathers data that indicate that the patient is perimenopausal. Which of the following statements made by this patient leads to this conclusion?a.“I have noticed that my muscles ache at night when I go to bed.”b.“I will be very happy when I can stop worrying about having a period.”c.“I have been noticing that I sweat a lot more than I used to, especially at night.”d.“I have only been pregnant twice, but both times I had breast tenderness as my first symptom.”
  • A 50-year-old woman calls the clinic because she has noticed some changes in her body and breasts, and wonders if they could be due to the estrogen replacement therapy she started 3 months ago. The nurse should tell her:a.“Estrogen replacement therapy is at such a low dose that side effects are very unusual.”b.“Estrogen replacement therapy has several side effects, including fluid retention, breast tenderness or enlargement, and vaginal bleeding.”c.“It would be very unusual to have breast tenderness with estrogen replacement therapy, and I suggest you contact your doctor immediately to have this evaluated.”d.“It sounds as if your dose of estrogen is too high. I think you should decrease the amount you are taking, and should call me back in a month to let me know how you’re feeling.”
  • A 52-year-old patient states that when she sneezes or coughs, she “wets herself a little.” She is very concerned that something may be wrong with her. The nurse knows that the problem is:a.hematuria, and usually needs to be evaluated by a urologist.b.stress incontinence, and is usually due to muscle weakness.c.true urinary incontinence, and may mean that she has a kidney infection.d.urgency incontinence, and she should empty her bladder before she sneezes or coughs.
  • During the interview, a patient reveals that she has some vaginal discharge. She is worried that it may be a sexually transmitted infection. The nurse’s most appropriate response to this would be:a.“Oh, don’t worry. Some cyclic vaginal discharge is normal.”b.“Have you been engaging in unprotected sexual intercourse?”c.“I’d like some information about the discharge. What colour is it?”d.“Have you had any urinary incontinence associated with the discharge?”
  • A woman states that 2 weeks ago she had a urinary tract infection that was treated with an antibiotic. As a part of the interview, the nurse must make sure to ask:a.“Have you noticed a change in your vaginal pH?”b.“Have you noticed any excessive vaginal bleeding?”c.“Have you noticed any unusual vaginal discharge or itching?”d.“Have you noticed any changes in your desire for intercourse?”
  • Which statement would be most appropriate when the nurse is introducing the topic of sexual relationships during an interview?a.“Now, it’s time to talk about your sexual history. When did you first have intercourse?”b.“Women often feel dissatisfied with their sexual relationships. Would it be okay to discuss this now?”c.“Often women have questions about their sexual relationships and how they affect their health. Do you?”d.“Most women your age have had more than one sexual partner. How many would you say you have had?”
  • A 22-year-old woman has been considering using oral contraceptives. As a part of her history, the nurse should ask:a.“Do you have a history of heart murmurs?”b.“Will you be in a monogamous relationship?”c.“I wonder if you have thought this choice through carefully.”d.“If you smoke, how many cigarettes do you smoke per day?”
  • A married couple has come to the clinic seeking advice on pregnancy. They have been trying to conceive for 6 months, and have not been successful. What should the nurse do first?a.Ascertain whether either of them has been using broad-spectrum antibiotics.b.Explain that couples are considered infertile after 1 year of unprotected intercourse.c.Immediately refer the woman to an expert in pelvic inflammatory disease (the most common cause of infertility).d.Explain that couples are considered infertile after 6 months of engaging in unprotected intercourse, and that they will need a referral to a fertility expert.
  • A nurse is assessing a patient’s risk of contracting a sexually transmitted infection (STI). An appropriate question would be:a.“You use condoms, don’t you?”b.“Do you use a condom at each episode of sexual intercourse?”c.“Do you have an STI?”d.“You are aware of the dangers of unprotected sex, aren’t you?”
  • When the nurse is interviewing a preadolescent girl, which of the following opening statement would be least threatening?a.“Do you have any questions about growing up?”b.“What has your mother told you about growing up?”c.“When did you notice that your body was changing?”d.“I remember being very scared when I got my period. How do you think you’ll feel?”
  • When the nurse is discussing sexuality and sexual issues with adolescents, a permission statement helps to convey that it is normal to think or feel a certain way. Which of the following is the best example of a permission statement?a.“It’s okay that you have become sexually active.”b.“Often girls your age have questions about sexual activity. Do you?”c.“If it’s okay with you, I’d like to ask you some questions about your sexual history.”d.“Often girls your age engage in sexual activity. It’s okay to tell me if you have had intercourse.”
  • Which of the following statements is true with regard to the history of a postmenopausal woman?a.The nurse should ask a postmenopausal woman if she ever has vaginal bleeding.b.Once a woman reaches menopause, the nurse does not need to ask any further history questions.c.The nurse should screen for monthly breast tenderness.d.Postmenopausal women are not at risk for contracting sexually transmitted infections, and thus these questions can be omitted.
  • During the examination portion of a patient’s visit, she will be in the lithotomy position. Which of the following statements reflect some things that the nurse can do to make this more comfortable for her?a.Ask her to place her hands and arms behind her head.b.Elevate her head and shoulders to maintain eye contact.c.Allow her to choose to have her feet in the stirrups or have them resting side by side on the edge of the table.d.Allow her to keep her buttocks about 15 cm (6 in.) from the edge of the table to prevent her from feeling as if she will fall off.
  • An 18-year-old patient is having her first pelvic examination. It would be appropriate to:a.invite her mother to be present during the examination.b.avoid the lithotomy position this first time because it can be uncomfortable and embarrassing.c.raise the head of the examination table and give her a mirror so that she can view the exam.d.drape her fully, leaving the drape between her legs elevated to avoid embarrassing her with eye contact.
  • The nurse has just completed an inspection of a woman’s external genitalia. Which of the following describes a finding within normal limits?a.Redness of the labia majorab.Multiple nontender sebaceous cystsc.Discharge that is sticky and yellow-greend.Swelling of the perineum before onset of menses
  • The order of examination of the internal genitalia is important. The nurse will use which order of examination at this time?a.Bimanual examination, speculum examination, rectovaginal examinationb.Speculum examination, rectovaginal examination, bimanual examinationc.Speculum examination, bimanual examination, rectovaginal examinationd.Rectovaginal examination, bimanual examination, speculum examination
  • During an internal examination of a woman’s genitalia, the nurse will use which technique for proper insertion of the speculum?a.Instruct the woman to bear down, open the speculum blades, and apply in a swift, upward movement.b.Insert the blades of the speculum on a horizontal plane, turning them to a 45-degree angle while continuing to insert them. Ask the woman to bear down to ease insertion.c.Push the introitus down and open, instruct the woman to bear down, and insert the speculum with the width of the blades at an oblique angle, applying any pressure downward.d.Lock the blades open by turning the thumbscrew. Once the blades are open, apply pressure to the introitus and insert the blades at a 45-degree angle downward to bring the cervix into view.
  • The nurse is examining a 35-year-old female patient. During the history, the nurse notes that she has had two term pregnancies, with both babies delivered vaginally. The nurse observes the following on internal examination: the cervical os is a horizontal slit with some healed lacerations; the cervix has some nabothian cysts that are small, smooth, and yellow; the cervical surface is granular and red, especially around the os; and the presence of stringy, opaque, odorless secretions. Which of these findings are abnormal?a.The presence of nabothian cystsb.The cervical os is a horizontal slitc.The cervical surface is granular and redd.The presence of stringy and opaque secretions
  • A patient calls the clinic for instructions before having a Papanicolaou (Pap) smear. The most appropriate instructions from the nurse would be:a.“If you are menstruating, please use pads to avoid placing anything into the vagina.”b.“Avoid intercourse, inserting anything into the vagina, or douching within 24 hours of your appointment.”c.“If you suspect that you have a vaginal infection, please gather a sample of the discharge to bring with you.”d.“We would like you to use a mild saline douche before your examination. You may pick this up in our office.”
  • Which of the following tests are usually collected when screening for cervical cancer?a.Endocervical specimen, cervical scrape, and vaginal poolb.Endocervical specimen, vaginal pool, and acetic acid washc.Endocervical specimen, KOH prep, and acetic acid washd.Cervical scrape, acetic acid wash, and saline mount (“wet prep”)
  • In performing the bimanual examination, the nurse notes that the cervix feels smooth and firm, is round, and is fixed in place (does not move). When cervical palpation is performed, the patient complains of some pain. Which of the following should be the nurse’s interpretation of these results?a.These findings are all within normal limits.b.The cervical consistency should be soft and velvety, not firm.c.The cervix should move when palpated; an immobile cervix may indicate malignancy.d.It is unusual to have pain when the cervix is palpated, but the rest of the findings are within normal limits.
  • The nurse is palpating a female patient’s adnexa. The findings include a firm, smooth uterine wall, palpable ovaries that feel smooth and firm, and a fallopian tube that is firm and pulsating. The nurse’s most appropriate course of action would be to:a.tell the patient that her examination was normal.b.give the patient an immediate referral to a gynecologist.c.suggest that the patient return in a month for a recheck to verify the findings.d.tell the patient that she may have an ovarian cyst that should be evaluated further.
  • A 65-year-old woman is visiting the office for routine gynecological care. She had a complete hysterectomy 3 months ago. The nurse knows that which of the following statements is true with regard to this visit?a.Her cervical mucosa will be red and look dry.b.She will not have a cervix, and thus does not need to have a Pap smear done.c.The nurse can expect to find that her uterus will be somewhat enlarged, and her ovaries will be small and hard.d.The nurse should plan to lubricate the instruments and the examining hand well in order to avoid a painful examination.
  • The nurse is preparing to examine the external genitalia of a school-age girl. Which of the following positions would be most appropriate in this situation?a.In the parent’s lapb.In a frog-leg position on the examining tablec.In the lithotomy position with the feet in stirrupsd.Lying flat on the examining table with legs extended
  • When assessing a newborn infant’s genitalia, the nurse notes that the genitalia are somewhat engorged. The labia majora are swollen, the clitoris looks large, and the hymen is thick. The vaginal opening is difficult to visualize. The infant’s mother states that she is worried about the labia being swollen. The nurse should reply:a.“This is a normal finding in newborns, and should resolve within a few weeks.”b.“This could possibly indicate an abnormality, and may need to be evaluated by a physician.”c.“We will need to have estrogen levels evaluated to make sure that they are within normal limits.”d.“We will need to keep close watch over the next few days to see if the genitalia decrease in size.”
  • During a vaginal examination of a 38-year-old woman, the nurse notes that the vulva and vagina are erythematous and edematous, with thick, white, curdlike discharge adhering to the vaginal walls. The woman reports intense pruritus and thick white discharge from her vagina. The nurse knows that these history and physical examination findings are most consistent with which of the following?a.Candidiasisb.Trichomoniasisc.Atrophic vaginitisd.Bacterial vaginosis
  • A 22-year-old woman is being seen at the clinic for problems with vulvar pain, dysuria, and fever. On physical examination, the nurse notes clusters of small, shallow vesicles with surrounding erythema on the labia. There is also inguinal lymphadenopathy present. The most likely cause of these lesions is:a.pediculosis pubis.b.contact dermatitis.c.human papillomavirus.d.herpes simplex virus type 2.
  • When performing an external genitalia examination for a 10-year-old girl, the nurse notes no pubic hair, and that the mons and the labia are covered with fine vellus hair. These findings are consistent with which stage of sexual maturity, according to the Sexual Maturity Rating scale?a.Stage 1b.Stage 2c.Stage 3d.Stage 4
  • A 46-year-old woman is in the clinic for her annual gynecological examination. She voices a concern about ovarian cancer, because her mother and sister died of it. The nurse knows that which of the following statements is correct regarding ovarian cancer?a.Ovarian cancer is manifested by severe abdominal pain.b.The Pap smear detects the presence of ovarian cancer.c.Women over 40 years of age should have a thorough pelvic examination every year.d.Women over 40 years of age should have a thorough pelvic examination every 3 years.
  • During a bimanual examination, the nurse detects a solid tumour on the patient’s ovary that is heavy and fixed, with a poorly defined mass. This finding is suggestive of:a.an ovarian cyst.b.endometriosis.c.ovarian cancer.d.an ectopic pregnancy.
  • A 25-year-old woman comes to the emergency department with a sudden fever of 38°C and abdominal pain. Upon examination, the nurse notes that she has rigid, boardlike lower abdominal musculature. When the nurse tries to perform a vaginal examination, the patient has severe pain when the uterus and cervix are moved. The nurse knows that these signs and symptoms are suggestive of:a.endometriosis.b.uterine fibroids.c.ectopic pregnancy.d.pelvic inflammatory disease.
  • During a female external genitalia examination, the nurse notes several lesions around the woman’s vulva. The lesions are pink, moist, soft, pointed papules. The patient states that she is not aware of any problems in that area. The nurse recognizes that these lesions may be:a.syphilitic chancre.b.herpes simplex virus type 2 (herpes genitalis).c.human papillomavirus (HPV), or genital warts.d.pediculosis pubis (crab lice).
  • During an examination, the nurse would expect the cervical os of a woman who has never had children to appear:a.stellate.b.small and round.c.as a horizontal irregular slit.d.everted.
  • A woman has just been diagnosed with HPV, or genital warts. The nurse should counsel her to receive regular examinations, because this virus makes her at a higher risk for:a.uterine cancer.b.cervical cancer.c.ovarian cancer.d.endometrial cancer.
  • Which of the following statements about the anal canal is true?a.The anal canal is about 2 cm long in the adult.b.The anal canal slants backward toward the sacrum.c.The anal canal contains hair and sebaceous glands.d.The anal canal is the outlet for the gastrointestinal tract.
  • Which of the following statements about the sphincters is true?a.The internal sphincter is under voluntary control.b.The external sphincter is under voluntary control.c.Both sphincters remain slightly relaxed at all times.d.The internal sphincter surrounds the external sphincter.
  • The nurse is performing an examination of the anus and rectum. Which of the following is important to remember during this examination?a.The rectum is about 8 cm long.b.The anorectal junction cannot be palpated.c.Above the anal canal, the rectum turns anteriorly.d.There are no sensory nerves in the anal canal or rectum.
  • The structure that secretes a thin, milky alkaline fluid to enhance the viability of sperm is the:a.Cowper’s gland.b.prostate gland.c.median sulcus.d.seminal vesicle.
  • A 46-year-old man requires assessment of his sigmoid colon. Which of the following is most appropriate for this examination?a.A proctoscopeb.An ultrasoundc.A colonoscoped.A rectal exam with an examining finger
  • The nurse is caring for a newborn infant. Thirty hours after birth, the infant passes a dark green meconium stool. The nurse recognizes that this is important because:a.this stool would indicate anal patency.b.the dark green colour could indicate occult blood in the stool.c.meconium stool can be reflective of distress in the newborn.d.the newborn should have passed the first stool within 12 hours after birth.
  • During the assessment of an 18-month-old, the mother expresses concern to the nurse about the infant’s inability to toilet train. What would be the nurse’s best response?a.“Some children are just more difficult to train, so I wouldn’t worry about it yet.”b.“Have you considered reading any of the books on toilet training? They can be very helpful.”c.“This could mean there is a problem in your baby’s development. We’ll watch her closely for the next few months.”d.“The nerves that will allow your baby to have control over the passing of stools are not developed until at least 18 to 24 months of age.”
  • A 60-year-old man has just been told he has benign prostatic hypertrophy. He has a friend who just died from prostate cancer, and is concerned this will happen to him. How should the nurse respond?a.“The swelling in your prostate is only temporary, and will go away.”b.“We will treat you with chemotherapy so we can control the cancer.”c.“It would be very unusual for a man your age to have prostate cancer.”d.“The enlargement of your prostate is caused by hormone changes and not cancer.”
  • A 30-year-old woman is visiting the clinic for a complaint of “pain in my bottom when I have a bowel movement.” The nurse should assess for which problem?a.Pinwormsb.Hemorrhoidsc.Food poisoningd.Fecal incontinence
  • A patient who is visiting the clinic complains of having “stomach pains for 2 weeks” and describes his stools as being “soft and black” for about the last 10 days. He denies taking any medications. The nurse is aware that these symptoms are most indicative of:a.excessive fat caused by malabsorption.b.increased iron intake resulting from a change in diet.c.occult blood resulting from gastrointestinal bleeding.d.increase in bile pigment from liver problems.
  • After completing an assessment of a 60-year-old man with a family history of colon cancer, the nurse discusses early detection measures for colon cancer with him. The nurse is sure to mention the need for a(n):a.annual proctoscopy.b.colonoscopy every 5 years.c.fecal test for blood every 2 years.d.digital rectal examination every 2 years.
  • The mother of a 5-year-old girl tells the nurse that she has noticed her daughter “scratching at her bottom a lot the last few days.” During the assessment, the nurse finds redness and raised skin in the anal area. This most likely indicates:a.pinworms.b.chickenpox.c.child abuse.d.bacterial infection.
  • The nurse is examining only the rectal area of a woman. In what position should the woman be placed?a.The lithotomy positionb.Lying in the prone positionc.The left lateral decubitus positiond.Bending over the table while standing
  • While doing an assessment of the perianal area of a patient, the nurse notes that the pigmentation of the anus is darker than the surrounding skin, that the anal opening is closed, and the presence of a skin sac that is shiny and blue. The patient mentions that he has had pain with bowel movements, and has noted some occasional spots of blood. What would this assessment and history most likely indicate?a.Anal fistulab.Pilonidal cystc.Rectal prolapsed.Thrombosed hemorrhoid
  • Which of the following techniques is correct for palpation of the rectum?a.Flex the finger and insert slowly toward the umbilicus.b.First instruct the patient that this will be a painful procedure.c.Insert an extended index finger at a right angle to the anus.d.Place the finger directly into the anus to overcome the tight sphincter.
  • While performing a rectal examination, the nurse notes a firm, irregularly shaped mass. What should the nurse do next?a.Continue with the exam, and note the finding in the chart.b.Instruct the patient to return for a repeat assessment in 1 month.c.Tell the patient that a mass was felt, but that it is nothing to worry about.d.Report the finding, and refer the patient to a specialist for further examination.
  • When testing stool for occult blood, the nurse is aware that a false-positive result may occur with:a.absent bile pigment.b.increased fat content.c.increased ingestion of iron medication.d.a large amount of red meat within the last 3 days.
  • During an assessment of the newborn, the nurse expects to see which finding when the anal area is slightly stroked?a.A jerking of the legsb.Flexion of the kneesc.A quick contraction of the sphincterd.Relaxation of the external sphincter
  • During an assessment of a 20-year-old man, the nurse finds a small, palpable lesion, containing a tuft of hair, located directly over the coccyx. The nurse knows that this lesion would most likely be a:a.polyp.b.pruritus ani.c.benign tumour.d.pilonidal cyst.
  • During an examination, the nurse asks the patient to perform the Valsalva manoeuvre and notes that the patient has a moist, red, doughnut-shaped protrusion from the anus. The nurse knows that this would be consistent with:a.a rectal polyp.b.hemorrhoids.c.a rectal fissure.d.rectal prolapse.
  • A 70-year-old man is visiting the clinic for “difficulty in passing urine.” In the history, he indicates he has to urinate frequently, especially at night. He experiences burning when he urinates, and has noticed pain in his back. Given this history, what might the nurse expect to find during the physical assessment?a.Asymmetric, hard, fixed prostate glandb.Occult blood and perianal pain with palpationc.Symmetrically enlarged, soft prostate glandd.A soft nodule protruding from rectal mucosa
  • A 40-year-old male of African descent is in the office for his annual physical. Which statement is true regarding screening for prostate cancer, according to the Canadian Cancer Society?a.DRE is more effective than PSA in finding prostate cancer.b.Ancestry is a factor in determining when PSA testing begins.c.PSA will be done at 50 years of age if there is a family history.d.BPH increases an individual’s risk for prostate cancer.
  • A 62-year-old man is experiencing fever, chills, malaise, and urinary frequency and urgency. He also reports urethral discharge and a dull, aching pain in the perineal and rectal area. These symptoms are most consistent with which of the following?a.Prostatitisb.Urinary tract infectionc.Carcinoma of the prostated.Benign prostatic hypertrophy
  • Which of the following is an abnormal finding upon palpation of the prostate gland through the rectum?a.Palpable central grooveb.Tenderness to palpationc.Heart shaped.Elastic and rubbery consistency
  • The nurse notes that a patient has had a pale, yellow, greasy stool, or steatorrhea, and recalls that this is caused by:a.occult bleeding.b.absent bile pigment.c.increased fat content.d.ingestion of bismuth preparations.
  • During a health history of a patient who complains of chronic constipation, she asks the nurse about high-fibre foods. The nurse relates that an example of a high-fibre food would be:a.broccoli.b.hamburger.c.iceberg lettuce.d.white dinner rolls.
  • While assessing a hospitalized, bedridden patient, the nurse notes that the patient has been incontinent of stool. The stool is loose and grey-tan in colour. The nurse recognizes that this finding indicates which of the following?a.Occult bloodb.Inflammationc.Absent bile pigmentd.Ingestion of iron preparations
  • During a digital examination of the rectum, the nurse notes that the patient has hard feces in the rectum. The patient complains of feeling “full,” has a distended abdomen, and states that she has not had a bowel movement “for several days.” The nurse suspects which condition?a.Rectal polypb.Fecal impactionc.Rectal abscessd.Rectal prolapse
  • The nurse is performing a digital examination of a patient’s prostate gland, and notes a normal prostate gland includes the following characteristics: Select all that apply.a.The gland is protruding 1 cm into the rectum.b.The gland is heart-shaped, with a palpable central groove.c.The gland is flat, with no palpable groove.d.The gland has a boggy and soft consistency.e.The gland has a smooth surface, and an elastic or rubbery consistency.f.The gland has fixed mobility.
  • The external male genital structures include the:a.testis.b.scrotum.c.epididymis.d.vas deferens.
  • An accessory glandular structure for the male genital organs is the:a.testis.b.penis.c.prostate.d.vas deferens.
  • Which of the following statements is true regarding the penis?a.The urethral meatus is located on the ventral side of the penis.b.The prepuce is the fold of foreskin covering the shaft of the penis.c.The penis is composed of two cylindrical columns of erectile tissue.d.The corpus spongiosum expands into a cone of erectile tissue called the glans.
  • When performing a genital examination on a 25-year-old man, the nurse notes deeply pigmented, wrinkled scrotal skin with large sebaceous follicles. On the basis of this information, the nurse would:a.squeeze the glans, checking for the presence of discharge.b.consider this a normal finding, and proceed with the examination.c.assess the testicles for the presence of masses or painless lumps.d.obtain a more detailed health history, focusing on any scrotal abnormalities the patient has noticed.
  • Which of the following statements about the testes is true?a.The lymphatics of the testes drain into the abdominal lymph nodes.b.The vas deferens is located along the inferior portion of each testis.c.The right testis is lower than the left because the right spermatic cord is longer.d.The cremaster muscle contracts in response to cold, and draws the testicles closer to the body.
  • A male patient with possible fertility problems asks the nurse where sperm is produced. The nurse knows that sperm production occurs in the:a.testes.b.prostate.c.epididymis.d.vas deferens.
  • A 62-year-old man states that his doctor told him that he has an “inguinal hernia.” He asks the nurse to explain what a hernia is. The nurse should:a.tell him not to worry, since most men his age develop hernias.b.explain that a hernia is often the result of prenatal growth abnormalities.c.refer him to his physician for additional consultation, because the physician made the initial diagnosis.d.explain that a hernia is a loop of bowel protruding through a weak spot in the abdominal muscles.
  • The mother of a 10-year-old boy asks the nurse to discuss how puberty can be recognized. The nurse should reply:a.“Puberty usually begins at about age 15.”b.“The first sign of puberty is enlargement of the testes.”c.“Penis size does not increase until about the age of 16.”d.“The development of pubic hair precedes testicular or penis enlargement.”
  • During an examination of an aging male, the nurse recognizes that a normal, expected change would be:a.premature ejaculation.b.declining testosterone production.c.difficulty in maintaining an erection.d.a decreased refractory state after ejaculation.
  • An older man is concerned about his sexual performance. The nurse knows that in the absence of disease, a withdrawal from sexual activity later in life may be due to:a.side effects of medications.b.decreased libido with aging.c.decreased sperm production.d.decreased pleasure from sexual intercourse.
  • A newborn baby boy is about to have a circumcision. The nurse knows that indications for circumcision include:a.cultural beliefs.b.prevention of testicular cancer.c.improving the sperm count later in life.d.preventing the transmission of human immunodeficiency virus during sexual intercourse.
  • A 59-year-old patient has been diagnosed with prostatitis, and is being seen at the clinic for complaints of burning and pain during urination. He is experiencing:a.dysuria.b.nocturia.c.polyuria.d.hematuria.
  • A 45-year-old carpenter is seen at the clinic for a complaint of “losing my urine when I lift heavy objects.” He is experiencing:a.frequency.b.urinary hesitancy.c.stress incontinence.d.urgency incontinence.
  • When the nurse is conducting a sexual history for a male adolescent, which statement would be most appropriate to use at the beginning of the interview?a.“Do you use condoms?”b.“You don’t masturbate, do you?”c.“Have you had sex in the last 6 months?”d.“Often boys your age have questions about sexual activity.”
  • Which of the following statements is most appropriate when the nurse is obtaining a genitourinary history from an elderly man?a.“Do you need to get up at night to urinate?”b.“Do you experience nocturnal emissions, or ‘wet dreams’?”c.“Do you know how to perform testicular self-examination?”d.“Has anyone ever touched your genitals when you did not want them to?”
  • While the nurse is performing a genital examination on a male patient, the patient has an erection. The nurse’s most appropriate action or response is to:a.ask the patient if he would like someone else to examine him.b.continue with the examination as though nothing has happened.c.stop the examination, state that the examination will resume at a later time, and leave the room.d.reassure the patient that this is a normal response, and continue with the examination.
  • The nurse knows that which of the following is a normal finding when examining the glans?a.The dorsal vein may be visible.b.Hair is without pest inhabitants.c.The skin is wrinkled and without lesions.d.Smegma may be present under the foreskin of an uncircumcised male.
  • When performing a genitourinary assessment, the nurse notes that the urethral meatus is positioned ventrally. This finding is:a.called hypospadius.b.the result of phimosis.c.probably due to a stricture.d.often associated with aging.
  • The nurse is performing a genital examination on a male patient, and notes urethral drainage. When collecting urethral discharge for microscopic examination and culture, the nurse should:a.ask the patient to urinate into a sterile cup.b.ask the patient to obtain a specimen of semen.c.insert a cotton-tipped applicator into the urethra.d.compress the glans between the examiner’s thumb and forefinger, and collect any discharge.
  • When assessing the scrotum of a male patient, the nurse notes the presence of multiple firm, nontender, yellow, 1-cm nodules. The nurse knows that these nodules are most likely:a.from urethritis.b.sebaceous cysts.c.subcutaneous plaques.d.from inflammation of the epididymis.
  • When performing a scrotal assessment, the nurse notes that the scrotal contents transilluminate and show a red glow. On the basis of this finding, the nurse would:a.assess the patient for the presence of a hernia.b.suspect the presence of serous fluid in the scrotum.c.consider this a normal finding, and proceed with the examination.d.refer the patient for evaluation of a possible mass in the scrotum.
  • When the nurse is performing a genital examination on a male patient, which of the following actions is correct?a.Auscultate for the presence of bowel sounds over the scrotum.b.Palpate for the vertical chain of lymph nodes along the groin inferior to the inguinal ligament.c.Palpate the inguinal canal only if there is a bulge present in the inguinal region during inspection.d.Have the patient shift his weight onto the left (unexamined) leg when palpating for a hernia on the right side.
  • The nurse is aware that which of the following statements is true regarding the incidence of testicular cancer?a.Testicular cancer is the most common cancer in men aged 30–50 years.b.The early symptoms of testicular cancer are pain and induration.c.Men with a history of cryptorchidism are at greatest risk for development of testicular cancer.d.Men of European descent are four times more likely to develop testicular cancer than men of non-European descent.
  • The nurse is describing how to perform a testicular self-examination to a patient. Which of the following statements is most appropriate?a.“A good time to examine your testicles is just before you take a shower.”b.“If you notice an enlarged testicle or a painless lump, call your health care provider.”c.“The testicle is egg shaped and movable. It feels firm and has a lumpy consistency.”d.“Perform a testicular exam at least once a week to detect the early stages of testicular cancer.”
  • A 2-month-old uncircumcised boy has been brought to the clinic for a well-baby checkup. How would the nurse proceed with his genital examination?a.Elicit the cremasteric reflex.b.Assess the glans for redness or lesions.c.Avoid retracting the foreskin until the infant is 3 months old.d.Note any dirt or smegma that has collected under the foreskin.
  • A 2-year-old boy has been diagnosed with “physiological cryptorchidism.” Given this diagnosis, during assessment, the nurse will most likely observe:a.testes that are hard and painful to palpation.b.an atrophic scrotum and absence of the testis bilaterally.c.an absence of the testis in the scrotum, but the testis can be milked down.d.testes that migrate into the abdomen when the child squats or sits cross-legged.
  • The nurse knows that a common assessment finding in a boy under 2 years old is:a.an inflamed and tender spermatic cord.b.the presence of a hernia in the scrotum.c.a penis that looks large in relation to the scrotum.d.the presence of a hydrocele, or fluid in the scrotum.
  • During an examination of an aging male, the nurse recognizes that a normal, expected change would be:a.a change in scrotal colour.b.a decrease in the size of the penis.c.enlargement of the testes and scrotum.d.an increase in the number of rugae over the scrotal sac.
  • When performing a genital assessment on a 34-year-old man, the nurse notes cauliflower-like patches of multiple soft, moist, painless scattered across the shaft of the penis. These lesions are characteristic of:a.carcinoma.b.syphilitic chancres.c.herpes progenitalis.d.genital warts
  • A 15-year-old boy is seen in the clinic for a complaint of “dull pain and pulling” in the scrotal area. On examination, the nurse palpates a soft, irregular mass posterior to and above the testis on the left. This mass collapses when the patient is supine and refills when he is upright. This description is consistent with:a.epididymitis.b.spermatocele.c.testicular torsion.d.a spermatic cord varicocele.
  • When performing a genitourinary assessment on a 16-year-old boy, the nurse notices a swelling in the scrotum that increases with intensified intra-abdominal pressure and decreases when he is lying down. The patient complains of pain when straining. The nurse knows that this description is most consistent with a(n):a.femoral hernia.b.incisional hernia.c.direct inguinal hernia.d.indirect inguinal hernia.
  • Which of the following findings is considered normal when the nurse is performing a testicular examination on a 25-year-old man?a.Nontender subcutaneous plaquesb.A scrotal area that is dry, scaly, and nodularc.Testes that feel oval and movable, and are slightly sensitive to compressiond.A single, hard, circumscribed, movable mass, less than 1 cm under the surface of the testes
  • The nurse is inspecting the scrotum and testes of a 43-year-old man. Which finding would require additional follow-up and evaluation?a.The skin on the scrotum is shiny and smooth.b.The left testicle hangs lower than the right testicle.c.The scrotum is a darker colour than the general skin colour.d.The testes move closer to the body in response to cold temperatures.
  • A 55-year-old man is experiencing severe, sudden onset pain onset in the scrotal area. It is somewhat relieved by elevation. On examination, the nurse notes an enlarged, red scrotum that is very tender to palpation. It is difficult to distinguish the epididymis from the testis, and the scrotal skin is thick and edematous. This description is consistent with which of the following?a.Varicoceleb.Epididymitisc.Spermatoceled.Testicular torsion
  • The nurse is performing a genitourinary assessment on a 50-year-old obese male labourer. On examination, the nurse notes a painless round swelling close to the pubis, in the area of the internal inguinal ring, which is easily reduced when the individual is supine. These findings are most consistent with which of the following conditions?a.A scrotal herniab.A femoral herniac.A direct inguinal herniad.An indirect inguinal hernia
  • Prolonged, painful erection of the penis without sexual desire is known as:a.orchitis.b.stricture.c.phimosis.d.priapism.
  • During an examination, the nurse notes that a male patient has a red, round, superficial ulcer with a yellowish serous discharge on his penis. On palpation, the nurse finds a nontender base that feels like a small button between the thumb and fingers. At this point the nurse suspects that this patient has:a.genital warts.b.a herpes infection.c.a syphilitic chancre.d.a carcinoma lesion.
  • During a health history, a patient tells the nurse that he has trouble in starting his urine stream. This problem is known as:a.urgency.b.dribbling.c.frequency.d.hesitancy.
  • During a genital examination, the nurse notes that a male patient has clusters of small vesicles on his glans, surrounded by erythema. The nurse recognizes that these lesions are:a.Peyronie’s disease.b.genital warts.c.genital herpes.d.syphilitic cancer.
  • During a physical examination, the nurse finds that a male patient’s foreskin is fixed and tight, and will not retract over the glans. The nurse recognizes that this condition is:a.phimosis.b.epispadias.c.urethral stricture.d.Peyronie’s disease.
  • The two parts of the nervous system are:a.motor and sensory.b.central and peripheral.c.peripheral and autonomic.d.hypothalamus and cerebral.
  • The wife of a 65-year-old man tells the nurse that she is concerned because she has noted a change in her husband’s personality and ability to understand. He also cries and becomes angry very easily. The nurse recalls that the cerebral lobe responsible for these behaviours is which of the following?a.Frontalb.Parietalc.Occipitald.Temporal
  • Which of the following statements concerning areas of the brain is accurate?a.The cerebellum is the center for speech and emotions.b.The hypothalamus controls temperature and regulates sleep.c.The basal ganglia are responsible for controlling voluntary movements.d.Motor pathways of the spinal cord and brainstem synapse in the thalamus.
  • The area of the nervous system that is responsible for mediating reflexes is the:a.medulla.b.cerebellum.c.spinal cord.d.cerebral cortex.
  • While gathering equipment for an intravenous injection, a nurse accidentally received a prick from an improperly capped needle. To interpret this sensation, which of the following areas must be intact?a.Corticospinal tract, medulla, and basal gangliab.Pyramidal tract, hypothalamus, and sensory cortexc.Lateral spinothalamic tract, thalamus, and sensory cortexd.Anterior spinothalamic tract, basal ganglia, and sensory cortex
  • A patient with a lack of oxygen to his heart will have pain in his chest, and also possibly in his shoulders, arms, or jaw. Which of the following best explains why this occurs?a.There is a problem with the sensory cortex and its ability to discriminate the location.b.The lack of oxygen in his heart has resulted in decreased amount of oxygen to these areas.c.The sensory cortex does not have the ability to localize pain in the heart, so the pain is felt elsewhere.d.There is a lesion in the dorsal root that is preventing the sensation from being transmitted normally.
  • The human ability to perform very skilled movements such as writing is controlled by the:a.basal ganglia.b.corticospinal tract.c.spinothalamic tract.d.extrapyramidal tract.
  • A 30-year-old woman tells the nurse that she has been very unsteady and has had difficulty in maintaining her balance. With these findings, which area of the brain would concern the nurse?a.Thalamusb.Brainstemc.Cerebellumd.Extrapyramidal tract
  • Which of the following statements about the peripheral nervous system is correct?a.The cranial nerves enter the brain through the spinal cord.b.Efferent fibres carry sensory input to the central nervous system through the spinal cord.c.The peripheral nerves are inside the central nervous system and carry impulses through their motor fibres.d.The peripheral nerves carry input to the central nervous system by afferent fibres and away by efferent fibres.
  • A patient has a severed spinal nerve as a result of trauma. Which of the following statements is true in this situation?a.Because there are 31 pairs of spinal nerves, there is no effect if only one is severed.b.The dermatome served by this nerve will no longer experience any sensation.c.The adjacent spinal nerves will continue to carry sensations for the dermatome served by the severed nerve.d.This will only affect the motor function of the patient, because spinal nerves have no sensory component.
  • A 21-year-old patient has a head injury resulting from trauma, and is unconscious. There are no other injuries. During the assessment, what would the nurse expect to find when testing the patient’s deep tendon reflexes?a.Reflexes will be normal.b.Reflexes will not be able to be elicited.c.All reflexes would be diminished, but present.d.Some reflexes would be present, depending on the area of injury.
  • A mother of a 1-month-old infant asks the nurse why it takes so long for babies to learn to roll over. The nurse knows that the reason for this is that:a.there must be a demyelinating process occurring with the baby.b.myelin is needed to conduct these impulses, and the neurons of a newborn are not myelinated.c.the cerebral cortex is not fully developed, so control over motor function occurs gradually.d.the spinal cord is controlling the movement because the cerebellum is not yet fully developed.
  • During an assessment of an 80-year-old patient, the nurse notes the patient’s inability to identify vibrations at the ankle and to identify the position of the big toe; a slower and more deliberate gait; and slightly impaired tactile sensation. All other neurological findings are normal. The nurse knows that these findings indicate:a.cranial nerve dysfunction.b.a lesion in the cerebral cortex.c.normal changes due to aging.d.demyelinization of nerves due to a lesion.
  • A 70-year-old woman tells the nurse that every time she gets up in the morning or after she’s been sitting, she gets “really dizzy” and feels like she is going to “fall over.” The nurse’s best response would be:a.“Have you been extremely tired lately?”b.“You probably just need to drink more liquids.”c.“I’ll refer you for a complete neurological examination.”d.“You need to get up slowly when you’ve been lying or sitting.”
  • During the history, a patient tells the nurse that “it feels like the room is spinning around me.” The nurse would document this as:a.vertigo.b.syncope.c.dizziness.d.seizure activity.
  • When doing the history on a patient with a seizure disorder, the nurse assesses whether the patient has an aura. Which of the following would be the best question to ask in order to obtain this information?a.“Does your muscle tone seem tense or limp?”b.“After the seizure, do you spend a lot of time sleeping?”c.“Do you have any warning sign before your seizure starts?”d.“Do you experience any colour change or incontinence during the seizure?”
  • While obtaining a history of a 3-month old infant from its mother, the nurse asks about the baby’s ability to suck and grasp the mother’s finger. What is the nurse assessing?a.Reflexesb.Intelligencec.Cranial nervesd.Cerebral cortex function
  • In obtaining a history for a 74-year-old patient, the nurse notes that he drinks alcohol daily, and that he has noticed a tremor in his hands that affects his ability to hold things. With this information, what should the nurse’s response be?a.“Does your family know you are drinking every day?”b.“Does the tremor change when you drink the alcohol?”c.“We’ll do some tests to see what is causing the tremor.”d.“You really shouldn’t drink so much alcohol; it may be causing your tremor.”
  • A 50-year-old woman is visiting the clinic for “weakness in my left arm and leg for the past week.” The nurse will perform which type of neurological examination?a.Glasgow Coma Scaleb.Neurological recheck examinationc.Screening neurological examinationd.Complete neurological examination
  • During an assessment of a patient’s cranial nerves, the nurse finds a lack of blink in the right eye with corneal reflex; intact ability to sense light touch on the face; and loss of movement with facial features on the right side. This would indicate dysfunction of which of the following cranial nerves?a.Motor component of IVb.Motor component of VIIc.Motor and sensory components of XId.Motor component of X and sensory component of VII
  • The nurse is testing the function of cranial nerve XI. Which of the following best describes the response the nurse would expect if the nerve is intact?a.Demonstrates full range of motion of the neckb.Sticks tongue out midline without tremors or deviationc.Follows an object with the eyes without nystagmus or strabismusd.Moves the head and shoulders against resistance with equal strength
  • During the neurological assessment of a “healthy” 35-year-old patient, the nurse asks him to relax his muscles completely. The nurse then moves each extremity through the full range of motion. Which of the following would the nurse expect to find?a.Firm, rigid resistance to movementb.Mild, even resistance to movementc.Hypotonic muscles as a result of total relaxationd.Slight pain with some directions of movement
  • When the nurse asks a 68-year-old patient to stand with his feet together, his arms at his sides, and his eyes closed, he starts to sway and moves his feet further apart. The nurse would document this finding as a(n):a.ataxia.b.lack of coordination.c.negative Homan’s sign.d.positive Romberg’s sign.
  • The nurse is doing an assessment on a 29-year-old woman who visits the clinic complaining of “always dropping things and falling down.” While testing rapid alternating movements, the nurse notices that the woman is unable to pat both her knees. Her response is very slow, and she misses frequently. What might the nurse suspect?a.Vestibular diseaseb.A lesion of cranial nerve IXc.Dysfunction of the cerebellumd.Inability to understand directions
  • The nurse is taking the health history of a 78-year-old man. During the history, his wife states that he occasionally has problems with short-term memory loss and confusion: “He can’t even remember how to button his shirt.” In doing the assessment of his sensory system, the nurse would do which of the following?a.The nurse would not do this part of the examination because results would not be valid.b.The nurse would perform the tests, knowing that mental status does not affect sensory ability.c.The nurse would proceed with the explanations of each test, making sure the wife understands.d.Before testing, the nurse would assess the patient’s mental status and ability to follow directions at this time.
  • The assessment of a 60-year-old patient has taken longer than anticipated. In testing his pain perception, the nurse decides to complete the test as quickly as possible. When the nurse applies the sharp point of the pin to his arm several times, he is only able to identify these as one “very sharp prick.” What would be the most accurate explanation for this?a.The patient has hyperesthesia as a result of the aging process.b.This is most likely the result of the summation effect.c.The nurse was probably not poking hard enough with the pin in the other areas.d.The patient most likely has analgesia in some areas of his arm and hyperalgesia in others.
  • The nurse is performing a neurological assessment for a 41-year-old woman with a history of diabetes. When testing her ability to feel the vibrations of a tuning fork, the nurse notes that she is unable to feel vibrations on the great toe or ankle bilaterally, but is able to feel vibrations on both patellae. Given this information, what would the nurse suspect?a.Hyperalgesiab.Hyperesthesiac.Peripheral neuropathyd.Lesion of sensory cortex
  • The nurse places a key in the hand of a patient, and he identifies it as a penny. What term would the nurse use to describe this finding?a.Extinctionb.Astereognosisc.Graphesthesiad.Tactile discrimination
  • The nurse is testing the deep tendon reflexes of a 30-year-old woman during her annual physical examination. When striking the Achilles and quadriceps, the nurse is unable to elicit reflexes. The nurse’s next response should be to:a.ask the patient to lock her fingers and “pull.”b.complete the examination, and then test these reflexes again.c.refer the patient to a specialist for further testing.d.document these reflexes as “0” on a scale of 0 to 4+.
  • In assessing a 70-year-old patient who has had a recent cerebrovascular accident, the nurse notes right-sided weakness. What might the nurse expect to find when testing his reflexes on the right side?a.Lack of reflexesb.Normal reflexesc.Diminished reflexesd.Hyperactive reflexes
  • When the nurse is testing the triceps reflex, what is the expected response?a.Flexion of the handb.Pronation of the handc.Extension of the forearmd.Flexion and supination of the forearm
  • The nurse is testing superficial reflexes on an adult patient. When stroking up the lateral side of the sole and across the ball of the foot, the nurse notices the plantar flexion of the toes. How would the nurse document this finding?a.Positive Babinski signb.Plantar reflex abnormalc.Plantar reflex presentd.Plantar reflex “2+” on a scale from “0 to 4+”
  • In the assessment of a 1-month-old infant, the nurse notes a lack of response to noise or stimulation. The mother reports that in the last week, he has been sleeping most of the time, and when awake, all he does is cry. The nurse hears that the infant’s cries are very high pitched and shrill. What would be the nurse’s appropriate response to these findings?a.Refer the infant for further testing.b.Talk with the mother about eating habits.c.Nothing; these are expected findings for an infant of this age.d.Tell the mother to bring the baby back in a week for a recheck.
  • Which of the following would the nurse use to test the motor coordination of an 11-month old infant?a.Denver II testb.Stereognosisc.Deep tendon reflexesd.Rapid alternating movements
  • To assess the head control of a 4-month-old infant, the nurse lifts the infant up in a prone position while supporting his chest. What normal response does the nurse look for?a.Raises head and arches backb.Extends arms and drops head downc.Flexes knees and elbows and keeps back straightd.Holds head at 45 degrees and keeps back straight
  • While assessing a 7-month-old infant, the nurse makes a loud noise and notes abduction and flexion of the arms and legs; fanning of the fingers, and curling of the index finger and thumb in a “C” position; and the infant bringing in its arms and legs towards its body. What does the nurse know about this response?a.This could indicate brachial nerve palsy.b.This is an expected startle response at this age.c.This reflex should have disappeared between 1 and 4 months of age.d.It is normal as long as movements are symmetric bilaterally.
  • To test for gross motor skill and coordination in a 6-year-old, which of the following techniques would be appropriate?a.Ask child to hop on one foot.b.Have the child stand on his head.c.Have the child touch his finger to his nose.d.Have the child make “funny” faces at the nurse.
  • During the assessment of an 80-year-old patient, the nurse notes that his hands show tremors when he reaches for something and that his head is always nodding. There is no associated rigidity with movement. Which of the following statements is most accurate?a.These are normal findings resulting from aging.b.These could be related to hyperthyroidism.c.These are the result of degenerative arthropathy.d.This patient should be evaluated for a cerebellar lesion.
  • While the nurse is taking the health history of a 68-year-old patient who sustained a head injury 3 days earlier, he tells the nurse that he is on a cruise ship and is 30 years old. The nurse knows that this finding is indicative of:a.a great sense of humour.b.uncooperative behaviour.c.inability to understand the question.d.a decreased level of consciousness.
  • The nurse is caring for a patient who has just had neurosurgery. To assess for increased intracranial pressure, what would the nurse include in the assessment?a.Cranial nerves, motor function, and sensory functionb.Deep tendon reflexes, vital signs, and coordinated movementsc.Level of consciousness, motor function, pupillary response, and vital signsd.Mental status, deep tendon reflexes, sensory function, and pupillary response
  • During an assessment of a 22-year-old woman who has a head injury from a car accident 4 hours ago, the nurse notes the following change: her pupils were equal, but now the right is fully dilated and nonreactive, while the left is 4 mm and reacts to light. What would this finding suggest?a.Injury to the right eyeb.Increased intracranial pressurec.The test was not performed accuratelyd.A normal response after a head injury
  • A 32-year-old woman tells the nurse that she has noticed “very sudden, jerky movements”, mainly in her hands and arms. “They seem to come and go, primarily when I am trying to do something. I haven’t noticed them when I’m sleeping.” This description suggests:a.chorea.b.athetosis.c.myoclonus.d.Parkinson’s disease.
  • During an assessment of a 62-year-old man the nurse notes stooped posture; shuffling, short steps when walking; a very rigid; flat facial expression; and movement of the fingers as if rolling a pill with them. These findings would be consistent with:a.parkinsonism.b.cerebral palsy.c.cerebellar ataxia.d.muscular dystrophy.
  • During an assessment of a 32-year-old patient with a recent head injury, the nurse notes that the patient responds to pain by extending, adducting, and internally rotating his arms. As well, his palms pronate and his lower extremities extend with plantar flexion. Which of the following statements about these findings is accurate?a.This indicates a lesion of the cerebral cortex.b.This indicates a completely nonfunctional brainstem.c.This is a normal response, and it will go away in 24 to 48 hours.d.This is a very ominous sign that may indicate brainstem injury.
  • A 78-year-old man has a history of a cerebrovascular accident. The nurse notes that when he walks, his left arm is immobile against the body, with flexion of the shoulder, elbow, wrist, and fingers and adduction of the shoulder. His left leg is stiff and extended, and circumducts with each step. What type of gait disturbance is this individual experiencing?a.Scissors gaitb.Cerebellar ataxiac.Parkinsonian gaitd.Spastic hemiparesis
  • In a person with an upper motor neuron lesion such as a cerebrovascular accident, which of the following physical assessment findings would the nurse expect to see?a.Hyperreflexiab.Fasciculationsc.Loss of muscle tone and flaccidityd.Atrophy and wasting of the muscles
  • A 59-year-old patient has a herniated intervertebral disk. Which of the following findings would the nurse expect to see on physical assessment of this individual?a.Hyporeflexiab.Increased muscle tonec.A positive Babinski’s signd.The presence of pathological reflexes
  • A patient is not able to perform rapid, alternating movements such as patting her knees quickly. The nurse would document this as:a.ataxia.b.astereognosis.c.the presence of dysdiadochokinesia.d.a probable abnormality in the cerebellum.
  • The nurse knows that determining whether a person is oriented to his or her surroundings will test the functioning of which of the following structures?a.Cerebrumb.Cerebellumc.Cranial nervesd.Medulla oblongata
  • During an examination, the nurse notes severe nystagmus in both of the patient’s eyes. Which of the following conclusions is correct?a.This is a normal occurrence.b.This may indicate disease of the cerebellum or brainstem.c.This is a sign that the patient is nervous about the examination.d.This indicates a visual problem and a referral to an ophthalmologist is indicated.
  • The nurse knows that testing kinesthesia is a test of a person’s:a.fine touch.b.position sense.c.motor coordination.d.perception of vibration.
  • The nurse knows that which of the following scores would indicate that a patient is in a coma based on the Glasgow Coma Scale criteria?a.6b.12c.15d.24
  • A man who was found wandering in a park at 2 a.m. has been brought to the emergency department for an examination because he said he fell and hit his head. During the examination, the nurse asks him to use his index finger to touch the nurse’s finger, then his own nose, and then the nurse’s finger again (which has been moved to a different location). The patient is clumsy, unable to follow the instructions, and overshoots the mark, missing the finger. The nurse suspects which of the following?a.Cerebral injuryb.Cerebrovascular accidentc.Acute alcohol intoxicationd.Peripheral neuropathy
  • The nurse is assessing the neurological status of a patient who has a late-stage brain tumour. With the reflex hammer, the nurse draws a light stroke up the lateral side of the sole of the foot and inward across the ball of the foot. In response, the patient’s toes fan out, and the big toe shows dorsiflexion. The nurse recognizes this as which of the following?a.A negative Babinski’s sign, which is normal for adultsb.A positive Babinski’s sign, which is abnormal for adultsc.Clonus, a hyperactive responsed.The Achilles reflex, an expected response
  • A patient is being assessed for his range of joint movement. The nurse asks him to move his arm in toward the centre of his body. This movement is called:a.flexion.b.abduction.c.adduction.d.extension.
  • A patient tells the nurse that she is having a hard time bringing her hand to her mouth when she eats or tries to brush her teeth. The nurse knows that for the patient to move her hand to her mouth, she must perform the following movement:a.flexion.b.abduction.c.adduction.d.extension.
  • The functional units of the musculoskeletal system are the:a.joints.b.bones.c.muscles.d.tendons.
  • Hematopoiesis takes place in the:a.liver.b.spleen.c.kidneys.d.bone marrow.
  • Fibrous bands running directly from one bone to another that strengthen the joint and help prevent movement in undesirable directions are called:a.bursa.b.tendons.c.cartilage.d.ligaments.
  • The nurse notices that a woman in an exercise class is unable to jump rope. The nurse knows that to jump rope, one’s shoulder has to be capable of:a.inversion.b.supination.c.protraction.d.circumduction.
  • The articulation of the mandible and the temporal bone is known as the:a.intervertebral foramen.b.condyle of the mandible.c.temporomandibular joint.d.zygomatic arch of the temporal bone.
  • To palpate the temporomandibular joint, the nurse’s fingers should be placed in the depression:a.distal to the helix of the ear.b.proximal to the helix of the ear.c.anterior to the tragus of the ear.d.posterior to the tragus of the ear.
  • Of the 33 vertebrae in the spinal column, there are:a.5 lumbar vertebrae.b.5 thoracic vertebrae.c.7 sacral vertebrae.d.12 cervical vertebrae.
  • An imaginary line connecting the highest point on each iliac crest would cross:a.the first sacral vertebra.b.the fourth lumbar vertebra.c.the seventh cervical vertebra.d.the twelfth thoracic vertebra.
  • The nurse is explaining to a patient that there are “shock absorbers” in his back to cushion the spine and to help it move. The nurse is referring to his:a.costal facets.b.nucleus pulposus.c.vertebral foramen.d.intervertebral discs.
  • The nurse is providing patient education for a man who has been diagnosed with a rotator cuff injury. The nurse knows that this injury involves:a.nucleus pulposus.b.the articular process.c.the medial epicondyle.d.the glenohumeral joint.
  • During an interview, a patient states, “I can feel this bump on the top of both of my shoulders—it doesn’t hurt but I am curious about what it might be.” The nurse should tell her:a.“That is your subacromial bursa.”b.“That is your acromion process.”c.“That is your glenohumeral joint.”d.“That is the greater tubercle of your humerus.”
  • The nurse is checking the range of motion in a patient’s knee, and knows that the knee is capable of the following movements:a.Flexion and extensionb.Supination and pronationc.Circumductiond.Inversion and eversion
  • A patient is visiting the clinic for an evaluation of a swollen, painful knuckle. The nurse notices that the knuckle above his wedding ring on the left hand is swollen, and he is unable to remove the ring. This joint is called the:a.interphalangeal joint.b.tarsometatarsal joint.c.metacarpophalangeal joint.d.talocalcaneonavicular joint.
  • The nurse is assessing a patient’s ischial tuberosity. To palpate the ischial tuberosity, the nurse knows that it is best to have the patient:a.stand.b.flex his hip.c.flex his knee.d.lie in the supine position.
  • The knee joint is the articulation of the femur, the tibia, and the:a.fibula.b.radius.c.patella.d.humerus.
  • The ankle joint is the articulation of the tibia, the fibula, and the:a.talus.b.cuboid.c.calcaneus.d.talocalcaneonavicular joint.
  • The nurse is explaining the growth of long bones to a mother of a toddler. The nurse knows that bones increase in width or diameter by deposition of new bony tissue around the shafts. The nurse also knows that lengthening occurs at the:a.bursa.b.calcaneus.c.epiphyses.d.tuberosities.
  • A woman who is 8 months pregnant comments that she has noticed a change in her posture and that she is having low back pain. The nurse tells her that during pregnancy, women have a posture shift to compensate for the enlarging fetus. This shift in posture is known as:a.lordosis.b.scoliosis.c.ankylosis.d.kyphosis.
  • An 85-year-old patient comments during his annual physical that he seems to be getting shorter as he ages. The nurse should explain that decreased height occurs with aging because:a.long bones tend to shorten with age.b.of the shortening of the vertebral column.c.there is a significant loss of subcutaneous fat.d.there is a thickening of the intervertebral discs.
  • A patient has been diagnosed with osteoporosis and asks the nurse, “What is osteoporosis?” The nurse knows that osteoporosis can be defined as:a.the loss of bone matrix.b.the loss of bone density.c.new, weaker bone growth.d.increased phagocytic activity.
  • The nurse knows that the incidence of osteoporosis is greatest in which group?a.Males of African descentb.Females of African descentc.Females of European descentd.Aboriginal males
  • A teenage girl is complaining of pain in her left wrist. She had been playing basketball and fell, landing on her left hand. The nurse examines her hand, and would expect a fracture if the girl complains:
  • A patient is complaining of pain in his joints that is worse in the morning, is better after he has moved around for awhile, and then gets worse again if he sits for long periods of time. The nurse suspects that he may have:a.tendinitis.b.osteoarthritis.c.rheumatoid arthritis.d.intermittent claudication.
  • A patient states, “I can hear a crunching or grating sound when I kneel.” She also states, “it is very difficult to get out of bed in the morning because of stiffness and pain in my joints.” The nurse suspects that the sound the patient hears is:a.crepitation.b.a bone spur.c.a loose tendon.d.fluid in the knee joint.
  • A patient is able to flex his right arm forward without difficulty or pain, but is unable to abduct his arm because of pain and muscle spasms. The nurse suspects:a.crepitation.b.rotator cuff lesions.c.dislocated shoulder.d.rheumatoid arthritis.
  • A professional tennis player comes into the clinic complaining of a sore elbow. The nurse suspects that he has tenderness at the:a.olecranon bursa.b.annular ligament.c.base of the radius.d.medial and lateral epicondyle.
  • The nurse suspects that a patient has carpal tunnel syndrome and wants to perform the Phalen’s test. To perform this test, the nurse will instruct the patient to:a.dorsiflex the foot.b.plantarflex the foot.c.hold both hands back to back while flexing the wrists 90 degrees for 60 seconds.d.hyperextend the wrists with the palmar surface of both hands touching and wait for 60 seconds.
  • An 80-year-old woman is visiting the clinic for a checkup. The nurse is observing for motor dysfunction in the woman’s hip, and would have her:a.internally rotate her hip while she is sitting.b.abduct her hip while she is lying on her back.c.adduct her hip while she is lying on her back.d.externally rotate her hip while she is standing.
  • The nurse has completed the musculoskeletal examination of a patient’s knee and has found a positive bulge sign. The nurse suspects:a.irregular bony margins.b.soft tissue swelling in the joint.c.swelling from fluid in the epicondyle.d.swelling from fluid in the suprapatellar pouch.
  • During an examination, the nurse asks a patient to bend forward from the waist and notes that the patient has lateral tilting. When one of his legs is raised straight up, he complains of a pain going down the buttock into the leg. The nurse suspects:a.scoliosis.b.a meniscus tear.c.a herniated nucleus pulposus.d.a spasm of paravertebral muscles.
  • The nurse is examining a 3-month-old infant. While holding her thumbs on the infant’s inner mid thighs and her fingers outside on the hips, touching the greater trochanter, the nurse adducts the infant’s legs until her thumbs touch and then abducts the legs until the knees touch the table. The nurse does not note any “clunking” sounds, and is confident to record a:a.positive Allis sign.b.negative Allis sign.c.positive Ortolani sign.d.negative Ortolani sign.
  • During a neonatal examination, the nurse notes that the newborn infant has six toes. This finding is documented as:a.unidactyly.b.syndactyly.c.polydactyly.d.multidactyly.
  • A mother brings her newborn baby boy in for a checkup. She tells the nurse that he doesn’t seem to be moving his right arm as much as his left, and that he seems to have pain when she lifts him up under the arms. The nurse suspects a fractured clavicle, and would observe for:a.a negative Allis sign.b.a positive Ortolani sign.c.limited range of motion during the Moro reflex.d.limited range of motion during LaSegue’s reflex.
  • A 40-year-old man has come into the clinic with complaints of “extreme tenderness in my toes.” The nurse notes that his toes are slightly swollen, reddened, and warm to the touch. His complaints would suggest:a.osteoporosis.b.acute gout.c.ankylosing spondylitis.d.degenerative joint disease.
  • A young swimmer comes to the sports clinic complaining of a very sore shoulder. He was running at the pool, slipped on some wet concrete, and tried to catch himself with his outstretched hand. He landed on this hand, and has not been able to move his shoulder since. The nurse suspects:a.joint effusion.b.tear of rotator cuff.c.adhesive capsulitis.d.dislocated shoulder.
  • A 68-year-old woman has come in for an assessment of her rheumatoid arthritis, and the nurse notes raised, firm, nontender nodules at the olecranon bursa and along the ulna. These nodules are most commonly diagnosed as:a.epicondylitis.b.gouty arthritis.c.olecranon bursitis.d.subcutaneous nodules.
  • A woman who has had rheumatoid arthritis for years is starting to notice that her fingers are drifting to the side. The nurse knows that this condition is commonly referred to as:a.radial drift.b.ulnar deviation.c.swan neck deformity.d.Dupuytren’s contracture.
  • A patient who has had rheumatoid arthritis for years comes to the clinic to ask about changes in her hands. The nurse knows that changes associated with rheumatoid arthritis include:a.Heberden’s nodes.b.Bouchard’s nodules.c.swan-neck deformities.d.Dupuytren’s contractures.
  • A patient’s annual physical examination reveals a lateral curvature of the thoracic and lumbar segments of his spine; however, this curvature disappears with forward bending. The nurse knows that this abnormality of the spine is called:a.structural scoliosis.b.functional scoliosis.c.herniated nucleus pulposus.d.dislocated nucleus pulposus.
  • A 14-year-old boy who has been diagnosed with Osgood-Schlatter disease reports painful swelling just below the knee for the past 5 months. Which response by the nurse is appropriate?a.“If these symptoms persist, you may need arthroscopic surgery.”b.“You are experiencing degeneration of your knee, which may not resolve.”c.“Your disease is due to repeated stress on the patellar tendon. It is usually self-limited, and your symptoms should resolve with rest.”d.“Increasing your activity and performing knee-strengthening exercises will help to decrease the inflammation and maintain mobility in the knee.”
  • When assessing muscle strength, the nurse observes that a patient has complete range of motion against gravity with full resistance. What should the nurse record using a 0 to 5+ scale?a.2+b.3+c.4+d.5+
  • The nurse is examining a 6-month-old baby and places the baby’s feet flat on the table and flexes his knees up. The nurse notes that the right knee is significantly lower than the left. Which of the following is true of this finding?a.This is a positive Allis sign and suggests hip dislocation.b.The infant probably has a dislocated patella on the right knee.c.This is a normal finding for the Allis test for an infant of this age.d.The infant should return to the clinic in 2 weeks to see if this has changed.
  • The nurse is assessing a 1-week-old infant and testing his muscle strength. The nurse lifts the infant with her hands under the axillae, and notes that the infant starts to “slip” between her hands. The nurse should:a.suspect a fractured clavicle.b.consider that the infant may have a deformity of the spine.c.suspect that the infant may have weakness of the shoulder muscles.d.consider this a normal finding because the musculature of an infant at this age is undeveloped.
  • The nurse is planning to measure a patient’s angles of joint flexion. What instrument will be used?a.Caliperb.Protracterc.Goniometerd.Measuring tape
  • Which test should the nurse use to check for large amounts of fluid around the patella?a.Ballottementb.Tinel’s signc.Phalen’s testd.McMurray’s test
  • The nurse knows that another term for “knock knees” is:a.genu varum.b.genu valgum.c.pes planus.d.metatarsus adductus.
  • A patient has been diagnosed with a ganglion cyst over the dorsum of his left wrist. He asks the nurse, “What is this thing?” The nurse’s best answer is:a.“It is a common benign tumour.”b.“It is a tumor that will have to be watched because it may turn malignant.”c.“It is caused by chronic repetitive motion injury.”d.“It is a skin infection that will need to be drained.”
  • A man who has had gout for several years comes to the clinic with a “problem with my toe.” On examination, the nurse notes the presence of hard, painless nodules over the great toe; one has burst open and released a chalky discharge. This finding is known as:a.a callus.b.a plantar wart.c.tenosynovitis.d.tophi.
  • When performing a musculoskeletal assessment, the nurse knows that the correct approach for the examination should be:a.proximal to distal.b.distal to proximal.c.posterior to anterior.d.anterior to posterior.
  • Which sound is normally elicited when percussing in the seventh right intercostal space at the midclavicular line over the liver?a.Dullnessb.Tympanyc.Resonanced.Hyperresonance
  • Which structure is located in the left lower quadrant of the abdomen?a.Liverb.Duodenumc.Gallbladderd.Sigmoid colon
  • A patient is having difficulty swallowing medications and food. The nurse would document that this patient has:a.aphasia.b.dysphasia.c.dysphagia.d.myophagia.
  • The nurse suspects that a patient has a distended bladder. How should the nurse assess for this condition?a.Percuss and palpate in the lumbar region.b.Inspect and palpate in the epigastric region.c.Auscultate and percuss in the inguinal region.d.Percuss and palpate in the hypogastric region.
  • Which of the following changes may occur in the gastrointestinal system of an aging adult?a.Increased salivationb.Increased esophageal emptyingc.Increased peristalsisd.Decreased gastric acid secretion
  • A 22-year-old man falls with his motorcycle, and lands on the handlebars on his left side. He comes to the clinic for an examination. The nurse suspects that he may have injured his spleen. Which of the following is true regarding assessment of the spleen in this situation?a.The spleen can be enlarged as a result of trauma.b.The spleen is normally felt upon routine palpation.c.If an enlarged spleen is noted, palpate it thoroughly to determine size.d.An enlarged spleen should not be palpated because it can rupture easily.
  • A patient’s abdomen appears bulging and stretched. The nurse would describe this finding as:a.obese.b.herniated.c.scaphoid.d.protuberant.
  • When viewed horizontally, a scaphoid contour of the abdomen displays:a.a flat profile.b.a convex profile.c.a bulging profile.d.a concave profile.
  • While examining a patient, the nurse observes abdominal pulsations between the xiphoid and umbilicus. The nurse would suspect that these are:a.pulsations of the renal arteries.b.pulsations of the inferior vena cava.c.normal abdominal aortic pulsations.d.increased peristalsis from a bowel obstruction.
  • The nurse knows that a potential cause of hypoactive bowel sounds is:a.diarrhea.b.peritonitis.c.laxative use.d.gastroenteritis.
  • The main reason auscultation precedes percussion and palpation of the abdomen is to:a.determine areas of tenderness.b.prevent distortion of bowel sounds.c.allow the patient more time to relax, and therefore be more comfortable with the physical examination.d.prevent distortion of vascular sounds, such as bruits and hums.
  • Which of the following is true of bowel sounds?a.They are usually loud, high-pitched, rushing, tinkling sounds.b.They are usually high-pitched, gurgling, irregular sounds.c.They sound like “two pieces of leather being rubbed together.”d.They originate from the movement of air and fluid through the large intestine.
  • The physician comments that a patient has abdominal “borborygmi.” The nurse knows that this term refers to:a.a loud, continuous hum.b.a peritoneal friction rub.c.hypoactive bowel sounds.d.hyperactive bowel sounds.
  • Which of the following is a normal finding in an abdominal assessment?a.The presence of a bruit in the femoral areab.A tympanic percussion note in the umbilical regionc.A palpable spleen between the ninth and eleventh ribs in the left midaxillary lined.A dull percussion note in the left upper quadrant at the midclavicular line
  • The nurse is assessing the abdomen of a pregnant woman, who is complaining of having a continuous “stomach ache”. The nurse knows that esophageal reflux during pregnancy can cause:a.diarrhea.b.pyrosis.c.dysphagia.d.constipation.
  • Percussion notes heard during an abdominal assessment may include:a.flatness, resonance, and dullness.b.resonance, dullness, and tympany.c.tympany, hyperresonance, and dullness.d.resonance, hyperresonance, and flatness.
  • A patient has been diagnosed with pernicious anemia. The nurse knows that this condition could be related to:a.increased gastric acid secretion.b.decreased gastric acid secretion.c.delayed gastrointestinal emptying time.d.increased gastrointestinal emptying time.
  • A patient is complaining of tenderness along the costovertebral angles. The nurse knows that this symptom is most often indicative of:a.ovary infection.b.liver enlargement.c.kidney inflammation.d.spleen enlargement.
  • Which of the following is present in a patient with ascites?a.Fluidb.Fecesc.Flatusd.Fibroid tumours
  • The nurse knows that during an abdominal assessment, deep palpation is used to determine:a.bowel motility.b.enlarged organs.c.superficial tenderness.d.overall impression of skin surface and superficial musculature.
  • The nurse notes that a patient has had a black, tarry stool. A possible cause would be:a.gallbladder disease.b.overuse of laxatives.c.gastrointestinal bleeding.d.localized bleeding around the anus.
  • Tenderness on light palpation in the right lower quadrant could indicate a disorder of which of the following structures?a.Spleenb.Sigmoidc.Appendixd.Gallbladder
  • Which of the following statements regarding abdominal assessment of the aging adult is true?a.The abdominal tone is increased.b.The abdominal musculature is thinner.c.The abdominal rigidity with acute abdominal conditions is more common.d.The aging person complains of more pain with an acute abdomen than a younger person would.
  • During an assessment of a newborn infant, the nurse recalls that pyloric stenosis would be manifested by:a.projectile vomiting.b.hypoactive bowel activity.c.a palpable, olive-sized mass in right lower quadrant.d.pronounced peristaltic waves crossing from right to left.
  • To detect diastasis recti, the nurse should have the patient perform which of the following manoeuvres?a.Relax in the supine position.b.Raise arms in the left lateral position.c.Raise arms over the head while supine.d.Raise the head while remaining supine.
  • Which of the following statements about aortic aneurysm is true?a.A bruit is absent.b.Femoral pulses are increased.c.A pulsating mass is usually present.d.Most are located below the umbilicus.
  • During an abdominal assessment, the nurse is unable to hear the patient’s bowel sounds. Before reporting this finding as “silent bowel sounds”, the nurse should listen for at least:a.1 minute.b.5 minutes.c.10 minutes.d.2 minutes in each quadrant.
  • A patient is suspected of having cholecystitis (inflammation of the gallbladder). The nurse will conduct which of the following to assess for this condition?a.Obturator testb.Murphy’s signc.Assessment for rebound tendernessd.Iliopsoas muscle test
  • Just before going home, a new mother asks the nurse about her baby’s umbilical cord. The nurse would tell her that:a.it should fall off by 10 to 14 days.b.at birth, the cord is a bluish colour.c.it contains two veins and one artery.d.skin will cover the area within 1 week.
  • Which of the following percussion findings would be found in a patient with a large amount of ascites?a.Dullness across the abdomenb.Flatness in the right upper quadrantc.Hyperresonance in the left upper quadrantd.Tympany in the right and left lower quadrants
  • A 40-year-old man states that his doctor told him that he has a hernia. He asks the nurse to explain what a hernia is. Which appropriate action can be taken by the nurse?a.Tell him not to worry, since most men his age develop hernias.b.Refer him to his physician for additional consultation, because he or she made the initial diagnosis.c.Explain that a hernia is a loop of bowel protruding through a weak spot in the abdominal muscles.d.Explain that hernias that occur in adulthood are often the result of prenatal growth abnormalities.
  • A 45-year-old man visits the clinic for a physical. During the abdominal assessment, the nurse percusses and notes a 10-cm area of dullness above the right costal margin. The nurse should:a.document the presence of hepatomegaly.b.ask additional history questions regarding the patient’s alcohol intake.c.describe this as an enlarged liver, and refer the patient to a physician.d.consider this a normal finding, and proceed with the examination.
  • When palpating the abdomen of a 20-year-old patient, the nurse notes the presence of tenderness with deep palpation in the left upper quadrant. Which of the following structures is most likely to be involved?a.Spleenb.Sigmoidc.Appendixd.Gallbladder
  • The incidence of lactose intolerance is higher in adults in which group?a.Canadian Aboriginalsb.Spanish descentc.European descentd.Canadians of northern European descent
  • The nurse is assessing a patient for possible peptic ulcer disease. Which condition often causes this problem?a.Hypertensionb.Streptococcus infectionsc.History of constipation and frequent laxative used.Frequent use of nonsteroidal anti-inflammatory drugs
  • While participating in patient assessment, the student nurse hears that a patient has “hepatomegaly”, and recognizes that this term refers to:a.an enlarged liver.b.an enlarged spleen.c.distended bowels.d.excessive diarrhea.
  • During an assessment, the nurse notes that a patient’s umbilicus is enlarged and everted. It is midline, and there is no change in skin color. The nurse recognizes that the patient may have which condition?a.Intra-abdominal bleedingb.Constipationc.Umbilical herniad.An abdominal tumor
  • During an abdominal assessment, the nurse tests for a fluid wave. A positive fluid wave test occurs with:a.splenomegaly.b.distended bladder.c.constipation.d.ascites.
  • The nurse is preparing to examine a patient who has been complaining of right lower quadrant pain. Which technique is correct during the assessment?a.The nurse should examine the tender area first.b.The nurse should examine the tender area last.c.The nurse should avoid palpating the tender area.d.The nurse should palpate the area first and then auscultate for bowel sounds.
  • Which of the following statements about the arterial system is true?a.Arteries are large-diameter vessels.b.The arterial system is a high-pressure system.c.The walls of arteries are thinner than those of veins.d.Arteries can expand greatly to accommodate a large blood volume increase.
  • The major artery supplying the arm is the:a.ulnar artery.b.radial artery.c.brachial artery.d.deep palmar artery.
  • To assess the dorsalis pedis artery, the nurse would palpate:a.behind the knee.b.over the lateral malleolus.c.in the groove behind the medial malleolus.d.lateral to the extensor tendon of the great toe.
  • A 65-year-old patient is experiencing pain in his left calf during exercise, which disappears after resting for a few minutes. The nurse recognizes that this description is most consistent with:a.venous obstruction of the left leg.b.claudication due to venous abnormalities in the left leg.c.ischemia caused by partial blockage of an artery supplying the left leg.d.ischemia caused by complete blockage of an artery supplying the left leg.
  • Which of the following statements best describes the mechanism(s) by which venous blood returns to the heart?a.Intraluminal valves ensure unidirectional flow toward the heart.b.Contracting skeletal muscles milk blood distally toward the veins.c.The high-pressure system of the heart helps to facilitate venous return.d.Increased thoracic pressure and decreased abdominal pressure facilitate venous return to the heart.
  • Which of the following veins are responsible for most of the venous return in the arm?a.Deep veinsb.Ulnar veinsc.Subclavian veinsd.Superficial veins
  • A 70-year-old patient is scheduled for open-heart surgery. The physicians plan to use the great saphenous vein for the coronary bypass grafts. The patient asks, “What happens to my circulation when the veins are removed?” The nurse should reply:a.“Venous insufficiency is a common problem after this type of surgery.”b.“Oh, there are lots of veins in your body—you won’t even notice that one has been removed.”c.“You will probably experience decreased circulation after the vein has been removed.”d.“Because the deeper veins in your leg are in good condition, this vein can be removed without harming your circulation.”
  • Which of the following situations best describes a person at risk for development of venous disease?a.A woman in her fifth month of pregnancyb.A person who has been in bed rest for 4 daysc.A person with a 30-year, 1-pack-per-day smoking historyd.An older adult taking anticoagulant medication (adsbygoogle = window.adsbygoogle || []).push({});
  • Which of the following statements about the lymphatic system is true?a.Lymph flow is propelled by the contraction of the heart.b.The flow of lymph is slow compared with that of blood.c.One of the functions of the lymph is to absorb lipids from the biliary tract.d.Lymph vessels have no valves, so there is a free flow of lymph from the tissue spaces into the bloodstream and back again.
  • During assessment of a patient, the nurse notes an enlarged right epitrochlear lymph node. What should the nurse do next?a.Palpate the patient’s abdomen, noting any tenderness.b.Carefully palpate the cervical lymph nodes, checking for any enlargement.c.Ask additional history questions regarding any recent ear infection or sore throat.d.Examine the patient’s lower arm and hand, checking for the presence of any infection or lesions.
  • A 35-year-old man is at the clinic for an “infection in my left foot.” Which of the following would the nurse expect to find during assessment of this patient?a.Hard and fixed cervical nodesb.Enlarged and tender inguinal nodesc.Bilateral enlargement of the popliteal nodesd.“Pellet-like” nodes in the supraclavicular region
  • The nurse is examining the lymphatic system of a healthy 3-year-old child. Which of the following would the nurse expect to find?a.Excessive swelling of the lymph nodesb.The presence of palpable lymph nodesc.No nodes palpable because of the immature immune system of a childd.Fewer numbers and decreased size of lymph nodes compared with an adult
  • Which of the following is a normal physiological change associated with the aging process?a.Hormonal changes causing vasodilation and a resulting drop in blood pressure.b.Progressive atrophy of the intramuscular calf veins, causing venous insufficiency.c.Peripheral blood vessels growing more rigid with age, producing a rise in systolic blood pressure.d.Narrowing of the inferior vena cava, causing low blood flow and increases in venous pressure, resulting in varicosities.
  • A 67-year-old patient states that he recently began experiencing pain in his left calf while climbing the 10 stairs to his apartment. This pain is relieved by sitting for about 2 minutes; then he is able to resume his activities. This patient is most likely experiencing:a.claudication.b.sore muscles.c.muscle cramps.d.venous insufficiency.
  • A patient complains of leg pain that wakes him up at night. He states that he “has been having problems” with his legs. He has pain in his legs when they are elevated, but the pain disappears when he dangles the legs. He recently noticed “a sore” on the inner aspect of the right ankle. This information would indicate that the patient is most likely experiencing:a.pain related to lymphatic abnormalities.b.problems related to arterial insufficiency.c.problems related to venous insufficiency.d.pain related to musculoskeletal abnormalities.
  • The nurse uses the “profile sign” to detect:a.pitting edema.b.early clubbing.c.symmetry of the fingers.d.insufficient capillary refill.
  • The nurse is performing well-child assessment on a 3-year-old child. The child’s vital signs are normal. Capillary refill time is 5 seconds. The nurse would:a.ask the parent if the child has had frostbite in the past.b.suspect that the child has a venous insufficiency problem.c.consider this a delayed capillary refill time and investigate further.d.consider this a normal capillary refill time that requires no further assessment.
  • When assessing a patient, the nurse documents the left femoral pulse as 0/0-4+. Which of the following findings would the nurse expect at the dorsalis pedis pulse?a.0/0-4+b.1+/0-4+c.2+/0-4+d.3+/0-4+
  • When performing peripheral vascular assessment on a patient, the nurse is unable to palpate the ulnar pulses. The patient’s skin is warm, and capillary refill time is normal. The nurse would next:a.check for the presence of claudication.b.refer the individual for further evaluation.c.consider this a normal finding and proceed with peripheral vascular evaluation.d.ask the patient if he or she has experienced any unusual cramping or tingling in the arm.
  • Which of the following pulses would most likely be a finding in an individual with untreated hyperthyroidism?a.A normal pulseb.An absent pulsec.A bounding pulsed.A weak, thready pulse
  • Why would the nurse would perform a modified Allen test?a.To measure the rate of lymphatic drainageb.To evaluate the adequacy of capillary patency before drawing venous bloodc.To evaluate the adequacy of collateral circulation before cannulating the radial arteryd.To evaluate the venous refill rate that occurs after the ulnar and radial arteries are temporarily occluded
  • The nurse is aware that a positive Homan’s sign in a patient:a.occurs with venous insufficiency.b.is indicative of possible thrombophlebitis.c.is seen in the presence of severe edema.d.indicates problems with arterial circulation.
  • A patient has been diagnosed with venous stasis. Which of the following would the nurse most likely observe?a.A cool footb.Thin, shiny, atrophic skinc.Pallor of the toes and cyanosis of the nailbedsd.Brownish discoloration of the skin on the lower leg
  • The nurse is attempting to assess the femoral pulse in an obese patient. Which of the following actions would be most appropriate?a.Have the patient assume a prone position.b.Ask the patient to bend the knees to the side in a froglike position.c.With the patient in a semi-Fowler’s position, press firmly against the bone.d.Listen with a stethoscope for pulsations because it is very difficult to palpate the pulse.
  • When auscultating over a patient’s femoral arteries, the nurse notes a bruit on the left side. The nurse knows that:a.bruits are often associated with venous disease.b.bruits occur in the presence of lymphadenopathy.c.hypermetabolic states will cause bruits in the femoral arteries.d.bruits occur with turbulent blood flow, indicating partial occlusion.
  • How would the nurse document mild, slightly pitting edema of the ankles of a pregnant patient?a.1+/0-4+b.3+/0-4+c.4+/0-4+d.Edema present
  • A patient has hard, nonpitting edema of the left lower leg and ankle. The nurse is aware that:a.nonpitting, hard edema occurs with lymphatic obstruction.b.alterations in arterial function will cause this edema.c.phlebitis of a superficial vein will cause bilateral edema.d.longstanding arterial obstruction will cause pitting edema.
  • When assessing a patient’s pulse, the nurse notes that the amplitude is weaker during inspiration and stronger during expiration. When the nurse measures the blood pressure, the reading decreases by 20 mm Hg during inspiration and increases with expiration. This patient is experiencing:a.pulsus alternans.b.pulsus bisferiens.c.pulsus bigeminus.d.pulsus paradoxus.
  • The nurse is performing peripheral vascular assessment on a bedridden patient, and notes the following findings in the right leg: increased warmth, swelling, redness, tenderness to palpation, and a positive Homan’s sign. The nurse would:a.re-evaluate the patient in a few hours.b.consider this a normal finding for a bedridden patient.c.seek emergency referral because of the risk of pulmonary embolism.d.ask the patient to raise his leg off of the bed and check for pain on elevation.
  • During assessment, the nurse has elevated a patient’s legs 30 cm (1 ft) off the table and has had him wiggle his feet to drain off venous blood. After helping him to sit up and dangle his legs over the side of the table, the nurse would expect that a normal finding at this point would be:a.marked elevational pallor.b.venous filling within 15 seconds.c.pain in the feet and lower legs when assuming a sitting position.d.colour returning to the feet within 20 seconds of assuming a sitting position.
  • A patient has bilateral pitting edema of the feet. While assessing the peripheral vascular system, the nurse’s primary focus should be:a.oxygenation of the lower extremities.b.arterial function of the lower extremities.c.venous function of the lower extremities.d.possible thrombophlebitis of the lower extremities.
  • Which of the following statements about the manual compression test is true?a.Rapid filling of the veins indicates incompetent veins.b.Competent valves in the veins will transmit a wave to the distal fingers.c.A palpable wave transmission occurs when the valves are incompetent.d.The test assesses whether the valves of varicosity are competent when the person is in the supine position.
  • During assessment, the nurse notes that a patient’s left arm is swollen from the shoulder down to the fingers, with nonpitting edema. The right arm is normal. The patient had a mastectomy 1 year ago. Which of the following problems does the nurse suspect?a.Venous stasisb.Lymphedemac.Arteriosclerosisd.Deep vein thrombosis
  • Which of the following statements about assessment of the ankle-brachial index (ABI) is true?a.Normal ABI indices are from 0.50 to 1.0.b.The normal ankle pressure is slightly lower than the brachial pressure.c.The ABI is a reliable measurement of peripheral vascular disease in diabetic individuals.d.An ABI of 0.90 to 0.70 indicates the presence of peripheral vascular disease and mild claudication.
  • The nurse is performing a well-child assessment on a 5-year-old boy. He has no current history that would lead the nurse to suspect any illness. His past medical history is unremarkable, and he received immunizations 1 week ago. Which of the following findings would be considered normal in this situation?a.Enlarged, warm, tender nodesb.Lymphadenopathy of the cervical nodesc.Palpable firm, small, shotty, mobile, nontender lymph nodesd.Firm, rubbery, large nodes, somewhat fixed to the underlying tissue
  • Which of the following sounds is heard when using a Doppler ultrasonic stethoscope to recognize venous flow?a.A low humming soundb.A regular “lub, dub” patternc.A swishing, whooshing soundd.A steady, even, flowing sound
  • When describing a weak, thready pulse, the nurse should document:a.“Easily palpable, pounds under the fingertips.”b.“Greater than normal force, then collapses suddenly.”c.“Hard to palpate, may fade in and out, easily obliterated by pressure.”d.“Rhythm is regular, but force varies with alternating beats of large and small amplitude.”
  • During assessment, a patient tells the nurse that her fingers often change colour when she goes out in cold weather. She describes these episodes as her fingers first turning white, then blue, then red with a burning, throbbing pain. The nurse suspects that she is experiencing:a.lymphedema.b.Raynaud’s syndrome.c.deep vein thrombosis.d.chronic arterial insufficiency.
  • During a routine visit to the clinic, a patient takes off his shoes and shows the nurse “this awful sore that won’t heal.” On examination, the nurse notes on the left great toe a 3-cm round ulcer with a pale ischemic base, well-defined edges, and no drainage. The nurse will assess for other signs and symptoms of:a.varicosities.b.a venous stasis ulcer.c.an arterial ischemic ulcer.d.pitting edema.
  • The nurse is reviewing the assessment of a patient’s peripheral pulses and notes that the radial pulses are “2+.” What type of pulse does this reading indicate?a.Boundingb.Normalc.Weakd.Absent
  • The sac that surrounds and protects the heart is called the:a.pericardium.b.myocardium.c.endocardium.d.pleural space.
  • Which of the following best describes the direction of blood flow through the heart?a.Vena cava → right atrium → right ventricle → lungs → pulmonary artery → left atrium → left ventricleb.Right atrium → right ventricle → pulmonary artery → lungs → pulmonary vein → left atrium → left ventriclec.Aorta → right atrium → right ventricle → lungs → pulmonary vein → left atrium → left ventricle → vena cavad.Right atrium → right ventricle → pulmonary vein → lungs → pulmonary artery → left atrium → left ventricle
  • Which of the following best describes what is meant by “atrial kick”?a.The atria contract during systole and attempt to push against closed valves.b.The contraction of the atria at the beginning of diastole can be felt as a palpitation.c.This is the pressure exerted against the atria as the ventricles contract during systole.d.The atria contract toward the end of diastole and push the remaining blood into the ventricles.
  • When listening to heart sounds, the nurse is aware that the valve closures that can be heard best at the base of the heart are:a.mitral, tricuspid.b.tricuspid, aortic.c.aortic, pulmonic.d.mitral, pulmonic.
  • Which of the following describes the closure of the valves in a normal cardiac cycle?a.The aortic valve closes slightly before the tricuspid valve does.b.The pulmonic valve closes slightly before the aortic valve does.c.The tricuspid valve closes slightly later than the mitral valve does.d.Both the tricuspid and pulmonic valves close at the same time.
  • The component of the conduction system referred to as the pacemaker of the heart is the:a.atrioventricular (AV) node.b.sinoatrial (SA) node.c.bundle of His.d.bundle branches.
  • Which sequence does the electrical stimulus of the cardiac cycle follow?a.AV node → SA node → bundle of Hisb.Bundle of His → AV node → SA nodec.SA node → AV node → bundle of His → bundle branchesd.AV node → SA node → bundle of His → bundle branches
  • Assessment of a 70-year-old patient with swelling in his ankles reveals jugular venous pulsations 5 cm above the sternal angle when the head of his bed is elevated 45 degrees. The nurse is aware that this indicates:a.decreased fluid volume.b.increased cardiac output.c.narrowing of jugular veins.d.increased pressure on the right side of his heart.
  • In the assessment of a newborn infant who is just 5 minutes old, which of the following would be true?a.The left ventricle is larger and weighs more than the right.b.The circulation of a newborn is identical to that of an adult.c.There is an opening in the atrial septum where blood can flow into the left side of the heart.d.The foramen ovale closes just minutes before birth and the ductus arteriosus closes immediately after.
  • A 25-year-old woman in her fifth month of pregnancy has a blood pressure of 100/70 mm Hg. In reviewing her previous examination findings, the nurse notes that her blood pressure in her second month was 124/80 mm Hg. In evaluating this change, what does the nurse know to be true?a.This is the result of peripheral vasodilatation and is an expected change.b.Because of increased cardiac output, the blood pressure should be higher this time.c.This is not an expected finding because it would mean a decreased cardiac output.d.This would mean a decrease in circulating blood volume, which is dangerous for the fetus.
  • In assessing a 70-year-old man, the nurse finds the following: BP 140/100 mm Hg; HR 104 and slightly irregular; split S2. Which of these findings can be explained by expected hemodynamic changes related to aging?a.Increase in resting heart rateb.Increase in systolic blood pressurec.Decrease in diastolic blood pressured.Increase in diastolic blood pressure
  • A 45-year-old man is in the clinic for “a routine physical.” During history taking, the patient states he has been having difficulty sleeping. “I’ll be sleeping great and then I wake up and feel like I can’t get my breath.” The nurse’s best response to this would be:a.“When was your last electrocardiogram?”b.“It’s probably because it’s been so hot at night.”c.“Do you have any history of problems with your heart?”d.“Have you had a recent sinus infection or upper respiratory infection?”
  • In assessing a patient’s major risk factors for heart disease, which of the following would the nurse want to include in the history?a.Family history, hypertension, stress, ageb.Personality type, high cholesterol, diabetes, smokingc.Smoking, hypertension, obesity, diabetes, high cholesterold.Alcohol consumption, obesity, diabetes, stress, high cholesterol
  • The mother of a 3-month-old states that her daughter has not been gaining weight. With further questioning, the nurse finds that the infant falls asleep after nursing and wakes up after a short amount of time, hungry again. What other information would the nurse want to obtain?a.The position that the baby sleeps inb.Sibling history of eating disordersc.Amount of background noise when the baby is eatingd.Presence of dyspnea or diaphoresis when sucking
  • While assessing the carotid arteries of an older adult with cardiovascular disease, the nurse would:a.palpate the artery in the upper one third of the neck.b.listen with the bell of the stethoscope to auscultate for bruits.c.palpate both arteries simultaneously to compare amplitude.d.instruct the patient to take slow deep breaths during auscultation.
  • During the assessment of a 68-year-old man with a recent onset of right-sided weakness, the nurse hears a blowing, swishing sound with the bell of the stethoscope over the left carotid artery. This finding would indicate:a.a valvular disorder.b.blood flow turbulence.c.fluid volume overload.d.ventricular hypertrophy.
  • During inspection of the precordium of an adult patient, the nurse notices the chest moving in a forceful manner along the fourth-to-fifth left intercostal space at the midclavicular line. This finding most likely suggests:a.a normal heart.b.a systolic murmur.c.enlargement of the left ventricle.d.enlargement of the right ventricle.
  • During assessment of a healthy adult, where would the nurse palpate for the apical impulse?a.Third left intercostal space at the midclavicular lineb.Fourth left intercostal space at the sternal borderc.Fourth left intercostal space at the anterior axillary lined.Fifth left intercostal space at the midclavicular line
  • In percussing the left cardiac border, the nurse would expect to hear dullness at the:a.third left intercostal space midclavicular line and fifth left intercostal space left sternal border.b.fourth left intercostal space medial to midclavicular line and second left intercostal space midclavicular line.c.fifth left intercostal space midclavicular line and second left intercostal space sternal border.d.fifth left intercostal space sternal border and second right intercostal space midclavicular line.
  • While auscultating for heart sounds, which sequence would the nurse use?a.Aortic area → pulmonic area → Erb’s point → tricuspid area → mitral areab.Pulmonic area → aortic area → Erb’s point → tricuspid area → mitral areac.Aortic area → tricuspid area → Erb’s point → mitral area → pulmonic aread.Pulmonic area → Erb’s point → tricuspid area → pulmonic area → mitral area
  • While counting the apical pulse in a 16-year-old patient, the nurse notes an irregular rhythm. His rate speeds up on inspiration and slows on expiration. What would the nurse do next?a.Talk to the patient about his intake of caffeine.b.Do an electrocardiogram after the examination.c.No further action is needed because this is normal.d.Refer the patient to a cardiologist for further testing.
  • Which of the following statements about S1 during cardiac auscultation is true?a.S1 is louder than S2 at the base.b.S1 indicates the beginning of diastole.c.S1 coincides with the carotid artery pulse.d.S1 is caused by closure of the semilunar valves.
  • During cardiac auscultation, the nurse hears a sound occurring immediately after S2 at the second left intercostal space. What would the nurse do to further assess this sound?a.Have the patient turn to the left side and listen with the bell.b.Ask the patient to hold his breath and listen again.c.No further assessment is needed because it is definitely an S3.d.Watch the patient’s respirations while listening for effect on the sound.
  • Which of the following would the nurse expect to find during cardiac assessment of a 4-year-old child?a.S3 when sitting upb.Persistent tachycardia above 150c.Murmur at second left intercostal space when supined.Palpable apical impulse in fifth left intercostal space lateral to the midclavicular line
  • While auscultating for heart sounds in a 7-year-old during a “routine physical,” the nurse hears the following: an S3, a soft murmur at left midsternal border, and a venous hum when the child is standing. Which of the following about the findings is true?a.S3 is indicative of heart disease in children.b.These can all be normal findings in a child.c.These are indicative of congenital problems.d.The venous hum most likely indicates an aneurysm.
  • During precordial assessment of an 8-month pregnant patient, the nurse palpates the apical impulse at the fourth left intercostal space lateral to the midclavicular line. This would indicate:a.right ventricular hypertrophy.b.increased volume and size of the heart as a result of pregnancy.c.displacement of the heart from elevation of the diaphragm.d.increased blood flow through the internal mammary artery.
  • In auscultating for an S4 with a stethoscope, the nurse would listen with the:a.bell at the base, with the patient leaning forward.b.bell at the apex, with the patient in the left lateral position.c.diaphragm in the aortic area, with the patient sitting.d.diaphragm in the pulmonic area, with the patient in the supine position.
  • The vital signs of a 70-year-old patient with a history of hypertension are BP 180/100 and HR 90. The nurse hears an extra heart sound at the apex immediately before S1. The sound is heard only with the bell while patient is in the left lateral position. With these findings and the patient’s history, the nurse knows that this extra heart sound is most likely:a.split S1.b.atrial gallop.c.diastolic murmur.d.summation sound.
  • The nurse is performing cardiac assessment on a 65-year-old patient 3 days after the patient had a myocardial infarction. Heart sounds are normal when she is supine, but while the patient is sitting and leaning forward, the nurse hears a high-pitched, scratchy sound at the apex with the diaphragm of the stethoscope. The sound disappears on inspiration. The nurse suspects:a.increased cardiac output.b.another myocardial infarction.c.inflammation of the precordium.d.ventricular hypertrophy resulting from muscle damage.
  • The mother of a 10-month-old tells the nurse that she has noticed that her son turns blue whenever he is crying and this is happening more and more frequently. He is also not crawling yet. During examination, the nurse identifies a thrill at the left lower sternal border as well as a loud systolic murmur. What would be the most likely cause of these findings?a.Tetralogy of Fallotb.Atrial septal defectc.Patent ductus arteriosusd.Ventricular septal defect
  • A 30-year-old woman with a history of mitral valve problems states that lately she has been “very tired.” She has started waking up at night and feels like her “heart is pounding.” During assessment, the nurse identifies a thrill and lift at the fifth left intercostal space midclavicular line. In the same area, the nurse also identifies a blowing, swishing sound right after S1. These findings would be most consistent with:a.heart failure.b.aortic stenosis.c.pulmonary edema.d.mitral regurgitation.
  • During cardiac assessment on an adult patient in the hospital for “chest pain,” the nurse finds the following: jugular vein pulsations 4 cm above sternal angle when he is elevated at 45 degrees, BP 98/60, HR 130; ankle edema; difficulty breathing when supine; and S3. Which of the following best explains the cause of these findings?a.Fluid overloadb.Atrial septal defectc.Myocardial infarctiond.Heart failure
  • Normal splitting of the second heart sound is associated with:a.expiration.b.inspiration.c.exercise state.d.low resting heart rate.
  • During cardiovascular assessment, the nurse is aware that a “thrill” is:a.a vibration that is palpable.b.palpated in the right epigastric area.c.associated with ventricular hypertrophy.d.a murmur auscultated at the third intercostal space.
  • During cardiovascular assessment, the nurse is aware that S4 is:a.heard at the onset of atrial diastole.b.usually a normal finding in the older adult.c.heard at the end of ventricular diastole.d.heard best over the second left intercostal space with the individual sitting upright.
  • Which of the following statements about the apical impulse is true?a.It is palpable in all adults.b.It occurs with the onset of diastole.c.Its location may be indicative of heart size.d.It should normally be palpable in the anterior axillary line.
  • A heart sound heard during the interval between the second heart sound (S2) and the next first sound (S1) is a(n):a.systolic sound.b.diastolic sound.c.atrial filling sound.d.ventricular contraction sound.
  • During assessment of a patient, the nurse notes that the apical impulse is displaced laterally and is palpable over a wide area. This indicates:a.systemic hypertension.b.pulmonic hypertension.c.pressure overload, as in aortic stenosis.d.volume overload, as in mitral regurgitation.
  • When the nurse is auscultating the carotid artery for bruits, which of the following reflects correct technique?a.While listening with the bell of the stethoscope, have the patient take a deep breath and hold it.b.While auscultating one side with the bell of the stethoscope, palpate the carotid artery on the other side to check pulsations.c.Lightly apply the bell of the stethoscope over the carotid artery; have the patient take a breath, exhale, and hold briefly after exhaling while you listen to it.d.Firmly place the bell of the stethoscope over the carotid artery, and have the patient take a breath, exhale, and hold briefly after exhaling while you listen to it.
  • Which racial group in Canada has the highest prevalence of heart disease and stroke?a.People of African descentb.People of European descentc.Aboriginal peopled.People of Asian descent
  • The nurse is assessing a patient suspected to have cardiomyopathy and is evaluating the hepatojugular reflux. If heart failure is present, what would the nurse find while pushing on the right upper quadrant of the patient’s abdomen, just below the rib cage?a.The jugular veins will rise for a few seconds and then recede back to the previous level if the heart is working properly.b.The jugular veins will remain elevated as long as pressure on the abdomen is maintained.c.An impulse will be visible at the fourth or fifth intercostal space, at or inside the midclavicular line.d.The jugular veins will not be detected during this manoeuvre.
  • The nurse is assessing the apical pulse of a 3-month-old infant and finds the rate to be 135 beats per minute. The nurse interprets this result as:a.normal for this age.b.lower than expected.c.higher than expected, probably caused by crying.d.higher than expected, reflecting persistent tachycardia.
  • Which of the following about the vertebra prominens is true? The vertebra prominens is:a.the spinous process of C7.b.usually not palpable in most individuals.c.opposite the interior border of the scapula.d.located next to the manubrium of the sternum.
  • When performing respiratory assessment on a patient, the nurse notes a costal angle of approximately 90 degrees. This characteristic is:a.seen in patients with kyphosis.b.indicative of pectus excavatum.c.a normal finding in a healthy adult.d.an expected finding in a patient with a “barrel chest.”
  • When assessing a patient’s lungs, the nurse is aware that the left lung:a.consists of two lobes.b.is divided by a horizontal fissure.c.consists primarily of an upper lobe on the posterior chest.d.is shorter than the right lung because of the underlying stomach.
  • Which statement about the apices of the lungs is true? The apices of the lungs:a.are at the level of the second rib anteriorly.b.extend 3 to 4 cm above the inner third of the clavicles.c.are located at the sixth rib anteriorly and the eighth rib laterally.d.rest on the diaphragm at the fifth intercostal space in the midclavicular line.
  • During examination of the anterior thorax, the nurse is aware that the trachea bifurcates anteriorly at the:a.costal angle.b.sternal angle.c.xiphoid process.d.suprasternal notch.
  • The nurse knows that expected assessment findings in the normal adult lung include:a.adventitious sounds and limited chest expansion.b.increased tactile fremitus and dull percussion tones.c.muffled voice sounds and symmetrical tactile fremitus.d.absent voice sounds and hyperresonant percussion tones.
  • The primary muscles of respiration include the:a.diaphragm and intercostals.b.sternomastoids and scaleni.c.trapezius and rectus abdominis.d.external obliques and pectoralis major.
  • A 65-year-old patient with a history of heart failure comes to the clinic with complaints of “being awakened from sleep with shortness of breath.” Which of the following actions by the nurse is most appropriate?a.Obtain a detailed history of the patient’s allergies and asthma.b.Tell the patient to sleep on his or her right side to facilitate ease of respiration.c.Assess for other signs and symptoms of paroxysmal nocturnal dyspnea.d.Reassure the patient that this is normal and will probably resolve within the next week.
  • When assessing for tactile fremitus, over which location is it normal to feel tactile fremitus most intensely?a.Between the scapulaeb.Third intercostal space, midclavicular line (MCL)c.Fifth intercostal space, midaxillary line (MAL)d.Over the lower lobes, posterior side
  • The nurse is aware that tactile fremitus is produced by:a.moisture in the alveoli.b.air in the subcutaneous tissues.c.sounds generated from the larynx.d.blood flow through the pulmonary arteries.
  • During percussion, the nurse knows that a dull percussion note elicited over a lung lobe most likely results from:a.shallow breathing.b.normal lung tissue.c.decreased adipose tissue.d.increased density of lung tissue.
  • The most important technique when progressing from one auscultatory site on the thorax to another is:a.side-to-side comparison.b.top-to-bottom comparison.c.posterior-to-anterior comparison.d.interspace-by-interspace comparison.
  • When auscultating the lungs of an adult patient, the nurse notes that over the posterior lower lobes, low-pitched, soft breath sounds are heard, with inspiration being longer than expiration. The nurse is aware that these are:a.sounds normally auscultated over the trachea.b.bronchial breath sounds and are normal in that location.c.vesicular breath sounds and are normal in that location.d.bronchovesicular breath sounds and are normal in that location.
  • When auscultating the chest in an adult, the nurse would:a.instruct the patient to take deep, rapid breaths.b.instruct the patient to breathe in and out through the nose.c.hold the diaphragm of the stethoscope firmly against the chest.d.hold the bell of the stethoscope lightly against the chest to avoid friction.
  • Percussion over an area of atelectasis in the lungs reveals:a.dullness.b.tympany.c.resonance.d.hyperresonance.
  • During auscultation of the lungs, the nurse is aware that decreased breath sounds would most likely be heard:a.when the bronchial tree is obstructed.b.when adventitious sounds are present.c.in conjunction with whispered pectoriloquy.d.in conditions of consolidation, such as pneumonia.
  • The nurse notes hyperresonant tones when percussing the thorax of an infant. The nurse’s best action would be to:a.notify the physician.b.suspect a pneumothorax.c.consider this a normal finding.d.monitor the infant’s respiratory rate and rhythm.
  • A normal finding when assessing the respiratory system of an older adult is:a.increased thoracic expansion.b.decreased mobility of the thorax.c.a decreased anteroposterior diameter.d.bronchovesicular breath sounds throughout the lungs.
  • A mother brings her 3-month-old infant with a cold to the clinic. The mother tells the nurse that the baby has had “a runny nose for a week.” When performing the physical assessment, the nurse notes that the child has nasal flaring as well as sternal and intercostal retractions. The nurse’s next action should be to:a.reassure the mother that these are normal symptoms of a cold.b.recognize that these are serious signs and contact the physician.c.recognize that these are symptoms of rachitic rosary and refer the infant to be seen by a physician within the week.d.perform a complete cardiac assessment because these are probably signs of early heart failure.
  • When assessing the respiratory system of a 4-year-old child, which of the following findings would the nurse expect?a.Crepitus palpated at the costochondral junctionsb.No diaphragmatic excursion as a result of decreased inspiratory volumec.The presence of bronchovesicular breath sounds in the peripheral lung fieldsd.An irregular respiratory pattern and a respiratory rate of 40 breaths per minute at rest
  • When inspecting the anterior chest of an adult, the nurse should assess for:a.diaphragmatic excursion.b.symmetric chest expansion.c.the presence of breath sounds.d.the shape and configuration of the chest wall.
  • In which of the following would fine crackles most likely be noted during auscultation?a.In a healthy 5-year-old childb.In the pregnant patientc.In the immediate newborn periodd.In association with a pneumothorax
  • The nurse has noted unequal chest expansion and is aware that this occurs when:a.the patient is obese.b.part of the lung is obstructed or collapsed.c.bulging of the intercostal spaces is present.d.accessory muscles are used to augment respiratory effort.
  • Bronchophony heard upon auscultation is associated with:a.pneumothorax.b.hyperresonance.c.pulmonary consolidation.d.decreased breath sounds.
  • Bronchovesicular breath sounds are:a.musical in quality.b.usually pathological.c.expected near the major airways.d.similar to bronchial sounds, except that they are shorter in duration.
  • Which of the following adventitious sounds would be produced by air passing through narrowed bronchioles?a.Wheezesb.Bronchial soundsc.Bronchophonyd.Whispered pectoriloquy
  • A patient has a long history of chronic obstructive pulmonary disease (COPD). During assessment, the nurse is most likely to observe:a.unequal chest expansion.b.increased tactile fremitus.c.atrophied neck and trapezius muscles.d.an anteroposterior-to-transverse diameter ratio of 1:1.
  • A teenage patient comes to the emergency department with complaining of inability to “breathe and a sharp pain in my left chest.” Assessment findings include cyanosis, tachypnea, tracheal deviation to the right, decreased tactile fremitus on the left, hyperresonance on the left, and decreased breath sounds on the left. This description is consistent with:a.bronchitis.b.a pneumothorax.c.acute pneumonia.d.an asthma attack.
  • An adult patient with a history of allergies comes to the clinic complaining of wheezing and difficulty breathing while working in his yard. Assessment findings include tachypnea, use of accessory neck muscles, prolonged expiration, intercostal retractions, decreased breath sounds, and expiratory wheezes. This description is consistent with:a.asthma.b.atelectasis.c.lobar pneumonia.d.congestive heart failure.
  • Which of the following describes normal changes in the respiratory system of the older adult?a.Severe dyspnea is experienced on exertion as a result of age-related changes in the lungs.b.Respiratory muscle strength increases to compensate for decreased vital capacity.c.There is decrease in Small airway closure is decreased, leading to problems with atelectasis.d.The lungs are less elastic and distensible, decreasing their ability to collapse and recoil.
  • A woman in her 26th week of pregnancy states that she is “not really short of breath” but feels that she is always aware of her breathing and the need to breathe. Which of the following would be the nurse’s best reply?a.“The diaphragm becomes fixed during pregnancy, making it difficult to take in deep breaths.”b.“The increase in estrogen levels during pregnancy often causes a decrease in the diameter of the rib cage and makes it difficult to breathe.”c.“What you are experiencing is normal. Some women may interpret this as shortness of breath, but nothing is wrong.”d.“This is normal as the fetus grows because of the increased oxygen demand on the mother’s body, which results in an increased respiratory rate.”
  • A 35-year-old recent immigrant complains of having a cough, expectoration of rust-coloured sputum, low-grade afternoon fevers, and night sweats for the past 2 months. The nurse’s preliminary analysis, based on this history, is that this patient may be suffering from:a.bronchitis.b.pneumonia.c.tuberculosis.d.pulmonary edema.
  • A 70-year-old patient is being seen in the clinic for severe exacerbation of his heart failure. Which of the following is the nurse most likely to observe in this patient?a.Shortness of breath, orthopnea, paroxysmal nocturnal dyspnea, ankle edemab.Rasping cough, thick mucoid sputum, wheezingc.Productive cough, dyspnea, weight loss, anorexiad.Fever, dry nonproductive cough, bronchial breath sounds
  • A patient complains of a cough that is worse at night. The nurse recognizes that this may indicate:a.pneumonia.b.postnasal drip or sinusitis.c.exposure to irritants at work.d.chronic bronchial irritation due to smoking.
  • During morning assessment, the nurse notes that the patient’s sputum is frothy and pink. Which of the following conditions could this finding indicate?a.Croupb.Tuberculosisc.Viral infectiond.Pulmonary edema
  • While auscultating for breath sounds, the nurse will use the stethoscope correctly by:a.listening to at least one full respiration in each location.b.listening as the patient inhales and then go to the next site during exhalation.c.having the patient breathe in and out rapidly while listening to the breath sounds.d.listening to sounds through clothing or the hospital gown if the patient is shy.
  • A patient has been admitted to the emergency department with a medical diagnosis of possible pulmonary embolism. Which assessment findings would the nurse recognize as related to this condition?a.Absent or decreased breath soundsb.Productive cough with thin, frothy sputumc.Chest pain that is worse on deep inspiration, dyspnead.Diffuse infiltrates with areas of dullness upon percussion
  • During palpation of the anterior chest wall, the nurse notes a coarse, crackling sensation over the skin surface and suspects:a.tactile fremitus.b.crepitus.c.friction rub.d.adventitious sounds.
  • The nurse is auscultating the lungs of a sleeping patient and notes short, popping, crackling sounds that stop after a few breaths. The nurse recognizes that these breath sounds are:a.atalectatic crackles, and not pathological.b.fine crackles, which may be a sign of pneumonia.c.vesicular breath sounds.d.fine wheezes.
  • During breast examination, it is especially important for the nurse to examine the upper outer quadrant of the breast because it is:a.the largest quadrant of the breast.b.the area where most breast tumours occur.c.where most of the suspensory ligaments are attached.d.more prone to injury and calcifications than are other areas in the breast.
  • During assessment of a woman’s axillary lymph system, the nurse will assess the:a.central, axillary, lateral, and sternal nodes.b.pectoral, lateral, anterior, and sternal nodes.c.central, lateral, pectoral, and subscapular nodes.d.lateral, pectoral, axillary, and suprascapular nodes.
  • If a patient reports a recent infection in the breast, the nurse would expect to find:a.nonspecific node enlargement.b.ipsilateral axillary node enlargement.c.contralateral axillary node enlargement.d.inguinal and cervical node enlargement.
  • A 9-year-old girl is in the clinic for a sports physical. Getting over her initial shyness, she asks, “Am I normal? I don’t seem to need a bra yet but I have some friends who do. What if I never get breasts?” The nurse’s best response would be:a.“Don’t worry, you still have plenty of time to develop.”b.“I know just how you feel; I was a late bloomer myself. Just be patient; they will develop.”c.“You will probably get your periods before you notice any significant development of your breasts.”d.“I understand that it is hard when you feel different from your friends. Breasts usually develop between 8 and 10 years of age.”
  • A patient contacts the clinic and tells the nurse that she is worried that her 10-year-old daughter may have breast cancer. She describes a unilateral enlargement of the right breast with associated tenderness. Which of the following would be the nurse’s best response?a.Tell the mother that breast development is usually fairly symmetrical, so the daughter should be examined right away.b.Tell the mother that she should bring her daughter in right away because breast cancer is fairly common in preadolescent girls.c.Tell the mother that a clinical examination of her daughter would help rule out her suspicions, but it is most likely that the condition is due to normal breast development.d.Tell the mother that it is unusual for newly developing breasts to feel tender because at this stage they would not have developed much fibrous tissue.
  • A 14-year-old girl is anxious about not having reached menarche yet. When taking history, which of the following should the nurse ascertain?a.The age she began to develop breastsb.The age her mother developed breastsc.The age she began to develop pubic haird.The age she began to develop axillary hair
  • A woman is in the family planning clinic to obtain birth control information. She states that her breasts “change all through the month” and that she is worried that this may be unusual. Which of the following would be the nurse’s best response?a.Tell her that it is unusual. Breasts of nonpregnant females usually remain unchanged throughout the month.b.Tell her that it is very common for breasts to change in response to stress and assess her life for stressful events.c.Tell her that because of hormonal changes during the monthly menstrual cycle, cyclical breast changes are common.d.Tell her that breast changes normally occur only during pregnancy and that a pregnancy test is needed at this time.
  • A woman has just learned that she is pregnant. What are some things the nurse should teach the woman about breasts?a.She can expect the areolae to become larger and darker in colour.b.Breasts may begin secreting milk after the fourth month of pregnancy.c.Breasts should be inspected for visible veins, which should be reported immediately if present.d.Breast changes are fairly uncommon during pregnancy; most of the changes occur after delivery.
  • The nurse is teaching a pregnant woman about breast milk. Which of the following statements would be true?a.Breast milk is secreted immediately after delivery of the baby.b.Breast milk is rich in protein and sugars (lactose) but has very little fat.c.Colostrum, a thin, watery fluid, is present on days 4 and 5 post partum (after the birth).d.Colostrum is a fluid that contains antibodies and is usually present after the fourth month of pregnancy.
  • A 65-year-old patient remarks that she just cannot believe that her breasts sag so much. She thinks it could be due to lack of exercise. What explanation should the nurse offer her?a.Only women with large breasts experience sagging after menopause.b.Sagging of breasts after menopause is usually due to decreasing muscle mass within the breast.c.After menopause, a high-protein diet will help maintain muscle mass, which would prevent sagging of breasts.d.After menopause, the glandular and fat tissues atrophy, causing breast size and elasticity to decrease, resulting in sagging.
  • While examining a 70-year-old male patient, the nurse notices that he has bilateral gynecomastia. Which of the following describes the nurse’s best course of action?a.Recommend that he make an appointment with his physician for a mammogram.b.Ignore it; it is not unusual for men to have benign breast enlargement.c.Explain that this condition may be the result of hormonal changes and recommend that he consult his physician.d.Tell him that gynecomastia in men is usually associated with prostate enlargement and recommend that he undergo thorough screening.
  • During a breast health interview, a patient states that she occasionally experiences pain in her left breast. The nurse’s most appropriate response to this would be:a.“Don’t worry about the pain; breast cancer is not painful.”b.“I would like some more information about the pain in your left breast.”c.“Oh, I had pain like that after my son was born; it turned out to be a blocked milk duct.”d.“Breast pain is almost always the result of benign breast disease, so let’s just ignore it.”
  • During history taking, a female patient reports that she has noticed a few drops of clear discharge from her right nipple. What should the nurse do next?a.Immediately report this to the physician.b.Discontinue the interview and refer the patient for a mammogram.c.Question the patient about medications she is taking.d.Make a note about the discharge but ask no further questions because clear discharge is often found in healthy women.
  • During physical examination, a 45-year-old woman states that she has had a crusty, itchy rash on her breast for about 2 weeks. In trying to find the cause of the rash, which of the following questions would be important for the nurse to ask?a.Is the rash raised and red?b.Does the rash appear cyclically?c.Where did it first appear—on the nipple, the areola, or the surrounding skin?d.What was she doing when she first noticed the rash, and do her actions make it worse?
  • A patient is newly diagnosed with benign breast disease. Which of the following statements about benign breast disease is true? The presence of benign breast disease:a.makes it more difficult to examine the breasts.b.frequently turns into cancer in a woman’s later years.c.is easily resolved with hormone replacement therapy.d.is usually diagnosed before a woman reaches the childbearing age.
  • During the annual physical examination, a 43-year-old patient states that she does not perform monthly breast self-examinations. She tells the nurse that she believes that mammograms “do a much better job than I ever could to find a lump.” The nurse should explain to her that:a.mammography may not detect all palpable lumps.b.breast self-examination is unnecessary until the age of 50 years.c.she is correct, mammography is a good replacement for breast self-examination.d.she does not need to perform breast self-examination as long as a physician examines her breasts yearly.
  • During assessment, a patient reveals that she is pregnant. She states that she is not sure whether she will breastfeed her baby and asks for some information on breastfeeding. Which of the following statements about breastfeeding is accurate?a.Breastfed babies tend to be more colicky.b.Breastfeeding may reduce the risk of breast cancer.c.Breastfed babies feed more frequently than do infants on formula.d.Breastfeeding is natural to women and every woman can do it.
  • Which of the following women have risk factors that place them at a higher risk for breast cancer?a.A 37-year-old who is slightly overweightb.A 42-year-old who had a diagnosis of ovarian cancerc.A 45-year-old who has never been pregnantd.A 65-year-old whose mother had breast cancer
  • During examination, the nurse notices that a woman’s left breast is slightly larger than her right breast. Which of the following is true?a.Breasts should always be symmetrical.b.This probably occurred due to breastfeeding and is nothing to worry about.c.This finding is not unusual, but the nurse should verify that this change is not new.d.This finding is very unusual and means that the woman may have an inflammation or growth.
  • Which of the following are abnormal findings during the inspection phase of breast examination?a.Nipples in different planes (deviated)b.The left breast slightly larger than the rightc.Skin of a nonpregnant woman marked with linear striaed.Breasts of a pregnant woman having a fine blue network of veins visible under the skin
  • During physical examination, the nurse notes that a female patient has an inverted left nipple. Which of the following statements about inverted nipples is most accurate?a.Normal nipple inversion is usually bilateral.b.Unilateral nipple inversion is always a serious sign.c.It should be determined whether the inversion is a recent change.d.Nipple inversion is not significant unless accompanied by an underlying palpable mass.
  • During breast examination, which of the following is the correct procedure to screen for nipple or skin retraction?a.Have the woman bend and touch her toes with the tips of her fingers.b.Have the woman lie down on her left side, and note any retraction.c.Have the woman change from a supine position to a standing position, and note any lag or retraction.d.Have the woman slowly lift her arms above her head, and note any retraction or lag in movement.
  • During breast palpation, which of the following positions is most likely to clearly delineate significant lumps?a.Supine, with arms raised over her headb.Sitting, with arms relaxed at the sidesc.Supine, with arms relaxed at the sidesd.Sitting, with arms flexed and fingertips touching shoulders
  • Which of the following clinical situations would the nurse consider as being outside normal limits?a.A patient has had one pregnancy. She states that she believes she may be entering menopause. Her breast examination reveals breasts that are soft and sag slightly.b.A patient has never been pregnant. Her breast examination reveals large pendulous breasts that have a firm, transverse ridge along the lower quadrant in both breasts.c.A patient has never been pregnant. She reports that she should begin her period tomorrow. Her breast examination reveals breast tissue that is nodular and somewhat engorged. She states that the examination was slightly painful.d.A patient has had two pregnancies and she breastfed both her children. Her youngest child is now 10 years old. Her breast examination reveals breast tissue that is somewhat soft, and she has a small amount of thick yellow discharge from both nipples.
  • A patient states during the assessment that she noticed a new lump in her left breast near her axilla while in the shower a few days ago. The nurse should plan to:a.palpate the lump first.b.palpate the unaffected breast first.c.avoid palpating the lump because it could be a cyst, which might rupture.d.palpate the breast with the lump first and the axilla last.
  • While palpating, the nurse has found a lump in a female patient’s right breast. The nurse documents this as a small, round, firm, distinct lump located at 2 o’clock, 2 cm from the nipple. It is nontender and fixed. There is no associated retraction of skin or nipple, no erythema, and no axillary lymphadenopathy. Which of the following statements about missing information in the documentation is true?a.It is missing information about the shape of the lump.b.It is missing information about the lump’s consistency.c.It is missing information about the exact size of the lump.d.It is missing information about whether the lump is solitary or multiple.
  • The nurse is conducting a class about breast self-examination (BSE). Which of the following statements indicates proper BSE?a.The best time to perform BSE is in the middle of the menstrual cycle.b.The woman needs to do BSE only bimonthly unless she has fibrocystic breast tissue.c.The best time to perform BSE is 4 to 7 days after the first day of the menstrual period.d.If she suspects that she is pregnant, the woman should not perform BSE until her baby is born.
  • Which of the following statements reflects the best approach to teaching a woman about BSE?a.“BSE is more important for you, since you have never had any children.”b.“BSE is very important because one out of nine women will develop breast cancer.”c.“Monthly BSE will help you feel familiar with your own breasts and their normal variations.”d.“BSE will save your life because there is a likelihood of finding a cancerous lump between mammograms.”
  • A 55-year-old postmenopausal woman is being seen in the clinic for her annual physical examination. She is concerned about the changes she has noticed in her breasts in the past 5 years. She states that her breasts have decreased in size and that the elasticity has decreased so much that her breasts seem “flat and flabby.” The nurse’s best reply would be:a.“This change occurs most often because of long-term use of bras that do not provide enough support to the breast tissues.”b.“This is a normal change that occurs as women get older. It is due to the increased levels of progesterone during the aging process.”c.“Decreases in progesterone and estrogen after menopause causes atrophy of the glandular tissue in the breast. This is a normal process of aging.”d.“Postural changes in the spine make it appear that your breasts have changed in shape. Exercises to strengthen the muscles of the upper back and chest wall will help prevent the decreases in elasticity and size.”
  • A 43-year-old woman is at the clinic for a routine examination. She reports that she has had a breast lump in her right breast for years. Recently, it has begun to change in consistency and is becoming harder. She reports that 5 years ago her physician evaluated the lump and had determined that it “was nothing to worry about.” The examination validates the presence of a mass in the right upper outer quadrant at 1 o’clock, approximately 5 cm from the nipple. It is firm, mobile, nontender, with borders that are not well defined. The nurse’s recommendation to her is:a.“Because of the change in consistency of the lump, it should be further evaluated by a physician.”b.“The changes could be related to your menstrual cycles. Keep track of changes in the mass each month.”c.“This is probably nothing to worry about because it has been present for years and has been determined to be noncancerous by your physician.”d.“Because you are experiencing no pain and the size has not changed, continue to monitor the lump and return to the clinic in 3 months.”
  • While discussing BSE with a 30-year-old woman, which of the following statements by the nurse is most appropriate?a.“The best time to examine your breasts is during ovulation.”b.“Examine your breasts every month on the same day of the month.”c.“Examine your breasts shortly after your menstrual cycle each month.”d.“The best time to examine your breasts is immediately before menstruation.”
  • The nurse is discussing BSE with a postmenopausal woman. The best time for postmenopausal women to perform BSE is:a.the same day every month.b.daily, during the shower or bath.c.1 week after the menstrual period.d.every year during the annual gynecological examination.
  • While examining a patient’s breasts, the nurse finds the left breast slightly larger than the right, as well as the presence of Montgomery’s glands bilaterally and a fine venous pattern in both breasts. The nurse would:a.palpate over the venous patterns, checking for drainage.b.consider these normal findings and proceed with the examination.c.ask extensive history questions regarding the woman’s breast asymmetry.d.continue with examination and then refer the patient for further evaluation of the Montgomery’s glands.
  • During examination, the nurse notes a supernumerary nipple just under the patient’s left breast. The patient tells the nurse that she always thought it was a mole. Which of the following statements about this finding is correct?a.It is a normal variation and not a significant finding.b.It is a significant finding and needs further investigation.c.It also contains glandular tissue and may leak milk during pregnancy and lactation.d.The patient is correct—it is actually a mole that happens to be located under the breast.
  • While examining a 75-year-old woman, the nurse notes that the skin over her right breast is thickened and the hair follicles appear exaggerated. This condition is known as:a.dimpling.b.retraction.c.peau d’orange.d.benign breast disease.
  • A breastfeeding mother is diagnosed with a breast abscess. Which of the following instructions from the nurse is correct? The mother needs to:a.continue to nurse on both sides to encourage milk flow.b.discontinue nursing immediately to allow for healing.c.temporarily discontinue nursing on the affected breast and must manually express the milk and discard it.d.temporarily discontinue nursing on the affected breast but can manually express the milk and give it to the baby.
  • A new mother, who has been breastfeeding for 1 month, calls the clinic to report that an area of her left breast is red, swollen, tender, very hot, and hard. She has a fever of 38.3ºC (101ºF). She has also had symptoms of flu, such as chills, sweating, and tiredness. From this description, what condition does the nurse suspect?a.Mastitisb.Paget’s diseasec.Plugged milk ductd.Mammary duct ectasia
  • During breast examination, the nurse notes that the woman’s nipple is flat, broad, and fixed. The patient states that this “started happening a few months ago.” This finding suggests:a.dimpling.b.a retracted nipple.c.nipple inversion.d.deviation in nipple pointing.
  • A 54-year-old man comes to the clinic with a “horrible problem.” He tells the nurse that he has just discovered a lump in his breast and is fearful of cancer. Which statement about breast cancer in men is true?a.Breast masses in men are difficult to detect because of minimal breast tissue.b.Breast masses in men are less noticeable because of minimal breast tissue.c.Fewer than 1% of all breast cancers occur in men.d.Most breast masses in men are diagnosed as gynecomastia.
  • A 54-year-old man comes to the clinic with a “horrible problem.” He tells the nurse that he has just discovered a lump in his breast and is fearful of cancer. Which statement about breast cancer in men is true?a.Breast masses in men are difficult to detect because of minimal breast tissue.b.Breast masses in men are less noticeable because of minimal breast tissue.c.Fewer than 1% of all breast cancers occur in men.d.Most breast masses in men are diagnosed as gynecomastia.
  • The primary purpose of the ciliated mucous membrane in the nose is to:a.warm the inhaled air.b.filter out dust and bacteria.c.filter coarse particles from inhaled air.d.facilitate movement of air through the nares.
  • The projections in the nasal cavity that increase the surface area are called the:a.meatus.b.septum.c.turbinates.d.Kiesselbach’s plexus.
  • Which of the following about a newborn infant is true?a.The sphenoid sinuses are at full size at birth.b.The maxillary sinuses reach full size after puberty.c.The frontal sinuses are fairly well developed at birth.d.The maxillary and ethmoid sinuses are the only ones present at birth.
  • The tissue that connects the tongue to the floor of the mouth is the:a.uvula.b.palate.c.papillae.d.frenulum.
  • The salivary gland that is located in the cheek in front of the ear is the:a.parotid gland.b.Stenson’s gland.c.sublingual gland.d.submandibular gland.
  • While assessing the tonsils of a 30-year-old, the nurse notes that they look involuted and granular and appear to have deep crypts. What is the correct follow up to these findings?a.Refer the patient to a throat specialist.b.Nothing, this is the appearance of normal tonsils.c.Continue with the assessment to look for any other abnormal findings.d.Obtain a throat specimen to culture for possible strep infection.
  • The nurse is obtaining history for a 3-month-old infant. During the interview, the mother states, “I think she is getting her first tooth because she has started drooling a lot.” The nurse’s best response would be:a.“You’re right, drooling is usually a sign of the eruption of the first tooth.”b.“It would be unusual for a 3-month-old to be getting her first tooth.”c.“This could be the sign of a problem with the salivary glands.”d.“She is just starting to salivate and hasn’t learned to swallow the saliva.”
  • During an assessment of an 80-year-old patient, the nurse would expect to find:a.hypertrophy of the gums.b.increased production of saliva.c.decreased ability to identify odours.d.finer and less prominent nasal hair.
  • The nurse is doing an oral assessment on a 40-year-old patient of African descent and notes the presence of a 1-cm, nontender, greyish-white lesion on the left buccal mucosa. Which of the following about this lesion is true?a.This lesion is leukoedema and is common in Blacks.b.This is the result of hyperpigmentation and is normal.c.This is torus palatinus and would normally only be found in smokers.d.This type of lesion is indicative of cancer and should be investigated immediately.
  • During history taking, a patient tells the nurse that he has frequent nosebleeds and asks about the best way to prevent them. What would be the nurse’s best response?a.“Sit straight, and place a cold compress over your nose.”b.“Sit straight with your head tilted forward, and pinch your nose.”c.“Just let the bleeding stop on its own, but don’t blow your nose.”d.“Lie on your back with your head tilted back, and pinch your nose.”
  • A 92-year-old patient has had a stroke, and the right side of his face is drooping. What else would the nurse suspect?a.Epistaxisb.Agenesisc.Dysphagiad.Xerostomia
  • While obtaining history for a 1-year-old from the mother, the nurse notices that the baby has had a bottle in his mouth the entire time. The mother states that “it makes a great pacifier.” The best response by the nurse would be:a.“You’re right, bottles make very good pacifiers.”b.“Use of a bottle is better for the teeth than thumb sucking.”c.“It’s okay to do this as long as the bottle contains milk and not juice.”d.“Prolonged use of a bottle can increase the risk for tooth decay and ear infections.”
  • A 72-year-old patient has a history of hypertension and chronic lung disease. An important question for the nurse to include in history taking would be:a.“Do you use a fluoride supplement?”b.“Have you had tonsillitis in the last year?”c.“At what age did you get your first tooth?”d.“Have you noticed any dryness in your mouth?”
  • When using an otoscope to assess the nasal cavity, which of the following would the nurse need to do?a.Insert the speculum at least 3 cm into the vestibule.b.Avoid touching the nasal septum with the speculum.c.Gently displace the nose to the side that is being examined.d.Keep the speculum tip medial to avoid touching the floor of the nares.
  • The nurse is performing an assessment on a 21-year-old patient and notes that his nasal mucosa appears pale, grey, and swollen. What would be the most appropriate question to ask the patient?a.“Are you aware of having any allergies?”b.“Do you have an elevated temperature?”c.“Have you had any symptoms of a cold?”d.“Have you been having frequent nosebleeds?”
  • The nurse is palpating the sinus areas. If they are normal, which of the following would the patient report?a.No sensationb.Firm pressurec.Pain during palpationd.Pain sensation behind eyes
  • During the oral assessment of a 30-year-old patient of African descent, the nurse notes bluish lips and a dark line along the gingival margin. What would the nurse do in response to this finding?a.Check the patient’s hemoglobin for anemia.b.Assess for other signs of insufficient oxygen supply.c.Proceed with the assessment, knowing that this is a normal finding.d.Ask the patient if he had been exposed to an excessive amount of carbon monoxide.
  • During the assessment of a 20-year-old patient with a 3-day history of nausea and vomiting, the nurse notes the following: dry mucosa and deep fissures in the tongue. This finding is indicative of:a.dehydration.b.irritation by gastric juices.c.a normal oral condition.d.side effects of nausea medication.
  • A 32-year-old woman is at the clinic for a checkup of “little white bumps in my mouth.” During the assessment, the nurse notes that she has a 0.5 cm white, nontender papule under her tongue and one on the mucosa of her right cheek. Which of the following would the nurse tell the patient?a.“These spots are seen with infections such as strep throat.”b.“These could be indicative of a serious lesion, so I will refer you to a specialist.”c.“This is called leukoplakia and can be caused by chronic irritation due to, for example, smoking.”d.“These bumps are Fordyce’s granules, which are sebaceous cysts and not a serious condition.”
  • Which of the following best describes the test to assess the function of cranial nerve X?a.Observe the patient’s ability to articulate specific words.b.Assess movement of the hard palate and uvula with the gag reflex.c.Have the patient stick out the tongue, and observe for tremors or pulling to one side.d.Ask the patient to say “ahhh,” and watch for movement of the soft palate and uvula.
  • A 10-year-old is at the clinic for “a sore throat lasting 6 days.” Which of the following would be consistent with an acute infection?a.Tonsils 1+/1–4+ and pinkb.Tonsils 2+/1–4+ with small plugs of white debrisc.Tonsils 3+/1–4+ with large white spotsd.Tonsils 3+/1–4+ with yellowish exudate
  • The nurse is unable to suction the nares of a newborn immediately following delivery. The attempt to pass a catheter through both nasal cavities has met with no success. What would be the nurse’s best action in this situation?a.Attempt to suction again with a bulb syringe.b.Wait for a few minutes and try again once the infant stops crying.c.Recognize that this situation requires immediate intervention.d.Request the physician’s assistance as soon as possible.
  • The nurse notices that the mother of a 2-year-old boy brings him to the clinic quite frequently for various injuries and suspects there may be some child abuse involved. The nurse should inspect the young child for:a.swollen, red tonsils.b.ulcerations on the hard palate.c.bruising on the buccal mucosa or gums.d.small yellow papules along the hard palate.
  • The nurse is assessing a 3-year-old who has been brought to the clinic for “drainage from the nose.” On assessment, it is found that there is a purulent, malodorous drainage from the left naris but no drainage from the right naris. The child is afebrile and has no other symptoms. What should the nurse do next?a.Refer the child to a physician for an antibiotic prescription.b.Have the mother bring the child back in 1 week.c.Perform an otoscopic examination of the left naris.d.Tell the mother that this is normal for children of this age.
  • During the assessment of a 26-year-old for “a spot on my lip I think is cancer,” the nurse notes the following: a cluster of clear vesicles with an erythematous base around them located at the lip–skin border. The patient mentions that she just returned from Hawaii. What would be the most appropriate response by the nurse?a.Tell the patient she will need to see a skin specialist.b.Discuss the benefits of doing a biopsy of any unusual lesion.c.Tell the patient this is herpes simplex virus I infection, which will heal in 4 to 10 days.d.Tell the patient this is most likely the result of a riboflavin deficiency and discuss nutrition.
  • While performing a mouth assessment on a patient, the nurse notices a 1-cm ulceration that is crusted and has an elevated border. It is located on the outer third of the lower lip. What other information would be most important for the nurse to obtain?a.Nutritional statusb.When the patient first noticed the lesionc.Whether the patient has had a recent coldd.Whether the patient has had any recent exposure to sick animals
  • A pregnant woman states that she is concerned about her gums because she has noticed they are swollen and have started bleeding. What would be an appropriate response by the nurse?a.“This is probably due to a vitamin C deficiency.”b.“I’m not sure what causes it, but let me know if it doesn’t get better in a few weeks.”c.“You need to make an appointment with your dentist as soon as possible to have this checked.”d.“This can be caused by hormonal changes during pregnancy.”
  • A 40-year-old patient who has just finished chemotherapy for breast cancer tells the nurse that she is concerned about her mouth. During assessment, the nurse finds the following: areas of buccal mucosa that are raw and red with some bleeding, as well as other areas that have a white, cheesy coating. The nurse recognizes that this abnormality is:a.carcinoma.b.candidiasis.c.leukoplakia.d.Koplik’s spots.
  • A hospitalized patient who has received numerous antibiotics is being assessed by the nurse, who notes that his tongue appears black and hairy. Which of the following would the nurse say to the patient?a.“We need to get a biopsy done and find out what the cause is.”b.“This is an overgrowth of hair and will go away in a few days.”c.“This is a fungal infection caused by all the antibiotics you’ve received.”d.“This is probably caused by the same bacteria you had in your lungs.”
  • The nurse is assessing a patient with a history of intravenous drug abuse. While assessing his mouth, the nurse notices a dark red confluent macule on the hard palate. This could be an early sign of:a.AIDS.b.measles.c.leukemia.d.carcinoma.
  • A mother brings her 4-month-old to the clinic with concerns about a small pad-like growth on the mid-upper lip that has been present since the baby was 1 month old. The infant has no health problems. On physical examination, the nurse notes a 0.5-cm, fleshy, elevated area on the mid-upper lip. There is no evidence of inflammation or drainage. What would the nurse tell this mother?a.“This is an area of irritation caused by teething and is nothing to worry about.”b.“This is an abnormal finding and should be evaluated by a specialist.”c.“This is the result of chronic drooling and should resolve within the next month or two.”d.“This is a sucking tubercle caused by friction due to breast- or bottlefeeding and is normal.”
  • A mother is concerned that her 18-month-old already has 12 teeth. She is wondering if this is normal for a child of this age. The nurse’s best response would be:a.“How many teeth did you have at this age?”b.“All 20 deciduous teeth are expected to have erupted by 4 years.”c.“This is a normal number of teeth for an 18-month-old.”d.“Normally, 16 deciduous teeth are expected to have erupted between 2 and 3 years.”
  • When examining an older adult, which of the following findings would the nurse recognize as related to the aging process?a.Teeth that appear shorterb.Tongue that looks smoother in appearancec.Buccal mucosa that is beefy red in appearanced.A small, painless lump on the dorsum of the tongue
  • When examining the nares of a 45-year-old patient who complains of rhinorrhea, itching of the nose and eyes, and sneezing, the nurse notes the following: pale turbinates, swelling of the turbinates, and clear rhinorrhea. Which of the following is most likely the cause?a.Nasal polypsb.Acute sinusitisc.Allergic rhinitisd.Nasal carcinoma
  • When assessing the tongue of an adult, an abnormal finding would be:a.a smooth glossy dorsal surface.b.a thin white coating over the tongue.c.raised papillae on the dorsal surface.d.visible venous patterns on the ventral surface.
  • Which of the following assessment findings would the nurse be most concerned about?a.A painful vesicle inside the cheek for 2 daysb.The presence of moist, nontender Stenson’s ductsc.Stippled gingival margins that adhere snugly to the teethd.An ulceration with rolled edges on the side of the tongue
  • A patient has been diagnosed with “strep throat.” Which complication may occur without treatment?a.Rubellab.Leukoplakiac.Rheumatic feverd.Scarlet fever
  • During a checkup, a 22-year-old woman tells the nurse that she uses an over-the-counter nasal spray for her allergies. She also reports that it does not work as well as it used to. The best response by the nurse would be:a.“You should never use over-the-counter nasal sprays because of the risk of addiction.”b.“You should try switching to another brand of medication to prevent this problem.”c.“It is important to keep using this spray to keep your allergies under control.”d.“Using these nasal medications irritates the lining of the nose and may cause rebound swelling.”
  • During examination of a 4-year-old Aboriginal child, the nurse notices that her uvula is partially split. Which of the following statements about this condition is accurate?a.This is cleft palate, which is commonly found in Aboriginal people.b.This is a bifid uvula, which is a common occurrence in some Aboriginal groups.c.This occurs due to an injury and should be reported to the authorities.d.This is torus palatinus, common among Aboriginal people.
  • A patient comes into the clinic complaining of facial pain, fever, and malaise. On examination, the nurse notes swollen turbinates and a purulent discharge from the nose. The patient also complains of a dull, throbbing pain in his cheeks and teeth on the right side and pain when the nurse palpates the areas. The nurse recognizes that this patient has:a.posterior epistaxis.b.frontal sinusitis.c.maxillary sinusitis.d.nasal polyps.
  • A woman in her second trimester of pregnancy mentions that since becoming pregnant she has had “more nosebleeds than ever.” The nurse recognizes that this is due to:a.coagulation problems in the patient.b.increased vascularity in the upper respiratory tract as a result of the pregnancy.c.increased susceptibility to colds and nasal irritation.d.inappropriate use of nasal sprays.
  • The portion of the ear that consists of movable cartilage and skin is called the:a.auricle.b.concha.c.outer meatus.d.mastoid process.
  • The nurse is examining a patient’s ears and notices cerumen in the external canal. Which of the following statements about cerumen is correct?a.Sticky honey-coloured cerumen is a sign of infection.b.The presence of cerumen is indicative of poor hygiene.c.The purpose of cerumen is to protect and lubricate the ear.d.Cerumen is necessary for transmitting sound through the auditory canal.
  • When examining the ear with an otoscope, the nurse would expect to find that the tympanic membrane is:a.light pink with a slight bulge.b.pearly grey and slightly concave.c.pulled in at the base of the cone of light.d.whitish, with a small fleck of light in the superior portion.
  • Which of the following statements about the Eustachian tube is true?a.It is responsible for the production of cerumen.b.It remains open except during swallowing or yawning.c.It allows passage of air between the middle and outer ear.d.It helps equalize air pressure on both sides of the tympanic membrane.
  • A patient with a middle ear infection asks the nurse, “What does the middle ear do?” The nurse says that the function of the middle ear is to:a.maintain balance.b.interpret sounds as they enter the ear.c.conduct vibrations of sounds to the inner ear.d.increase amplitude of sound to enable the inner ear to function.
  • Which of the following cranial nerves is responsible for conducting nerve impulses to the brain from the organ of Corti?a.CN Ib.CN IIIc.CN VIIId.CN XI
  • Which of the following statements about air conduction is true?a.It is the most efficient pathway for hearing.b.It is caused by the vibrations of bones in the skull.c.The amplitude of sound determines the pitch that is heard.d.A loss of air conduction is called a conductive hearing loss.
  • Tests have shown that a patient has sensorineural hearing loss. During the assessment, it would be important for the nurse to:a.speak loudly so he can hear the questions.b.assess for middle ear infection as a possible cause.c.ask the patient what medications he is currently taking.d.look for the source of the obstruction in the external ear.
  • During an interview, the patient says that he experiences a sensation as if “everything around him is spinning.” The nurse recognizes that the portion of the ear responsible for this sensation is:a.the cochlea.b.cranial nerve VIII.c.the organ of Corti.d.the bony labyrinth.
  • A patient in her first trimester of pregnancy is diagnosed with rubella. Which of the following indicates the significance of this in relation to the infant’s hearing?a.Rubella may affect the mother’s hearing but not the child’s.b.Rubella can damage the organ of Corti in the fetus, which will result in impaired hearing.c.Rubella is dangerous to the fetus only when it occurs in the second trimester of pregnancy.d.Rubella can impair the development of CN VIII and thus affect hearing.
  • The mother of a 2-year-old is concerned because her son has had three ear infections in the past year. Which of the following would be an appropriate response by the nurse?a.“It is unusual for a small child to have frequent ear infections unless there is something else wrong.”b.“We need to check the immune system of your son to see why he is having so many ear infections.”c.“Ear infections are not uncommon in infants and toddlers because they tend to have more cerumen in the external ear.”d.“Your child’s Eustachian tube is shorter and wider than that in an adult, which allows infections to develop more easily.”
  • A 31-year-old patient tells the nurse that he is experiencing a progressive loss of hearing. He says that it does seem to help when people speak more loudly or if he turns up the volume. The most likely cause of his hearing loss is:a.otosclerosis.b.presbycusis.c.trauma to the bones.d.frequent ear infections.
  • A 70-year-old patient reports to the nurse that he is having trouble hearing, especially when he is in large groups. He says he “can’t always tell where the sound is coming from” and the words often sound “mixed up.” What might the nurse suspect as the cause for this?a.Atrophy of the apocrine glandsb.Cilia becoming coarse and stiffc.Degeneration of nerves in the inner eard.Scarring of the tympanic membrane
  • During an assessment of a 20-year-old patient of Asian descent, the nurse notices that he has dry, flaky cerumen in his ear canal. What is the significance of this finding?a.This is probably the result of eczema lesions in his ear.b.This indicates poor hygiene.c.This is a normal finding, and no further follow-up is necessary.d.This could be indicative of change in cilia; the nurse should assess for conductive hearing loss.
  • The nurse is taking history from a patient who may have a perforated eardrum. What would be an important question in this situation?a.“Do you ever notice ringing or crackling in your ears?”b.“When was the last time you had your hearing checked?”c.“Have you ever been told you have any type of hearing loss?”d.“Was there any relationship between the ear pain and the discharge you mentioned?”
  • A 31-year-old patient tells the nurse that he has noticed pain in his left ear when people speak loudly to him. The nurse knows that this finding:a.is normal for people of that age.b.is a characteristic of recruitment.c.may indicate a middle ear infection.d.indicates that the patient has a cerumen impaction.
  • While discussing the history of a 6-month-old infant, the mother tells the nurse that she took a great deal of aspirin while she was pregnant. What question would the nurse want to include in the history?a.“Does your baby startle at loud noises?”b.“Has the baby had any surgeries on her ears?”c.“Have you noticed any drainage from her ears?”d.“How many ear infections has your baby had since birth?”
  • The nurse is performing an otoscopic examination on an adult. Which of the following is true?a.Tilt the person’s head forward during the examination.b.Once the speculum is in the ear, release the traction.c.Pull the pinna up and back before inserting the speculum.d.Use the smallest speculum to decrease the amount of discomfort.
  • The nurse is assessing a 16-year-old patient with head injuries from a recent motor vehicle accident. Which of the following statements indicates the most important reason for assessing for any drainage from the canal?a.If the drum has ruptured, there will be purulent drainage.b.Bloody or clear watery drainage can indicate a basal skull fracture.c.The auditory canal many be occluded from increased cerumen.d.There may be occlusion of the canal caused by foreign bodies from the accident.
  • A colleague is assessing an 80-year-old patient who has ear pain and asks him to hold his nose and swallow. Which of the following about this technique is true?a.This should not be done in an 80-year-old patient.b.This technique is helpful in assessing for otitis media.c.This is especially useful in assessing a patient with an upper respiratory infection.d.This will cause the eardrum to bulge slightly and make landmarks more visible.
  • While performing a voice test to assess hearing in a patient, which of the following would the nurse do?a.Shield the lips while speaking so that the sound is muffled to the patient.b.Whisper two-syllable words and ask the patient to repeat them.c.Ask the patient to place his finger in his ear to occlude outside noise.d.Stand about 120 cm away to ensure that the patient can really hear at this distance.
  • While performing an examination of a 3-year-old with a suspected ear infection, the nurse would:a.omit the otoscopic exam if the child has a fever.b.pull the ear up and back before inserting the speculum.c.ask the mother to leave the room while examining the child.d.perform the otoscopic examination at the end of the assessment.
  • Which of the following statements about otoscopic examination of a newborn would be true?a.Immobility of the drum is a normal finding.b.An “injected” membrane would indicate infection.c.The normal membrane may appear thick and opaque.d.The appearance of the membrane is identical to that of an adult.
  • The nurse is assessing the hearing of a 7-month-old. What would be the expected response to clapping of hands?a.Turns the head to localize the soundb.No obvious response to noisec.A startle and acoustic blink reflexd.Stops all movement and appears to listen.
  • During the ear examination of an 80-year-old patient, which of the following would be a normal finding?a.Loss of high-tone frequency hearingb.Increased elasticity of the pinnac.A thin, translucent membraned.A shiny, pink tympanic membrane
  • An assessment of a 23-year-old patient reveals the following: an auricle that is tender and reddish-blue in colour and has small vesicles. What additional information would the nurse need to know?a.Any change in the ability to hearb.Any recent drainage from the earc.Recent history of trauma to the eard.Any prolonged exposure to extreme cold
  • While performing the otoscopic examination of a 3-year-old boy who has been pulling on his left ear, the nurse finds that his left tympanic membrane is bright red and the light reflex is not visible. The most likely cause is:a.fungal infection.b.acute otitis media.c.rupture of the drum.d.blood behind the drum.
  • The mother of a 2-year-old is concerned about tympanostomy tubes that are going to be inserted in her son’s ears. Which of the following would the nurse include in the teaching plan?a.The tubes are placed in the inner ear.b.The tubes are used in children with sensorineural loss.c.The tubes are permanently inserted during a surgical procedure.d.The purpose of the tubes is to decrease the pressure and allow for drainage.
  • During hearing assessment using the Weber test, the nurse finds that sound lateralizes to the patient’s left ear. What can the nurse conclude from this?a.The patient has a conductive hearing loss in the right ear.b.Lateralization is a normal finding from the Weber test.c.The patient could have either a sensorineural loss or a conductive loss.d.A mistake has occurred; the test must be repeated.
  • A patient states that she is unable to hear well with her left ear. The Weber test shows lateralization to the right ear. The patient has AC>BC with ratio of 2:1 in both ears, left-AC 10 sec and BC 5 sec, right-AC 30 sec, and BC 15 sec. What would be the interpretation of these results?a.The patient may have sensorineural loss.b.The test results are reflective of normal hearing.c.Conduction of sound through bones is impaired.d.These results make no sense, so further tests should be done.
  • Which of the following signs would the nurse expect to find on assessment of an individual with otitis externa?a.Rhinorrheab.Periorbital edemac.Pain over the maxillary sinusesd.Enlarged superficial cervical nodes
  • When performing an otoscopic examination on a 5-year-old child with a history of chronic ear infections, the nurse sees that his right tympanic membrane is amber-yellow in colour and there are air bubbles behind the tympanic membrane. The child reports occasional hearing loss and a popping sound with swallowing. The preliminary analysis based on this information would be that:a.this is most likely serous otitis media.b.the child has acute purulent otitis media.c.there is evidence of a resolving cholesteatoma.d.the child is experiencing the early stages of perforation.
  • A 65-year-old male has reported a crusty nodule behind the pinna. Over the past 6 months, the nodule has been bleeding intermittently and has not healed. On physical assessment, the nurse finds an ulcerated crusted nodule with an indurated base. The preliminary analysis in this situation would be that this:a.is most likely a benign sebaceous cyst.b.is most likely a Darwin’s tubercle and is not significant.c.could be potential carcinoma and warrants referral.d.is a tophus, which is common in older adults, and is a sign of gout.
  • The nurse suspects that a patient has otitis media. Which of the following findings of the tympanic membrane are early signs of otitis media?a.Red and bulgingb.Hypomobilityc.Retraction with landmarks clearly visibled.Flat, slightly pulled in at the centre, and moves with insufflation
  • The nurse is performing middle ear assessment on a 15-year-old patient who has a history of chronic ear infections. When examining the right tympanic membrane, the nurse sees dense white patches. The tympanic membrane is otherwise unremarkable. It is pearly, with the light reflex at 5 o’clock and visible landmarks. The nurse should:a.refer the patient for investigating the possibility of a fungal infection.b.know that these are scars caused from frequent ear infections.c.consider that these findings may be an indication of blood in the middle ear.d.be concerned about the patient’s ability to hear because of this abnormality of the tympanic membrane.
  • The nurse is preparing to do an otoscopic examination on a 2-year-old child. Which of the following reflects correct procedure?a.Pull the pinna down.b.Pull the pinna up and back.c.Tilt the child’s head slightly toward the examiner.d.Have the child touch his chin to his chest.
  • Which of the following is a risk factor for ear infections in young children?a.Family historyb.Air conditioningc.Excessive cerumend.Secondhand cigarette smoke
  • During an otoscopic examination, the nurse notes an area of black and white dots on the tympanic membrane and ear canal wall. What does this finding suggest?a.Malignancyb.Viral infectionc.Blood in the middle eard.Yeast or fungal infection
  • A 17-year-old student is a swimmer on her high school’s swim team. She has had three bouts of otitis externa so far this season and wants to know how to prevent it. The nurse instructs her to:a.use a cotton-tipped swab to dry the ear canals thoroughly after each swim.b.use rubbing alcohol or 2% acetic acid eardrops after every swim.c.irrigate the ears with warm water and suction with a bulb syringe after each swim.d.rinse the ears with a warmed solution of mineral oil and hydrogen peroxide.
  • During ear examination, the patient reports that he is hears a buzzing sound that is “driving me crazy!” The nurse recognizes that this symptom is:a.vertigo.b.pruritus.c.tinnitus.d.cholesteatoma.

Physical examination is the process of evaluating objective anatomic findings through the use of observation, palpation, percussion, and auscultation. The information obtained must be thoughtfully integrated with the patient's history and pathophysiology. Moreover, it is a unique situation in which both patient and physician understand that the interaction is intended to be diagnostic and therapeutic. The physical examination, thoughtfully performed, should yield 20% of the data necessary for patient diagnosis and management.

Almost without exception, some medical history about the patient is available at the time of the physical examination. Rarely, there may be no history, or at best brief recordings of acute events. Information pertinent to the physical examination can be learned from observation of speech, gestures, habits, gait, and manipulation of features and extremities. Interactions with relatives and staff are often revealing. Pigmentary changes such as cyanosis, jaundice, and pallor may be noted. Diaphoresis, blanching, and flushing may provide clues about vasomotor tone related to mood or physiologic abnormalities. Aspects of patient habits, interests, and relationships can be ascertained from pictures, books, magazines, and personal objects at the bedside.

Aside from the hospital room and office, physical examination may occur in a variety of other settings where it is difficult to establish privacy and quiet. The best resource available to the physician to set the stage for the physical examination is to communicate respect and a genuine interest in the patient's welfare. The patient should be addressed politely and asked to perform the required maneuvers of the examination, a technique far preferable to imperative language such as, "I want you to. …" Patients should be prepared for unpleasant portions of the examination.

Aside from explanations and reassurance, it is not necessary to maintain a continuous conversation with the patient during the examination. Avoid embarrassing the patient. Be certain that draping material is used appropriately and that personal areas are not subjected to undue exposure. An examination that ends abruptly may diminish the value of the doctor–patient relationship and may destroy its therapeutic content. The patient may benefit from a brief summary of relevant findings and may require reassurance about what has and has not been found.

The single most useful device for optimal performance of the physical examination is an inquisitive and sensitive mind. Next most useful is mastery of the techniques of observation, palpation, percussion, and auscultation. Less important are the tools required for the examination (Table 4.1).

As the environment affects the quality of the physical examination, it is wise to arrange for quiet and privacy, darkening the room for parts of the examination, and comfort for the patient and examiner.

The complete examination should proceed in an orderly fashion with a minimum of required position shifts by the patient (Table 4.2). On the other hand, the physician must be able to ascertain the integrity of the various organ systems from regional examinations. For instance, from examination of the head and neck, the physician must identify the vascular, neurologic, lymphatic, skeletal, and integumentary components and must relate them to their complements in other body regions. It would be tedious, by contrast, to examine the vascular system in its entirety, followed by a complete neurologic examination and the other organ systems each in turn. When examining an anatomic region, the observer must be alert to the appearance of any abnormality and question at the time the morphologic aspects of the abnormality and its clinical significance.

The general physical examination can take many forms depending upon circumstances. Most often, the examiner evaluates body regions in a general way, looking for abnormalities. Clues derived from the history signal the need for a more precise and detailed examination of a given system. A thorough physical examination often includes the sequence presented in Table 4.3.

The clinically significant physical examination is a flexible entity that should vary with the needs of the patient. Periodic examinations for health assessment need to be comprehensive, as do most hospital admission examinations. In contrast, it will not be cost effective to undertake a complete physical examination in most patients presenting with symptoms of an upper respiratory tract infection or a urinary tract infection.

The physical examination is a key part of a continuum that extends from the history of the present illness to the therapeutic outcome. If the history and physical examination are linked properly by the physician's reasoning capabilities, laboratory tests should in large measure be confirmatory. The physical examination, however, can be the weak link in this chain if it is performed in a perfunctory and superficial manner. Understanding the pathophysiologic mechanism of a physical abnormality is essential for correct diagnosis and management. For instance, the failure to discriminate between and know the origin of carotid bruits and transmitted sounds of valvular origin can have critical significance.

As knowledge of disease changes, the techniques of physical examination become augmented. The astute physician constantly reviews and adds to the repertoire of techniques for physical examination.

Evaluation of the physical examination in terms of sensitivity and specificity is difficult. Interpretation of isolated physical findings is often influenced by the presence or absence of historical information and coexisting physical findings. For instance, the assessment of whether clubbing of the fingers is present or absent has significant interobserver variability and has been demonstrated to be influenced by the clinical appearance of the patient.

A number of studies have attempted to look at the validity of the physical exam as a diagnostic tool. The concept of interobserver and intraobserver variability has been introduced when looking at specific isolated findings. For example, judging the presence or absence of râles is more likely to be agreed upon by several observers and on repeated exams by a single blinded observer, than is the graded intensity of breath sounds. The presence or absence of ascites in patients with known liver disease has been shown to be difficult to determine when using physical exam techniques alone. The bedside measurement of forced expiratory time by auscultation however, has been shown to have a small interobserver variability in trained observers and to have clinical value in following the degree of airway obstruction.

Because of the large degree of variability in observing many physical signs, the following recommendations can be made when reporting and interpreting physical findings.

  1. Emphasis should be placed on dichotomous variables (i.e., presence or absence of râles) rather than on graded variables (i.e., intensity of breath sounds).

  2. Some physical signs (i.e., clubbing of the fingers) represent a continuum from obviously normal to obviously abnormal. Emphasis should be placed on those findings which represent the extremes rather than the "borderline" cases.

  3. Recognition of those physical findings which have a high degree of interobserver variability is important. Good examples of this include detection of moderate or small amounts of ascitic fluid and detection of diaphragmatic movement by percussion. These findings should be deemphasized in favor of those with better reproducibility.

  4. It is beneficial to use the body's "symmetry" to advantage. Differences auscultated in breath sounds between similar area of the right and left lung are far more clinically important than an overall decrease in breath sounds.

If these points are kept in mind, the physical exam will fill its proper role in the care of the patient. That is as an adjunct to a thorough history and as a way for the physician to interact physically with the patient.

Which definition correctly describes a person’s functional ability?a.Functional ability is the measure of the expected changes of aging that a person is experiencing.b.Functional ability refers to an individual’s motivation to live independently.c.Functional ability refers to the level of cognition present in an older person.d.Functional ability refers to a person’s ability to perform activities necessary to live in modern society.

The nurse is preparing to perform a functional assessment of an older patient, and knows that a good approach would be to:a.observe the patient’s ability to perform tasks.b.ask the patient’s wife how well he performs tasks.c.review the medical record for information about the patient’s abilities.d.ask the patient’s physician for information about the patient’s abilities.

The nurse is assessing an older adult’s advanced activities of daily living, which would include:a.recreational activities.b.meal preparation.c.balancing the chequebook.d.self-grooming activities.

The nurse is administering a test that is timed over 15 minutes, and assesses a person’s upper body fine and coarse motor activities, balance, mobility, coordination, and endurance. During this test, activities such as dressing and stair climbing are timed. Which test is described by these activities?a.The Up and Go Testb.The Performance Activities of Daily Livingc.The Physical Performance Testd.Tinetti Gait and Balance Evaluation

A patient will be ready to be discharged from the hospital soon, and the patient’s family members are concerned about whether he is able to go outside alone safely. The nurse will perform which test to assess this ability?a.The Up and Go Testb.The Performance Activities of Daily Livingc.The Physical Performance Testd.Tinetti Gait and Balance Evaluation

The nurse is assessing the forms of support an older patient has before she is discharged. Which of the following illustrates an informal source of support?a.The local senior centreb.Her cleaning ladyc.Her Meals on Wheels meal delivery serviced.Her neighbour, who visits with her daily

An 85-year-old man has been hospitalized after a fall at home, and his 86-year-old wife is at his bedside. She tells the nurse that she is his primary caregiver. The nurse should assess the caregiver for signs of possible caregiver burnout, such as:a.depression.b.weight gain.c.hypertension.d.social phobias.

During a morning assessment, the nurse notices that an older patient is less attentive and is unable to recall yesterday’s events. The nurse administers the Mini Mental State Examination, which will screen for:a.dementia.b.depression.c.delirium.d.psychosis.

During an assessment of a newly admitted 92-year-old woman, the nurse notes that her son does not want to leave the room. The woman has signs of old bruises and healed cuts that happened “last week,” according to the son. Which of the following actions by the nurse is appropriate?a.Ask the son for details about the nature of the patient’s injuries.b.Recognize that older people are often unsteady on their feet, and that falls do occur.c.Notify the authorities of a potential abusive situation.d.Recognize that these findings do not necessarily indicate that abuse has occurred, but are signs that further assessment is needed.

Which of the following statements regarding common environmental hazards is most appropriate for the nurse to make during a functional assessment of an older person’s home environment?a.“These low toilet seats are safe because they are nearer to the ground in case of falls.”b.“Ask a relative or friend to help you to install grab bars in your shower.”c.“These small rugs are ideal for preventing you from slipping on the hard floor.”d.“It would be safer to keep the lighting low in this room to avoid glare in your eyes.”

Which of the following questions would be most appropriate for the nurse to ask when beginning to assess a person’s spirituality?a.“Do you believe in God?”b.“Do you consider yourself to be a spiritual person?”c.“What religious faith do you follow?”d.“Do you believe in the power of prayer?”

The nurse is preparing to assess an older adult, and discovers that he is in severe pain. Which of the following statements about pain and the older adult is true?a.Pain is inevitable with aging.b.Older adults with cognitive impairment feel less pain.c.Alleviating pain should take priority over other aspects of the assessment.d.The assessment should take priority so that care decisions can be made.

Which of the following best describes the action of the hormone progesterone during pregnancy?a.It produces the hormone human chorionic gonadotropin.b.It stimulates duct formation in the breast.c.It promotes sloughing of the endometrial wall.d.It maintains the endometrium around the fetus.

A female patient is experiencing nausea, breast tenderness, fatigue, and amenorrhea. Her last menstrual period was 6 weeks ago. The nurse recognizes that this patient is experiencing:a.positive signs of pregnancy.b.possible signs of pregnancy.c.probable signs of pregnancy.d.presumptive signs of pregnancy.

When performing the examination of a woman who is 8 weeks pregnant, the nurse notes that her cervix is a bluish colour. The nurse would document this finding as:a.Hegar’s sign.b.Homan’s sign.c.Chadwick’s sign.d.Goodell’s sign.

A woman who is 8 weeks pregnant is visiting the clinic for a checkup. Her systolic blood pressure is 30 mm Hg higher than her pre-pregnancy blood pressure. The nurse would:a.consider this a normal finding.b.expect the blood pressure to decrease as the estrogen levels increase throughout the pregnancy.c.consider this an abnormal finding, because blood pressure is typically lower at this point in the pregnancy.d.recommend that she decrease her salt intake in an attempt to decrease her peripheral vascular resistance.

A patient is being seen at the clinic for her 10-week prenatal visit. She asks when she will be able to hear the baby’s heartbeat. The nurse should reply:a.“The baby’s heartbeat is not usually heard until the second trimester.”b.“The baby’s heartbeat may be heard anywhere from the ninth to the twelfth week.”c.“It is often difficult to hear the heartbeat at this point, but we can try.”d.“It is normal to hear the heartbeat at 6 weeks. We may be able to hear it today.”

A patient who is in her first trimester of pregnancy tells the nurse that she is experiencing significant nausea and vomiting, and asks when it will improve. The nurse should reply:a.“Did your mother have significant nausea and vomiting?”b.“Many women experience nausea and vomiting until the third trimester.”c.“Usually, by the beginning of the second trimester, the nausea and vomiting improve.”d.“At about the time you begin to feel the baby move, the nausea and vomiting will subside.”

During the examination of a woman in her second trimester of pregnancy, the nurse notes the presence of a small amount of yellow drainage from the nipples. The nurse knows that this is:a.an indication that the woman’s milk is coming in.b.a sign of possible breast cancer in a pregnant woman.c.most likely colostrum, which is considered a normal finding at this stage of the pregnancy.d.an early stage in the pregnancy for lactation to begin. The woman should be referred to a specialist.

A woman in her second trimester of pregnancy complains of heartburn and indigestion. The nurse should offer which of the following explanations for these problems?a.Tone and motility of the gastrointestinal tract increase during the second trimester.b.Sluggish emptying of the gallbladder, resulting from the effects of progesterone, often causes heartburn.c.Lower blood pressure at this time decreases blood flow to the stomach and gastrointestinal tract.d.The enlarging uterus and altered esophageal sphincter tone predispose the woman to have heartburn.

A patient who is 20 weeks pregnant tells the nurse that she feels more short of breath as her pregnancy progresses. The nurse recognizes that which of the following is true?a.High levels of estrogen cause shortness of breath.b.Feelings of shortness of breath are abnormal during pregnancy.c.The hormones of pregnancy cause an increased respiratory effort.d.The patient should get more exercise in an attempt to increase her respiratory reserve.

The nurse auscultates a functional systolic murmur, grade ii/iv, in a woman in week 30 of her pregnancy. The remainder of her physical assessment is within normal limits. The nurse would:a.consider this an abnormal finding, and refer her for additional consultation.b.ask the woman to run in place, and then assess for an increase in the intensity of the murmur.c.know that this is a normal finding, resulting from the increase in blood volume during pregnancy.d.ask the woman to restrict her activities, and return to the clinic in 1 week for re-evaluation.

A woman who is 28 weeks pregnant has edema in her lower legs bilaterally after working 8 hours a day as a cashier at a local grocery store. What should the nurse tell her?a.“You will be at risk for development of varicose veins when your legs are edematous.”b.“I would like to listen to your heart sounds. Edema can indicate a problem with your heart.”c.“Edema is usually the result of too much salt and fluids in your diet. You may need to try to cut down on salty foods.”d.“As your baby grows, it slows blood return from your legs, causing the swelling. This often occurs with prolonged standing.”

The nurse knows that classic symptoms associated with preeclampsia include:a.proteinuria, headaches, and seizures.b.elevated blood pressure and proteinuria.c.elevated liver enzymes and high platelets.d.neurological signs, elevated blood pressure, and edema.

The nurse knows that the best time to assess a woman’s blood pressure during an initial prenatal visit is:a.at the end of the examination, when she will be the most relaxed.b.at the beginning of the interview, as a nonthreatening method of establishing rapport.c.during the middle of the physical examination, when she is the most comfortable.d.before beginning the pelvic examination, because her blood pressure will be higher after the pelvic examination.

When examining the face of a 28-week pregnant woman, the nurse notes the presence of a butterfly-shaped increase in pigmentation on her face. When documenting, the correct term for this finding is:a.striae.b.chloasma.c.linea nigra.d.the mask of pregnancy.

Which of the following findings is considered normal and expected when the nurse is performing a physical examination on a pregnant woman?a.A palpable, full thyroidb.Spontaneously bleeding gingivac.Significant, diffuse enlargement of the thyroidd.Pale, hypertrophied mucous membranes of the mouth

When auscultating the anterior thorax of a pregnant woman, the nurse notes the presence of a murmur over the second, third, and fourth intercostal spaces. It is continuous, but can be obliterated by pressure with the stethoscope or finger on the thorax just lateral to the murmur. The nurse knows that this is:a.the murmur of aortic stenosis.b.most likely a mammary souffle.c.associated with aortic insufficiency.d.an indication of a patent ductus arteriosis.

When the nurse is assessing the deep tendon reflexes (DTRs) of a 32-week pregnant woman, which of the following findings would be considered normal, on a 0–4+ scale?a.Absent DTRsb.2+c.4+d.Brisk reflexes and the presence of clonus

When performing an examination of a 34-week pregnant woman, the nurse notes that as the woman raises her head and shoulders off of the bed, there is a midline linear protrusion in the abdomen over the area of the rectus abdominis muscles. The nurse would:a.document the presence of diastasis rectus abdominis.b.discuss this condition with the physician, because it will most likely need to be surgically repaired.c.suspect that the woman has a hernia from the increased pressure within the abdomen from pregnancy.d.tell the woman that she may have a difficult time with delivery because of the weakness in her abdominal muscles.

When palpating the fundus, the nurse knows that:a.it should be hard and slightly tender to palpation during the first trimester.b.fetal movement should be felt by the examiner at the beginning of the second trimester.c.after 20 weeks’ gestation, the number of centimetres should approximate the number of weeks of gestation.d.fundal height is usually less than the number of weeks of gestation, unless there is an abnormal condition such as the presence of too much amniotic fluid.

The nurse is palpating the abdomen of a woman who is 35 weeks pregnant, and notes that the fetal head is facing downward toward the pelvis. The nurse would document this as:a.fetal lie.b.fetal variety.c.fetal attitude.d.fetal presentation.

During the health history of a woman who is pregnant with her first child, the woman states, “I just cannot stop crunching on ice! What’s wrong with me?” The nurse recognizes that:a.she is experiencing a common food intolerance.b.the woman is experiencing pica, or craving for nonfood items, which is sometimes associated with anemia.c.she may be experiencing gastrointestinal changes associated with pregnancy.d.she probably craved ice before her pregnancy, and this is nothing new.

Which of the following findings would be most consistent with an 8-week pregnant uterus?a.The uterus seems slightly enlarged and softened.b.It reaches to the pelvic brim, and is about the size of a grapefruit.c.It rises above the pelvic brim, and is about the size of a cantaloupe.d.It is approximately 8 cm across the fundus, and is about the size of an avocado.

A woman in week 25 of her pregnancy has come to the clinic with a complaint of 3 weeks of bouts of severe vomiting. The nurse notes that she is showing signs of dehydration. Her blood pressure is lower than usual, and she is extremely fatigued. The nurse recognizes that this patient is experiencing:a.preeclampsia.b.polyhydramnios.c.proteinuria.d.hyperemesis.

Which of the following time periods correctly describes the average length of pregnancy?a.38 weeksb.9 lunar monthsc.280 days from the last day of the last menstrual periodd.280 days from the first day of the last menstrual period

A patient’s pregnancy test is positive, and she wants to know when the baby is due. The first day of her last menstrual period was June 14, and that period ended June 20. What is her expected date of delivery, using Nägele’s rule?a.March 7b.March 14c.March 21d.March 27

A patient, who is 24 weeks pregnant, asks about wearing a seatbelt while driving. The nurse should reply:a.“Seatbelts should not be worn during pregnancy.”b.“Place the lap belt below the uterus, and use the shoulder strap at the same time.”c.“Place the lap belt below the uterus, but omit the shoulder strap during pregnancy.”d.“Place the lap belt at your waist above the uterus, and use the shoulder strap at the same time.”

A 25-year-old woman is visiting the clinic for her first prenatal visit. Which laboratory screening is appropriate at this time?a.Human chorionic gonadotropinb.Complete blood cell countc.Alpha-fetoproteind.Carrier screening for cystic fibrosis

A woman, who is in week 25 of her pregnancy, comes to the clinic for her prenatal visit. The nurse notes that her face is swollen and that her blood pressure is 144/94 mm Hg. She states that she has had headaches and blurry vision, but thought she was just tired. What should the nurse suspect?a.Eclampsiab.Preeclampsiac.Diabetes type 1d.Premature labour

In week 34 of pregnancy, a woman she is told that she has preeclampsia. The nurse knows that which of the following statements about preeclampsia is true?a.Preeclampsia has little effect on the fetus.b.Edema is one of the main indications of preeclampsia.c.Eclampsia only occurs before delivery of the baby.d.Untreated preeclampsia may progress to eclampsia, which is manifested by generalized tonic-clonic seizures

During an internal examination of a woman during her first prenatal visit, the nurse notes that the cervix is soft. This is known as:a.Hegar’s sign.b.Chadwick’s sign.c.Homan’s sign.d.Goodell’s sign.

During a group prenatal teaching session, the nurse teaches Kegel exercises. Which of the following statements would be appropriate for this teaching session? Select all that apply.a.“Kegel exercises help to keep your uterus strong during the pregnancy.”b.“Kegel exercises should be performed twice a day.”c.“Kegel exercises should be performed 50 to 100 times a day.”d.“To perform Kegel exercises, squeeze slowly to a peak at the count of eight, then release slowly to a count of eight.”e.“To perform Kegel exercises, perform rapidly alternating squeeze-release exercises up to the count of eight.”

During an assessment, the nurse is unable to palpate pulses in the left lower leg. The nurse should:a.document that the pulses are not palpable.b.reassess the pulses in 1 hour.c.have the patient turn to the side, and then palpate for the pulses again.d.use a Doppler device to assess the pulses.

During a morning assessment, the nurse notes that a patient’s urine output is below the expected amount. What should be done next?a.An order for a Foley catheter should be obtained.b.An order for a straight catheter should be obtained.c.Perform a bladder scan test.d.Refer the patient to a urologist.

What should the nurse assess before entering the patient’s room on morning rounds?a.Posted conditions, such as isolation precautionsb.The patient’s input and output chart from the previous shiftc.The patient’s general appearanced.The presence of any visitors in the room

The nurse has administered a pain medication to a patient by intravenous infusion. The nurse should reassess the patient’s response to the pain medication within:a.5 minutes.b.15 minutes.c.30 minutes.d.60 minutes.

During an assessment of a hospitalized patient, the nurse pinches a fold of skin under the clavicle or on the forearm to note:a.mobility and turgor.b.the patient’s response to pain.c.the percentage of the patient’s fat-to-muscle ratio.d.the presence of edema.

When assessing the neurological system of a hospitalized patient during morning rounds, the nurse will include which of the following assessments?a.Blood pressureb.The patient’s rating of pain on a 1 to 10 scalec.The patient’s ability to communicated.The patient’s personal hygiene level

When assessing a patient’s general appearance, the nurse will include which of the following assessments?a.Is the patient’s muscle strength equal in both arms?b.Is ptosis or facial droop present?c.Does the patient respond appropriately to questions?d.Are the pupils equal in reaction and size?

Which statement reflects the assessment of a patient in the hospital setting?a.The patient will need a brief assessment at least every 4 hours.b.The patient will need a consistent, specialized exam every 8 hours that focuses on certain parameters.c.The patient will need a complete head-to-toe physical examination every 24 hours.d.Most patients require a minimal examination during each shift, unless they are in critical condition.

The nurse is assessing the intravenous (IV) infusion at the beginning of the shift. Which of the following should be included in the assessment of the infusion? Select all that apply.a.The proper IV solution is infusing, according to physician’s orders.b.Infusion is occurring at the proper rate, according to physician’s orders.c.The infusion solution and rate is proper, according to the nurse’s own assessment of the patient’s needs.

An 85-year-old man has come into the clinic for a physical examination, and the nurse notes that he uses a cane. When documenting general appearance, the nurse will document this information under the section that covers:a.posture.b.mobility.c.mood and affect.d.physical deformity.

After the health history has been obtained, and before beginning the physical examination, the nurse should ask the patient to first:a.empty the bladder.b.completely disrobe.c.lie on the examination table.d.walk around the room.

While the nurse palpates the maxillary sinuses, the patient tells the nurse that he has some tenderness in that area. The nurse should proceed by:a.tapping on the sinus area.b.auscultating the sinus area.c.asking him to blow his nose.d.transilluminating the sinuses.

A patient states “whenever I open my mouth real wide, I feel this popping sensation in front of my ears.” To further examine this, the nurse would:a.place the stethoscope over the temporomandibular joint, and listen for bruits.b.place the hands over his ears, and ask him to open his mouth “really wide.”c.place one hand on his forehead and the other on his jaw, and ask him to try to open his mouth.d.place a finger on his temporomandibular joint, and ask him to open and close his mouth.

The nurse has just completed an examination of a patient’s extraocular muscles. When documenting the findings, the nurse would note the assessment of which cranial nerves?a.II, III, VIb.II, IV, Vc.III, IV, Vd.III, IV, VI

A patient’s uvula rises midline when she says “ahh,” and she has a positive gag reflex. The nurse has just tested which cranial nerves?a.IX, Xb.IX, XIIc.X, XIId.XI, XII

During an examination, the nurse notices that a patient is unable to stick out his tongue. Which cranial nerve is involved with the successful performance of this action?a.Ib.Vc.XId.XII

A patient is unable to shrug her shoulders against the nurse’s resistant hands. What cranial nerve is involved with successful shoulder shrugging?a.VIIb.IXc.XId.XII

During an examination, a patient has successfully completed the finger-to-nose and the rapid-alternating-movements tests, and is able to run each heel down the opposite shin. The nurse will conclude that the patient’s:a.occipital function is intact.b.cerebral function is intact.c.temporal function is intact.d.cerebellar function is intact.

A 5-year-old child is in the clinic for a checkup. The nurse would expect him to:a.have to be held on his mother’s lap.b.be able to sit on the examination table.c.be able to stand on the floor for the examination.d.be able to remain alone in the examination room.

When the nurse performs the confrontation test, the nurse has assessed:a.EOMs.b.PERRLA.c.near vision.d.visual fields.

Which of the following statements is true regarding the recording of data from the health history and physical examination?a.Use long, descriptive sentences to document findings.b.Record the data as soon as possible after the interview and physical examination.c.If the information is not documented, it can be assumed that it was done as a standard of care.d.The examiner should avoid taking any notes during the history and examination because of the possibility of decreasing rapport with the patient.

Which of the following is included in assessment of general appearance?a.Heightb.Weightc.Skin colourd.Vital signs

Gloves should be worn for which of the following examinations?a.Measurement of vital signsb.Palpation of the sinusesc.Palpation of the mouth and tongued.Inspection of the eye with an ophthalmoscope

Which of the following is an appropriate location for eliciting deep tendon reflexes?a.Achillesb.Femoralc.Scapulard.Abdominal

During inspection of a patient’s face, the nurse notes that the facial features are symmetrical. This finding indicates that which cranial nerve is intact?a.VIIb.IXc.XId.XII

During inspection of the posterior chest, the nurse should assess for:a.symmetrical expansion.b.symmetry of the shoulders and muscles.c.tactile fremitus.d.diaphragmatic excursion.

When assessing the neonate, the nurse should use which of the following methods to test for hip stability?a.Elicit the Moro reflexb.Perform Romberg’s testc.Check for Ortolani’s signd.Assess the stepping reflex

During an examination, the patient tells the nurse that she sometimes feels as if objects are spinning around her. The nurse would note that she occasionally experiences:a.vertigo.b.tinnitus.c.syncope.d.dizziness.

A patient tells the nurse that “sometimes I wake up at night and I have real trouble breathing. I have to sit up in bed to get a good breath.” When documenting this information, the nurse would note:a.orthopnea.b.acute emphysema.c.paroxysmal nocturnal dyspnea.d.acute shortness of breath episode.

During the examination of a patient, the nurse notes that the patient has several small, flat macules on her posterior thorax. These macules are less than 1 cm wide. Another name for these macules is:a.warts.b.bullas.c.freckles.d.papules.

During an examination, the nurse notes that a patient’s legs turn white when they are raised above her head. The nurse would suspect:a.lymphedema.b.Raynaud’s disease.c.chronic arterial insufficiency.d.chronic venous insufficiency.

The nurse documents that a patient has coarse, thickened skin and brown discoloration over the lower legs. Pulses are present. This finding is likely the result of:a.lymphedema.b.Raynaud’s disease.c.chronic arterial insufficiency.d.chronic venous insufficiency.

The nurse notes that a patient has ulcerations on the tips of the toes and on the lateral ankles. This finding would indicate:a.lymphedema.b.Raynaud’s disease.c.arterial insufficiency.d.venous insufficiency.

When the nurse flexes the patient’s knee and gently compresses the gastrocnemius muscle anteriorly against the tibia, the patient indicates that he is having calf pain. The nurse would document:a.positive Allen’s sign.b.negative Allen’s sign.c.positive Homan’s sign.d.negative Homan’s sign.

The nurse has just recorded a positive obturator test on a patient who has abdominal pain. This test is used to confirm a(n):a.inflamed liver.b.perforated spleen.c.perforated appendix.d.enlarged gallbladder.

The nurse is documenting the assessment of an infant. During the abdominal assessment, the nurse notes a very loud splash auscultating over the upper abdomen when the nurse rocks her from side to side. This finding would indicate:a.epigastric hernia.b.pyloric obstruction.c.hypoactive bowel sounds.d.hyperactive bowel sounds.

If the nurse records the results to the Hirschberg test, the nurse has:a.tested the patellar reflex.b.assessed for appendicitis.c.tested the corneal light reflex.d.assessed for thrombophlebitis.

During the examination of a patient’s mouth, the nurse observes a nodular, bony ridge down the middle of the hard palate. The nurse would chart this finding as:a.cheilosis.b.leukoplakia.c.ankyloglossia.d.torus palatinus.

During an examination, the nurse finds that a patient is unable to distinguish objects placed in his hand. The nurse would document:a.stereognosis.b.astereognosis.c.graphesthesia.d.agraphesthesia.

After the examination of an infant, the nurse notes opisthotonos. The nurse recognizes that this finding often occurs with:a.cerebral palsy.b.meningeal irritation.c.lower motor neuron lesion.d.upper motor neuron lesion.

After assessing a female patient, the nurse notes flesh-coloured, soft, pointed, moist papules in a cauliflower-like patch around her introitus. This finding is most likely:a.a urethral caruncle.b.a syphilitic chancre.c.herpes.d.human papillomavirus.

While examining a 48-year-old patient’s eyes, the nurse notes that he has to move the hand-held vision screener farther away from his face. The nurse would suspect:a.myopia.b.omniopia.c.hyperopia.d.presbyopia.

Which of the following assessments is most appropriate to perform on a 9-month-old well child?a.Assessment for Ortolani’s signb.Assessment for stereognosisc.Assessment of blood pressured.Assessment for the presence of the startle reflex

The female structure that corresponds with the male penis is called the:a.labia.b.clitoris.c.prepuce.d.frenulum.

When observing the vestibule, the nurse should be able to see the:a.urethral meatus and vaginal orifice.b.vaginal orifice and vestibular (Bartholin’s) glands.c.urethral meatus and paraurethral (Skene’s) glands.d.paraurethral (Skene’s) and vestibular (Bartholin’s) glands.

During an inspection of the vagina, the nurse would expect to see what at the end of the vagina?a.Cervixb.Uterusc.Ovariesd.Fallopian tubes

The uterus is usually positioned tilting forward and superior to the bladder. This position is known as:a.anteverted and anteflexed.b.retroverted and anteflexed.c.retroverted and retroflexed.d.superior verted and anteflexed.

An 11-year-old girl is in the clinic for a sports physical. The nurse notes that she has begun to develop breasts, and during the conversation the girl reveals that she is unsure about the progression of development. The nurse should use which of the following to best assist the young girl in understanding the expected sequence for development?a.The nurse should use Tanner’s table on the five stages of sexual development.b.The nurse should describe her development and compare it with that of other girls her age.c.Jacobsen’s table on expected development on the basis of height and weight data should be used.d.The nurse should reassure her that her development is within normal limits, and tell her not to worry about the next step.

A woman who is 8 weeks pregnant is in the clinic for a check. The nurse reads on her chart that her cervix is softened and looks cyanotic. The nurse knows that the woman is exhibiting which of the following signs?a.Tanner’s sign and Hegar’s signb.Hegar’s sign and Goodell’s signc.Chadwick’s sign and Hegar’s signd.Goodell’s sign and Chadwick’s sign

The changes normally associated with menopause occur generally because the cells in the reproductive tract are:a.aging.b.becoming fibrous.c.estrogen dependent.d.able to respond to estrogen.

Which of the following are changes associated with menopause?a.Uterine and ovarian atrophy, along with thinning vaginal epitheliumb.Ovarian atrophy, increased vaginal secretions, and increasing clitoral sizec.Cervical hypertrophy, ovarian atrophy, and increased acidity of vaginal secretionsd.Vaginal mucosa fragility, increased acidity of vaginal secretions, and uterine hypertrophy

A 54-year-old woman who has just completed menopause is visiting the clinic for a yearly physical examination. Which of the following should the nurse include in patient education?a.A postmenopausal woman is not at any greater risk for heart disease than a younger woman is.b.A postmenopausal woman should be aware that she is at increased risk for dyspareunia because of decreased vaginal secretions.c.A postmenopausal woman has only stopped menstruating; there really are no other significant changes that she should be concerned with.d.A postmenopausal woman is likely to have difficulty with sexual pleasure as a result of drastic changes in the female sexual response cycle.

A patient has had three pregnancies and two live births. How would the nurse record this information?a.Gravida 2, para 2, AB 1b.Gravida 3, para 2, AB 0c.Gravida 3, para 2, AB 1d.Gravida 3, para 3, AB 1 (adsbygoogle = window.adsbygoogle || []).push({});

During the interview with a female patient, the nurse gathers data that indicate that the patient is perimenopausal. Which of the following statements made by this patient leads to this conclusion?a.“I have noticed that my muscles ache at night when I go to bed.”b.“I will be very happy when I can stop worrying about having a period.”c.“I have been noticing that I sweat a lot more than I used to, especially at night.”d.“I have only been pregnant twice, but both times I had breast tenderness as my first symptom.”

A 50-year-old woman calls the clinic because she has noticed some changes in her body and breasts, and wonders if they could be due to the estrogen replacement therapy she started 3 months ago. The nurse should tell her:a.“Estrogen replacement therapy is at such a low dose that side effects are very unusual.”b.“Estrogen replacement therapy has several side effects, including fluid retention, breast tenderness or enlargement, and vaginal bleeding.”c.“It would be very unusual to have breast tenderness with estrogen replacement therapy, and I suggest you contact your doctor immediately to have this evaluated.”d.“It sounds as if your dose of estrogen is too high. I think you should decrease the amount you are taking, and should call me back in a month to let me know how you’re feeling.”

A 52-year-old patient states that when she sneezes or coughs, she “wets herself a little.” She is very concerned that something may be wrong with her. The nurse knows that the problem is:a.hematuria, and usually needs to be evaluated by a urologist.b.stress incontinence, and is usually due to muscle weakness.c.true urinary incontinence, and may mean that she has a kidney infection.d.urgency incontinence, and she should empty her bladder before she sneezes or coughs.

During the interview, a patient reveals that she has some vaginal discharge. She is worried that it may be a sexually transmitted infection. The nurse’s most appropriate response to this would be:a.“Oh, don’t worry. Some cyclic vaginal discharge is normal.”b.“Have you been engaging in unprotected sexual intercourse?”c.“I’d like some information about the discharge. What colour is it?”d.“Have you had any urinary incontinence associated with the discharge?”

A woman states that 2 weeks ago she had a urinary tract infection that was treated with an antibiotic. As a part of the interview, the nurse must make sure to ask:a.“Have you noticed a change in your vaginal pH?”b.“Have you noticed any excessive vaginal bleeding?”c.“Have you noticed any unusual vaginal discharge or itching?”d.“Have you noticed any changes in your desire for intercourse?”

Which statement would be most appropriate when the nurse is introducing the topic of sexual relationships during an interview?a.“Now, it’s time to talk about your sexual history. When did you first have intercourse?”b.“Women often feel dissatisfied with their sexual relationships. Would it be okay to discuss this now?”c.“Often women have questions about their sexual relationships and how they affect their health. Do you?”d.“Most women your age have had more than one sexual partner. How many would you say you have had?”

A 22-year-old woman has been considering using oral contraceptives. As a part of her history, the nurse should ask:a.“Do you have a history of heart murmurs?”b.“Will you be in a monogamous relationship?”c.“I wonder if you have thought this choice through carefully.”d.“If you smoke, how many cigarettes do you smoke per day?”

A nurse is assessing a patient’s risk of contracting a sexually transmitted infection (STI). An appropriate question would be:a.“You use condoms, don’t you?”b.“Do you use a condom at each episode of sexual intercourse?”c.“Do you have an STI?”d.“You are aware of the dangers of unprotected sex, aren’t you?”

When the nurse is interviewing a preadolescent girl, which of the following opening statement would be least threatening?a.“Do you have any questions about growing up?”b.“What has your mother told you about growing up?”c.“When did you notice that your body was changing?”d.“I remember being very scared when I got my period. How do you think you’ll feel?”

When the nurse is discussing sexuality and sexual issues with adolescents, a permission statement helps to convey that it is normal to think or feel a certain way. Which of the following is the best example of a permission statement?a.“It’s okay that you have become sexually active.”b.“Often girls your age have questions about sexual activity. Do you?”c.“If it’s okay with you, I’d like to ask you some questions about your sexual history.”d.“Often girls your age engage in sexual activity. It’s okay to tell me if you have had intercourse.”

Which of the following statements is true with regard to the history of a postmenopausal woman?a.The nurse should ask a postmenopausal woman if she ever has vaginal bleeding.b.Once a woman reaches menopause, the nurse does not need to ask any further history questions.c.The nurse should screen for monthly breast tenderness.d.Postmenopausal women are not at risk for contracting sexually transmitted infections, and thus these questions can be omitted.

During the examination portion of a patient’s visit, she will be in the lithotomy position. Which of the following statements reflect some things that the nurse can do to make this more comfortable for her?a.Ask her to place her hands and arms behind her head.b.Elevate her head and shoulders to maintain eye contact.c.Allow her to choose to have her feet in the stirrups or have them resting side by side on the edge of the table.d.Allow her to keep her buttocks about 15 cm (6 in.) from the edge of the table to prevent her from feeling as if she will fall off.

The nurse has just completed an inspection of a woman’s external genitalia. Which of the following describes a finding within normal limits?a.Redness of the labia majorab.Multiple nontender sebaceous cystsc.Discharge that is sticky and yellow-greend.Swelling of the perineum before onset of menses

The order of examination of the internal genitalia is important. The nurse will use which order of examination at this time?a.Bimanual examination, speculum examination, rectovaginal examinationb.Speculum examination, rectovaginal examination, bimanual examinationc.Speculum examination, bimanual examination, rectovaginal examinationd.Rectovaginal examination, bimanual examination, speculum examination

During an internal examination of a woman’s genitalia, the nurse will use which technique for proper insertion of the speculum?a.Instruct the woman to bear down, open the speculum blades, and apply in a swift, upward movement.b.Insert the blades of the speculum on a horizontal plane, turning them to a 45-degree angle while continuing to insert them. Ask the woman to bear down to ease insertion.c.Push the introitus down and open, instruct the woman to bear down, and insert the speculum with the width of the blades at an oblique angle, applying any pressure downward.d.Lock the blades open by turning the thumbscrew. Once the blades are open, apply pressure to the introitus and insert the blades at a 45-degree angle downward to bring the cervix into view.

The nurse is examining a 35-year-old female patient. During the history, the nurse notes that she has had two term pregnancies, with both babies delivered vaginally. The nurse observes the following on internal examination: the cervical os is a horizontal slit with some healed lacerations; the cervix has some nabothian cysts that are small, smooth, and yellow; the cervical surface is granular and red, especially around the os; and the presence of stringy, opaque, odorless secretions. Which of these findings are abnormal?a.The presence of nabothian cystsb.The cervical os is a horizontal slitc.The cervical surface is granular and redd.The presence of stringy and opaque secretions

A patient calls the clinic for instructions before having a Papanicolaou (Pap) smear. The most appropriate instructions from the nurse would be:a.“If you are menstruating, please use pads to avoid placing anything into the vagina.”b.“Avoid intercourse, inserting anything into the vagina, or douching within 24 hours of your appointment.”c.“If you suspect that you have a vaginal infection, please gather a sample of the discharge to bring with you.”d.“We would like you to use a mild saline douche before your examination. You may pick this up in our office.”

Which of the following tests are usually collected when screening for cervical cancer?a.Endocervical specimen, cervical scrape, and vaginal poolb.Endocervical specimen, vaginal pool, and acetic acid washc.Endocervical specimen, KOH prep, and acetic acid washd.Cervical scrape, acetic acid wash, and saline mount (“wet prep”)

In performing the bimanual examination, the nurse notes that the cervix feels smooth and firm, is round, and is fixed in place (does not move). When cervical palpation is performed, the patient complains of some pain. Which of the following should be the nurse’s interpretation of these results?a.These findings are all within normal limits.b.The cervical consistency should be soft and velvety, not firm.c.The cervix should move when palpated; an immobile cervix may indicate malignancy.d.It is unusual to have pain when the cervix is palpated, but the rest of the findings are within normal limits.

A 65-year-old woman is visiting the office for routine gynecological care. She had a complete hysterectomy 3 months ago. The nurse knows that which of the following statements is true with regard to this visit?a.Her cervical mucosa will be red and look dry.b.She will not have a cervix, and thus does not need to have a Pap smear done.c.The nurse can expect to find that her uterus will be somewhat enlarged, and her ovaries will be small and hard.d.The nurse should plan to lubricate the instruments and the examining hand well in order to avoid a painful examination.

The nurse is preparing to examine the external genitalia of a school-age girl. Which of the following positions would be most appropriate in this situation?a.In the parent’s lapb.In a frog-leg position on the examining tablec.In the lithotomy position with the feet in stirrupsd.Lying flat on the examining table with legs extended

When assessing a newborn infant’s genitalia, the nurse notes that the genitalia are somewhat engorged. The labia majora are swollen, the clitoris looks large, and the hymen is thick. The vaginal opening is difficult to visualize. The infant’s mother states that she is worried about the labia being swollen. The nurse should reply:a.“This is a normal finding in newborns, and should resolve within a few weeks.”b.“This could possibly indicate an abnormality, and may need to be evaluated by a physician.”c.“We will need to have estrogen levels evaluated to make sure that they are within normal limits.”d.“We will need to keep close watch over the next few days to see if the genitalia decrease in size.”

During a vaginal examination of a 38-year-old woman, the nurse notes that the vulva and vagina are erythematous and edematous, with thick, white, curdlike discharge adhering to the vaginal walls. The woman reports intense pruritus and thick white discharge from her vagina. The nurse knows that these history and physical examination findings are most consistent with which of the following?a.Candidiasisb.Trichomoniasisc.Atrophic vaginitisd.Bacterial vaginosis

When performing an external genitalia examination for a 10-year-old girl, the nurse notes no pubic hair, and that the mons and the labia are covered with fine vellus hair. These findings are consistent with which stage of sexual maturity, according to the Sexual Maturity Rating scale?a.Stage 1b.Stage 2c.Stage 3d.Stage 4

A 46-year-old woman is in the clinic for her annual gynecological examination. She voices a concern about ovarian cancer, because her mother and sister died of it. The nurse knows that which of the following statements is correct regarding ovarian cancer?a.Ovarian cancer is manifested by severe abdominal pain.b.The Pap smear detects the presence of ovarian cancer.c.Women over 40 years of age should have a thorough pelvic examination every year.d.Women over 40 years of age should have a thorough pelvic examination every 3 years.

During a bimanual examination, the nurse detects a solid tumour on the patient’s ovary that is heavy and fixed, with a poorly defined mass. This finding is suggestive of:a.an ovarian cyst.b.endometriosis.c.ovarian cancer.d.an ectopic pregnancy.

A 25-year-old woman comes to the emergency department with a sudden fever of 38°C and abdominal pain. Upon examination, the nurse notes that she has rigid, boardlike lower abdominal musculature. When the nurse tries to perform a vaginal examination, the patient has severe pain when the uterus and cervix are moved. The nurse knows that these signs and symptoms are suggestive of:a.endometriosis.b.uterine fibroids.c.ectopic pregnancy.d.pelvic inflammatory disease.

During a female external genitalia examination, the nurse notes several lesions around the woman’s vulva. The lesions are pink, moist, soft, pointed papules. The patient states that she is not aware of any problems in that area. The nurse recognizes that these lesions may be:a.syphilitic chancre.b.herpes simplex virus type 2 (herpes genitalis).c.human papillomavirus (HPV), or genital warts.d.pediculosis pubis (crab lice).

During an examination, the nurse would expect the cervical os of a woman who has never had children to appear:a.stellate.b.small and round.c.as a horizontal irregular slit.d.everted.

A woman has just been diagnosed with HPV, or genital warts. The nurse should counsel her to receive regular examinations, because this virus makes her at a higher risk for:a.uterine cancer.b.cervical cancer.c.ovarian cancer.d.endometrial cancer.

Which of the following statements about the anal canal is true?a.The anal canal is about 2 cm long in the adult.b.The anal canal slants backward toward the sacrum.c.The anal canal contains hair and sebaceous glands.d.The anal canal is the outlet for the gastrointestinal tract.

Which of the following statements about the sphincters is true?a.The internal sphincter is under voluntary control.b.The external sphincter is under voluntary control.c.Both sphincters remain slightly relaxed at all times.d.The internal sphincter surrounds the external sphincter.

The nurse is performing an examination of the anus and rectum. Which of the following is important to remember during this examination?a.The rectum is about 8 cm long.b.The anorectal junction cannot be palpated.c.Above the anal canal, the rectum turns anteriorly.d.There are no sensory nerves in the anal canal or rectum.

A 46-year-old man requires assessment of his sigmoid colon. Which of the following is most appropriate for this examination?a.A proctoscopeb.An ultrasoundc.A colonoscoped.A rectal exam with an examining finger

A 60-year-old man has just been told he has benign prostatic hypertrophy. He has a friend who just died from prostate cancer, and is concerned this will happen to him. How should the nurse respond?a.“The swelling in your prostate is only temporary, and will go away.”b.“We will treat you with chemotherapy so we can control the cancer.”c.“It would be very unusual for a man your age to have prostate cancer.”d.“The enlargement of your prostate is caused by hormone changes and not cancer.”

A 30-year-old woman is visiting the clinic for a complaint of “pain in my bottom when I have a bowel movement.” The nurse should assess for which problem?a.Pinwormsb.Hemorrhoidsc.Food poisoningd.Fecal incontinence

After completing an assessment of a 60-year-old man with a family history of colon cancer, the nurse discusses early detection measures for colon cancer with him. The nurse is sure to mention the need for a(n):a.annual proctoscopy.b.colonoscopy every 5 years.c.fecal test for blood every 2 years.d.digital rectal examination every 2 years.

The mother of a 5-year-old girl tells the nurse that she has noticed her daughter “scratching at her bottom a lot the last few days.” During the assessment, the nurse finds redness and raised skin in the anal area. This most likely indicates:a.pinworms.b.chickenpox.c.child abuse.d.bacterial infection.

The nurse is examining only the rectal area of a woman. In what position should the woman be placed?a.The lithotomy positionb.Lying in the prone positionc.The left lateral decubitus positiond.Bending over the table while standing

While doing an assessment of the perianal area of a patient, the nurse notes that the pigmentation of the anus is darker than the surrounding skin, that the anal opening is closed, and the presence of a skin sac that is shiny and blue. The patient mentions that he has had pain with bowel movements, and has noted some occasional spots of blood. What would this assessment and history most likely indicate?a.Anal fistulab.Pilonidal cystc.Rectal prolapsed.Thrombosed hemorrhoid

Which of the following techniques is correct for palpation of the rectum?a.Flex the finger and insert slowly toward the umbilicus.b.First instruct the patient that this will be a painful procedure.c.Insert an extended index finger at a right angle to the anus.d.Place the finger directly into the anus to overcome the tight sphincter.

While performing a rectal examination, the nurse notes a firm, irregularly shaped mass. What should the nurse do next?a.Continue with the exam, and note the finding in the chart.b.Instruct the patient to return for a repeat assessment in 1 month.c.Tell the patient that a mass was felt, but that it is nothing to worry about.d.Report the finding, and refer the patient to a specialist for further examination.

When testing stool for occult blood, the nurse is aware that a false-positive result may occur with:a.absent bile pigment.b.increased fat content.c.increased ingestion of iron medication.d.a large amount of red meat within the last 3 days.

During an assessment of the newborn, the nurse expects to see which finding when the anal area is slightly stroked?a.A jerking of the legsb.Flexion of the kneesc.A quick contraction of the sphincterd.Relaxation of the external sphincter

During an assessment of a 20-year-old man, the nurse finds a small, palpable lesion, containing a tuft of hair, located directly over the coccyx. The nurse knows that this lesion would most likely be a:a.polyp.b.pruritus ani.c.benign tumour.d.pilonidal cyst.

During an examination, the nurse asks the patient to perform the Valsalva manoeuvre and notes that the patient has a moist, red, doughnut-shaped protrusion from the anus. The nurse knows that this would be consistent with:a.a rectal polyp.b.hemorrhoids.c.a rectal fissure.d.rectal prolapse.

A 70-year-old man is visiting the clinic for “difficulty in passing urine.” In the history, he indicates he has to urinate frequently, especially at night. He experiences burning when he urinates, and has noticed pain in his back. Given this history, what might the nurse expect to find during the physical assessment?a.Asymmetric, hard, fixed prostate glandb.Occult blood and perianal pain with palpationc.Symmetrically enlarged, soft prostate glandd.A soft nodule protruding from rectal mucosa

A 40-year-old male of African descent is in the office for his annual physical. Which statement is true regarding screening for prostate cancer, according to the Canadian Cancer Society?a.DRE is more effective than PSA in finding prostate cancer.b.Ancestry is a factor in determining when PSA testing begins.c.PSA will be done at 50 years of age if there is a family history.d.BPH increases an individual’s risk for prostate cancer.

A 62-year-old man is experiencing fever, chills, malaise, and urinary frequency and urgency. He also reports urethral discharge and a dull, aching pain in the perineal and rectal area. These symptoms are most consistent with which of the following?a.Prostatitisb.Urinary tract infectionc.Carcinoma of the prostated.Benign prostatic hypertrophy

Which of the following is an abnormal finding upon palpation of the prostate gland through the rectum?a.Palpable central grooveb.Tenderness to palpationc.Heart shaped.Elastic and rubbery consistency

The nurse notes that a patient has had a pale, yellow, greasy stool, or steatorrhea, and recalls that this is caused by:a.occult bleeding.b.absent bile pigment.c.increased fat content.d.ingestion of bismuth preparations.

During a health history of a patient who complains of chronic constipation, she asks the nurse about high-fibre foods. The nurse relates that an example of a high-fibre food would be:a.broccoli.b.hamburger.c.iceberg lettuce.d.white dinner rolls.

During a digital examination of the rectum, the nurse notes that the patient has hard feces in the rectum. The patient complains of feeling “full,” has a distended abdomen, and states that she has not had a bowel movement “for several days.” The nurse suspects which condition?a.Rectal polypb.Fecal impactionc.Rectal abscessd.Rectal prolapse

The nurse is performing a digital examination of a patient’s prostate gland, and notes a normal prostate gland includes the following characteristics: Select all that apply.a.The gland is protruding 1 cm into the rectum.b.The gland is heart-shaped, with a palpable central groove.c.The gland is flat, with no palpable groove.d.The gland has a boggy and soft consistency.e.The gland has a smooth surface, and an elastic or rubbery consistency.f.The gland has fixed mobility.

The external male genital structures include the:a.testis.b.scrotum.c.epididymis.d.vas deferens.

An accessory glandular structure for the male genital organs is the:a.testis.b.penis.c.prostate.d.vas deferens.

Which of the following statements is true regarding the penis?a.The urethral meatus is located on the ventral side of the penis.b.The prepuce is the fold of foreskin covering the shaft of the penis.c.The penis is composed of two cylindrical columns of erectile tissue.d.The corpus spongiosum expands into a cone of erectile tissue called the glans.

When performing a genital examination on a 25-year-old man, the nurse notes deeply pigmented, wrinkled scrotal skin with large sebaceous follicles. On the basis of this information, the nurse would:a.squeeze the glans, checking for the presence of discharge.b.consider this a normal finding, and proceed with the examination.c.assess the testicles for the presence of masses or painless lumps.d.obtain a more detailed health history, focusing on any scrotal abnormalities the patient has noticed.

Which of the following statements about the testes is true?a.The lymphatics of the testes drain into the abdominal lymph nodes.b.The vas deferens is located along the inferior portion of each testis.c.The right testis is lower than the left because the right spermatic cord is longer.d.The cremaster muscle contracts in response to cold, and draws the testicles closer to the body.

A male patient with possible fertility problems asks the nurse where sperm is produced. The nurse knows that sperm production occurs in the:a.testes.b.prostate.c.epididymis.d.vas deferens.

A 62-year-old man states that his doctor told him that he has an “inguinal hernia.” He asks the nurse to explain what a hernia is. The nurse should:a.tell him not to worry, since most men his age develop hernias.b.explain that a hernia is often the result of prenatal growth abnormalities.c.refer him to his physician for additional consultation, because the physician made the initial diagnosis.d.explain that a hernia is a loop of bowel protruding through a weak spot in the abdominal muscles.

The mother of a 10-year-old boy asks the nurse to discuss how puberty can be recognized. The nurse should reply:a.“Puberty usually begins at about age 15.”b.“The first sign of puberty is enlargement of the testes.”c.“Penis size does not increase until about the age of 16.”d.“The development of pubic hair precedes testicular or penis enlargement.”

During an examination of an aging male, the nurse recognizes that a normal, expected change would be:a.premature ejaculation.b.declining testosterone production.c.difficulty in maintaining an erection.d.a decreased refractory state after ejaculation.

An older man is concerned about his sexual performance. The nurse knows that in the absence of disease, a withdrawal from sexual activity later in life may be due to:a.side effects of medications.b.decreased libido with aging.c.decreased sperm production.d.decreased pleasure from sexual intercourse.

A newborn baby boy is about to have a circumcision. The nurse knows that indications for circumcision include:a.cultural beliefs.b.prevention of testicular cancer.c.improving the sperm count later in life.d.preventing the transmission of human immunodeficiency virus during sexual intercourse.

A 59-year-old patient has been diagnosed with prostatitis, and is being seen at the clinic for complaints of burning and pain during urination. He is experiencing:a.dysuria.b.nocturia.c.polyuria.d.hematuria.

A 45-year-old carpenter is seen at the clinic for a complaint of “losing my urine when I lift heavy objects.” He is experiencing:a.frequency.b.urinary hesitancy.c.stress incontinence.d.urgency incontinence.

When the nurse is conducting a sexual history for a male adolescent, which statement would be most appropriate to use at the beginning of the interview?a.“Do you use condoms?”b.“You don’t masturbate, do you?”c.“Have you had sex in the last 6 months?”d.“Often boys your age have questions about sexual activity.”

Which of the following statements is most appropriate when the nurse is obtaining a genitourinary history from an elderly man?a.“Do you need to get up at night to urinate?”b.“Do you experience nocturnal emissions, or ‘wet dreams’?”c.“Do you know how to perform testicular self-examination?”d.“Has anyone ever touched your genitals when you did not want them to?”

While the nurse is performing a genital examination on a male patient, the patient has an erection. The nurse’s most appropriate action or response is to:a.ask the patient if he would like someone else to examine him.b.continue with the examination as though nothing has happened.c.stop the examination, state that the examination will resume at a later time, and leave the room.d.reassure the patient that this is a normal response, and continue with the examination.

The nurse knows that which of the following is a normal finding when examining the glans?a.The dorsal vein may be visible.b.Hair is without pest inhabitants.c.The skin is wrinkled and without lesions.d.Smegma may be present under the foreskin of an uncircumcised male.

When performing a genitourinary assessment, the nurse notes that the urethral meatus is positioned ventrally. This finding is:a.called hypospadius.b.the result of phimosis.c.probably due to a stricture.d.often associated with aging.

The nurse is performing a genital examination on a male patient, and notes urethral drainage. When collecting urethral discharge for microscopic examination and culture, the nurse should:a.ask the patient to urinate into a sterile cup.b.ask the patient to obtain a specimen of semen.c.insert a cotton-tipped applicator into the urethra.d.compress the glans between the examiner’s thumb and forefinger, and collect any discharge.

When assessing the scrotum of a male patient, the nurse notes the presence of multiple firm, nontender, yellow, 1-cm nodules. The nurse knows that these nodules are most likely:a.from urethritis.b.sebaceous cysts.c.subcutaneous plaques.d.from inflammation of the epididymis.

When performing a scrotal assessment, the nurse notes that the scrotal contents transilluminate and show a red glow. On the basis of this finding, the nurse would:a.assess the patient for the presence of a hernia.b.suspect the presence of serous fluid in the scrotum.c.consider this a normal finding, and proceed with the examination.d.refer the patient for evaluation of a possible mass in the scrotum.

When the nurse is performing a genital examination on a male patient, which of the following actions is correct?a.Auscultate for the presence of bowel sounds over the scrotum.b.Palpate for the vertical chain of lymph nodes along the groin inferior to the inguinal ligament.c.Palpate the inguinal canal only if there is a bulge present in the inguinal region during inspection.d.Have the patient shift his weight onto the left (unexamined) leg when palpating for a hernia on the right side.

The nurse is aware that which of the following statements is true regarding the incidence of testicular cancer?a.Testicular cancer is the most common cancer in men aged 30–50 years.b.The early symptoms of testicular cancer are pain and induration.c.Men with a history of cryptorchidism are at greatest risk for development of testicular cancer.d.Men of European descent are four times more likely to develop testicular cancer than men of non-European descent.

The nurse is describing how to perform a testicular self-examination to a patient. Which of the following statements is most appropriate?a.“A good time to examine your testicles is just before you take a shower.”b.“If you notice an enlarged testicle or a painless lump, call your health care provider.”c.“The testicle is egg shaped and movable. It feels firm and has a lumpy consistency.”d.“Perform a testicular exam at least once a week to detect the early stages of testicular cancer.”

A 2-month-old uncircumcised boy has been brought to the clinic for a well-baby checkup. How would the nurse proceed with his genital examination?a.Elicit the cremasteric reflex.b.Assess the glans for redness or lesions.c.Avoid retracting the foreskin until the infant is 3 months old.d.Note any dirt or smegma that has collected under the foreskin.

A 2-year-old boy has been diagnosed with “physiological cryptorchidism.” Given this diagnosis, during assessment, the nurse will most likely observe:a.testes that are hard and painful to palpation.b.an atrophic scrotum and absence of the testis bilaterally.c.an absence of the testis in the scrotum, but the testis can be milked down.d.testes that migrate into the abdomen when the child squats or sits cross-legged.

The nurse knows that a common assessment finding in a boy under 2 years old is:a.an inflamed and tender spermatic cord.b.the presence of a hernia in the scrotum.c.a penis that looks large in relation to the scrotum.d.the presence of a hydrocele, or fluid in the scrotum.

During an examination of an aging male, the nurse recognizes that a normal, expected change would be:a.a change in scrotal colour.b.a decrease in the size of the penis.c.enlargement of the testes and scrotum.d.an increase in the number of rugae over the scrotal sac.

When performing a genital assessment on a 34-year-old man, the nurse notes cauliflower-like patches of multiple soft, moist, painless scattered across the shaft of the penis. These lesions are characteristic of:a.carcinoma.b.syphilitic chancres.c.herpes progenitalis.d.genital warts

A 15-year-old boy is seen in the clinic for a complaint of “dull pain and pulling” in the scrotal area. On examination, the nurse palpates a soft, irregular mass posterior to and above the testis on the left. This mass collapses when the patient is supine and refills when he is upright. This description is consistent with:a.epididymitis.b.spermatocele.c.testicular torsion.d.a spermatic cord varicocele.

When performing a genitourinary assessment on a 16-year-old boy, the nurse notices a swelling in the scrotum that increases with intensified intra-abdominal pressure and decreases when he is lying down. The patient complains of pain when straining. The nurse knows that this description is most consistent with a(n):a.femoral hernia.b.incisional hernia.c.direct inguinal hernia.d.indirect inguinal hernia.

Which of the following findings is considered normal when the nurse is performing a testicular examination on a 25-year-old man?a.Nontender subcutaneous plaquesb.A scrotal area that is dry, scaly, and nodularc.Testes that feel oval and movable, and are slightly sensitive to compressiond.A single, hard, circumscribed, movable mass, less than 1 cm under the surface of the testes

The nurse is inspecting the scrotum and testes of a 43-year-old man. Which finding would require additional follow-up and evaluation?a.The skin on the scrotum is shiny and smooth.b.The left testicle hangs lower than the right testicle.c.The scrotum is a darker colour than the general skin colour.d.The testes move closer to the body in response to cold temperatures.

A 55-year-old man is experiencing severe, sudden onset pain onset in the scrotal area. It is somewhat relieved by elevation. On examination, the nurse notes an enlarged, red scrotum that is very tender to palpation. It is difficult to distinguish the epididymis from the testis, and the scrotal skin is thick and edematous. This description is consistent with which of the following?a.Varicoceleb.Epididymitisc.Spermatoceled.Testicular torsion

The nurse is performing a genitourinary assessment on a 50-year-old obese male labourer. On examination, the nurse notes a painless round swelling close to the pubis, in the area of the internal inguinal ring, which is easily reduced when the individual is supine. These findings are most consistent with which of the following conditions?a.A scrotal herniab.A femoral herniac.A direct inguinal herniad.An indirect inguinal hernia

Prolonged, painful erection of the penis without sexual desire is known as:a.orchitis.b.stricture.c.phimosis.d.priapism.

During an examination, the nurse notes that a male patient has a red, round, superficial ulcer with a yellowish serous discharge on his penis. On palpation, the nurse finds a nontender base that feels like a small button between the thumb and fingers. At this point the nurse suspects that this patient has:a.genital warts.b.a herpes infection.c.a syphilitic chancre.d.a carcinoma lesion.

During a health history, a patient tells the nurse that he has trouble in starting his urine stream. This problem is known as:a.urgency.b.dribbling.c.frequency.d.hesitancy.

During a genital examination, the nurse notes that a male patient has clusters of small vesicles on his glans, surrounded by erythema. The nurse recognizes that these lesions are:a.Peyronie’s disease.b.genital warts.c.genital herpes.d.syphilitic cancer.

During a physical examination, the nurse finds that a male patient’s foreskin is fixed and tight, and will not retract over the glans. The nurse recognizes that this condition is:a.phimosis.b.epispadias.c.urethral stricture.d.Peyronie’s disease.

The two parts of the nervous system are:a.motor and sensory.b.central and peripheral.c.peripheral and autonomic.d.hypothalamus and cerebral.

The wife of a 65-year-old man tells the nurse that she is concerned because she has noted a change in her husband’s personality and ability to understand. He also cries and becomes angry very easily. The nurse recalls that the cerebral lobe responsible for these behaviours is which of the following?a.Frontalb.Parietalc.Occipitald.Temporal

Which of the following statements concerning areas of the brain is accurate?a.The cerebellum is the center for speech and emotions.b.The hypothalamus controls temperature and regulates sleep.c.The basal ganglia are responsible for controlling voluntary movements.d.Motor pathways of the spinal cord and brainstem synapse in the thalamus.

While gathering equipment for an intravenous injection, a nurse accidentally received a prick from an improperly capped needle. To interpret this sensation, which of the following areas must be intact?a.Corticospinal tract, medulla, and basal gangliab.Pyramidal tract, hypothalamus, and sensory cortexc.Lateral spinothalamic tract, thalamus, and sensory cortexd.Anterior spinothalamic tract, basal ganglia, and sensory cortex

A patient with a lack of oxygen to his heart will have pain in his chest, and also possibly in his shoulders, arms, or jaw. Which of the following best explains why this occurs?a.There is a problem with the sensory cortex and its ability to discriminate the location.b.The lack of oxygen in his heart has resulted in decreased amount of oxygen to these areas.c.The sensory cortex does not have the ability to localize pain in the heart, so the pain is felt elsewhere.d.There is a lesion in the dorsal root that is preventing the sensation from being transmitted normally.

The human ability to perform very skilled movements such as writing is controlled by the:a.basal ganglia.b.corticospinal tract.c.spinothalamic tract.d.extrapyramidal tract.

A 30-year-old woman tells the nurse that she has been very unsteady and has had difficulty in maintaining her balance. With these findings, which area of the brain would concern the nurse?a.Thalamusb.Brainstemc.Cerebellumd.Extrapyramidal tract

Which of the following statements about the peripheral nervous system is correct?a.The cranial nerves enter the brain through the spinal cord.b.Efferent fibres carry sensory input to the central nervous system through the spinal cord.c.The peripheral nerves are inside the central nervous system and carry impulses through their motor fibres.d.The peripheral nerves carry input to the central nervous system by afferent fibres and away by efferent fibres.

A patient has a severed spinal nerve as a result of trauma. Which of the following statements is true in this situation?a.Because there are 31 pairs of spinal nerves, there is no effect if only one is severed.b.The dermatome served by this nerve will no longer experience any sensation.c.The adjacent spinal nerves will continue to carry sensations for the dermatome served by the severed nerve.d.This will only affect the motor function of the patient, because spinal nerves have no sensory component.

A 21-year-old patient has a head injury resulting from trauma, and is unconscious. There are no other injuries. During the assessment, what would the nurse expect to find when testing the patient’s deep tendon reflexes?a.Reflexes will be normal.b.Reflexes will not be able to be elicited.c.All reflexes would be diminished, but present.d.Some reflexes would be present, depending on the area of injury.

A mother of a 1-month-old infant asks the nurse why it takes so long for babies to learn to roll over. The nurse knows that the reason for this is that:a.there must be a demyelinating process occurring with the baby.b.myelin is needed to conduct these impulses, and the neurons of a newborn are not myelinated.c.the cerebral cortex is not fully developed, so control over motor function occurs gradually.d.the spinal cord is controlling the movement because the cerebellum is not yet fully developed.

During an assessment of an 80-year-old patient, the nurse notes the patient’s inability to identify vibrations at the ankle and to identify the position of the big toe; a slower and more deliberate gait; and slightly impaired tactile sensation. All other neurological findings are normal. The nurse knows that these findings indicate:a.cranial nerve dysfunction.b.a lesion in the cerebral cortex.c.normal changes due to aging.d.demyelinization of nerves due to a lesion.

A 70-year-old woman tells the nurse that every time she gets up in the morning or after she’s been sitting, she gets “really dizzy” and feels like she is going to “fall over.” The nurse’s best response would be:a.“Have you been extremely tired lately?”b.“You probably just need to drink more liquids.”c.“I’ll refer you for a complete neurological examination.”d.“You need to get up slowly when you’ve been lying or sitting.”

During the history, a patient tells the nurse that “it feels like the room is spinning around me.” The nurse would document this as:a.vertigo.b.syncope.c.dizziness.d.seizure activity.

When doing the history on a patient with a seizure disorder, the nurse assesses whether the patient has an aura. Which of the following would be the best question to ask in order to obtain this information?a.“Does your muscle tone seem tense or limp?”b.“After the seizure, do you spend a lot of time sleeping?”c.“Do you have any warning sign before your seizure starts?”d.“Do you experience any colour change or incontinence during the seizure?”

While obtaining a history of a 3-month old infant from its mother, the nurse asks about the baby’s ability to suck and grasp the mother’s finger. What is the nurse assessing?a.Reflexesb.Intelligencec.Cranial nervesd.Cerebral cortex function

In obtaining a history for a 74-year-old patient, the nurse notes that he drinks alcohol daily, and that he has noticed a tremor in his hands that affects his ability to hold things. With this information, what should the nurse’s response be?a.“Does your family know you are drinking every day?”b.“Does the tremor change when you drink the alcohol?”c.“We’ll do some tests to see what is causing the tremor.”d.“You really shouldn’t drink so much alcohol; it may be causing your tremor.”

A 50-year-old woman is visiting the clinic for “weakness in my left arm and leg for the past week.” The nurse will perform which type of neurological examination?a.Glasgow Coma Scaleb.Neurological recheck examinationc.Screening neurological examinationd.Complete neurological examination

The nurse is testing the function of cranial nerve XI. Which of the following best describes the response the nurse would expect if the nerve is intact?a.Demonstrates full range of motion of the neckb.Sticks tongue out midline without tremors or deviationc.Follows an object with the eyes without nystagmus or strabismusd.Moves the head and shoulders against resistance with equal strength

During the neurological assessment of a “healthy” 35-year-old patient, the nurse asks him to relax his muscles completely. The nurse then moves each extremity through the full range of motion. Which of the following would the nurse expect to find?a.Firm, rigid resistance to movementb.Mild, even resistance to movementc.Hypotonic muscles as a result of total relaxationd.Slight pain with some directions of movement

When the nurse asks a 68-year-old patient to stand with his feet together, his arms at his sides, and his eyes closed, he starts to sway and moves his feet further apart. The nurse would document this finding as a(n):a.ataxia.b.lack of coordination.c.negative Homan’s sign.d.positive Romberg’s sign.

The nurse is doing an assessment on a 29-year-old woman who visits the clinic complaining of “always dropping things and falling down.” While testing rapid alternating movements, the nurse notices that the woman is unable to pat both her knees. Her response is very slow, and she misses frequently. What might the nurse suspect?a.Vestibular diseaseb.A lesion of cranial nerve IXc.Dysfunction of the cerebellumd.Inability to understand directions

The nurse is taking the health history of a 78-year-old man. During the history, his wife states that he occasionally has problems with short-term memory loss and confusion: “He can’t even remember how to button his shirt.” In doing the assessment of his sensory system, the nurse would do which of the following?a.The nurse would not do this part of the examination because results would not be valid.b.The nurse would perform the tests, knowing that mental status does not affect sensory ability.c.The nurse would proceed with the explanations of each test, making sure the wife understands.d.Before testing, the nurse would assess the patient’s mental status and ability to follow directions at this time.

The assessment of a 60-year-old patient has taken longer than anticipated. In testing his pain perception, the nurse decides to complete the test as quickly as possible. When the nurse applies the sharp point of the pin to his arm several times, he is only able to identify these as one “very sharp prick.” What would be the most accurate explanation for this?a.The patient has hyperesthesia as a result of the aging process.b.This is most likely the result of the summation effect.c.The nurse was probably not poking hard enough with the pin in the other areas.d.The patient most likely has analgesia in some areas of his arm and hyperalgesia in others.

The nurse is performing a neurological assessment for a 41-year-old woman with a history of diabetes. When testing her ability to feel the vibrations of a tuning fork, the nurse notes that she is unable to feel vibrations on the great toe or ankle bilaterally, but is able to feel vibrations on both patellae. Given this information, what would the nurse suspect?a.Hyperalgesiab.Hyperesthesiac.Peripheral neuropathyd.Lesion of sensory cortex

The nurse places a key in the hand of a patient, and he identifies it as a penny. What term would the nurse use to describe this finding?a.Extinctionb.Astereognosisc.Graphesthesiad.Tactile discrimination

The nurse is testing the deep tendon reflexes of a 30-year-old woman during her annual physical examination. When striking the Achilles and quadriceps, the nurse is unable to elicit reflexes. The nurse’s next response should be to:a.ask the patient to lock her fingers and “pull.”b.complete the examination, and then test these reflexes again.c.refer the patient to a specialist for further testing.d.document these reflexes as “0” on a scale of 0 to 4+.

In assessing a 70-year-old patient who has had a recent cerebrovascular accident, the nurse notes right-sided weakness. What might the nurse expect to find when testing his reflexes on the right side?a.Lack of reflexesb.Normal reflexesc.Diminished reflexesd.Hyperactive reflexes

When the nurse is testing the triceps reflex, what is the expected response?a.Flexion of the handb.Pronation of the handc.Extension of the forearmd.Flexion and supination of the forearm

In the assessment of a 1-month-old infant, the nurse notes a lack of response to noise or stimulation. The mother reports that in the last week, he has been sleeping most of the time, and when awake, all he does is cry. The nurse hears that the infant’s cries are very high pitched and shrill. What would be the nurse’s appropriate response to these findings?a.Refer the infant for further testing.b.Talk with the mother about eating habits.c.Nothing; these are expected findings for an infant of this age.d.Tell the mother to bring the baby back in a week for a recheck.

Which of the following would the nurse use to test the motor coordination of an 11-month old infant?a.Denver II testb.Stereognosisc.Deep tendon reflexesd.Rapid alternating movements

To assess the head control of a 4-month-old infant, the nurse lifts the infant up in a prone position while supporting his chest. What normal response does the nurse look for?a.Raises head and arches backb.Extends arms and drops head downc.Flexes knees and elbows and keeps back straightd.Holds head at 45 degrees and keeps back straight

While assessing a 7-month-old infant, the nurse makes a loud noise and notes abduction and flexion of the arms and legs; fanning of the fingers, and curling of the index finger and thumb in a “C” position; and the infant bringing in its arms and legs towards its body. What does the nurse know about this response?a.This could indicate brachial nerve palsy.b.This is an expected startle response at this age.c.This reflex should have disappeared between 1 and 4 months of age.d.It is normal as long as movements are symmetric bilaterally.

While the nurse is taking the health history of a 68-year-old patient who sustained a head injury 3 days earlier, he tells the nurse that he is on a cruise ship and is 30 years old. The nurse knows that this finding is indicative of:a.a great sense of humour.b.uncooperative behaviour.c.inability to understand the question.d.a decreased level of consciousness.

The nurse is caring for a patient who has just had neurosurgery. To assess for increased intracranial pressure, what would the nurse include in the assessment?a.Cranial nerves, motor function, and sensory functionb.Deep tendon reflexes, vital signs, and coordinated movementsc.Level of consciousness, motor function, pupillary response, and vital signsd.Mental status, deep tendon reflexes, sensory function, and pupillary response

During an assessment of a 22-year-old woman who has a head injury from a car accident 4 hours ago, the nurse notes the following change: her pupils were equal, but now the right is fully dilated and nonreactive, while the left is 4 mm and reacts to light. What would this finding suggest?a.Injury to the right eyeb.Increased intracranial pressurec.The test was not performed accuratelyd.A normal response after a head injury

A 32-year-old woman tells the nurse that she has noticed “very sudden, jerky movements”, mainly in her hands and arms. “They seem to come and go, primarily when I am trying to do something. I haven’t noticed them when I’m sleeping.” This description suggests:a.chorea.b.athetosis.c.myoclonus.d.Parkinson’s disease.

During an assessment of a 62-year-old man the nurse notes stooped posture; shuffling, short steps when walking; a very rigid; flat facial expression; and movement of the fingers as if rolling a pill with them. These findings would be consistent with:a.parkinsonism.b.cerebral palsy.c.cerebellar ataxia.d.muscular dystrophy.

During an assessment of a 32-year-old patient with a recent head injury, the nurse notes that the patient responds to pain by extending, adducting, and internally rotating his arms. As well, his palms pronate and his lower extremities extend with plantar flexion. Which of the following statements about these findings is accurate?a.This indicates a lesion of the cerebral cortex.b.This indicates a completely nonfunctional brainstem.c.This is a normal response, and it will go away in 24 to 48 hours.d.This is a very ominous sign that may indicate brainstem injury.

A 78-year-old man has a history of a cerebrovascular accident. The nurse notes that when he walks, his left arm is immobile against the body, with flexion of the shoulder, elbow, wrist, and fingers and adduction of the shoulder. His left leg is stiff and extended, and circumducts with each step. What type of gait disturbance is this individual experiencing?a.Scissors gaitb.Cerebellar ataxiac.Parkinsonian gaitd.Spastic hemiparesis

In a person with an upper motor neuron lesion such as a cerebrovascular accident, which of the following physical assessment findings would the nurse expect to see?a.Hyperreflexiab.Fasciculationsc.Loss of muscle tone and flaccidityd.Atrophy and wasting of the muscles

A 59-year-old patient has a herniated intervertebral disk. Which of the following findings would the nurse expect to see on physical assessment of this individual?a.Hyporeflexiab.Increased muscle tonec.A positive Babinski’s signd.The presence of pathological reflexes

A patient is not able to perform rapid, alternating movements such as patting her knees quickly. The nurse would document this as:a.ataxia.b.astereognosis.c.the presence of dysdiadochokinesia.d.a probable abnormality in the cerebellum.

The nurse knows that determining whether a person is oriented to his or her surroundings will test the functioning of which of the following structures?a.Cerebrumb.Cerebellumc.Cranial nervesd.Medulla oblongata

During an examination, the nurse notes severe nystagmus in both of the patient’s eyes. Which of the following conclusions is correct?a.This is a normal occurrence.b.This may indicate disease of the cerebellum or brainstem.c.This is a sign that the patient is nervous about the examination.d.This indicates a visual problem and a referral to an ophthalmologist is indicated.

The nurse knows that testing kinesthesia is a test of a person’s:a.fine touch.b.position sense.c.motor coordination.d.perception of vibration.

The nurse knows that which of the following scores would indicate that a patient is in a coma based on the Glasgow Coma Scale criteria?a.6b.12c.15d.24

A man who was found wandering in a park at 2 a.m. has been brought to the emergency department for an examination because he said he fell and hit his head. During the examination, the nurse asks him to use his index finger to touch the nurse’s finger, then his own nose, and then the nurse’s finger again (which has been moved to a different location). The patient is clumsy, unable to follow the instructions, and overshoots the mark, missing the finger. The nurse suspects which of the following?a.Cerebral injuryb.Cerebrovascular accidentc.Acute alcohol intoxicationd.Peripheral neuropathy

A patient is being assessed for his range of joint movement. The nurse asks him to move his arm in toward the centre of his body. This movement is called:a.flexion.b.abduction.c.adduction.d.extension.

A patient tells the nurse that she is having a hard time bringing her hand to her mouth when she eats or tries to brush her teeth. The nurse knows that for the patient to move her hand to her mouth, she must perform the following movement:a.flexion.b.abduction.c.adduction.d.extension.

The functional units of the musculoskeletal system are the:a.joints.b.bones.c.muscles.d.tendons.

Hematopoiesis takes place in the:a.liver.b.spleen.c.kidneys.d.bone marrow.

Fibrous bands running directly from one bone to another that strengthen the joint and help prevent movement in undesirable directions are called:a.bursa.b.tendons.c.cartilage.d.ligaments.

The nurse notices that a woman in an exercise class is unable to jump rope. The nurse knows that to jump rope, one’s shoulder has to be capable of:a.inversion.b.supination.c.protraction.d.circumduction.

The articulation of the mandible and the temporal bone is known as the:a.intervertebral foramen.b.condyle of the mandible.c.temporomandibular joint.d.zygomatic arch of the temporal bone.

To palpate the temporomandibular joint, the nurse’s fingers should be placed in the depression:a.distal to the helix of the ear.b.proximal to the helix of the ear.c.anterior to the tragus of the ear.d.posterior to the tragus of the ear.

Of the 33 vertebrae in the spinal column, there are:a.5 lumbar vertebrae.b.5 thoracic vertebrae.c.7 sacral vertebrae.d.12 cervical vertebrae.

An imaginary line connecting the highest point on each iliac crest would cross:a.the first sacral vertebra.b.the fourth lumbar vertebra.c.the seventh cervical vertebra.d.the twelfth thoracic vertebra.

The nurse is explaining to a patient that there are “shock absorbers” in his back to cushion the spine and to help it move. The nurse is referring to his:a.costal facets.b.nucleus pulposus.c.vertebral foramen.d.intervertebral discs.

During an interview, a patient states, “I can feel this bump on the top of both of my shoulders—it doesn’t hurt but I am curious about what it might be.” The nurse should tell her:a.“That is your subacromial bursa.”b.“That is your acromion process.”c.“That is your glenohumeral joint.”d.“That is the greater tubercle of your humerus.”

The nurse is checking the range of motion in a patient’s knee, and knows that the knee is capable of the following movements:a.Flexion and extensionb.Supination and pronationc.Circumductiond.Inversion and eversion

The nurse is assessing a patient’s ischial tuberosity. To palpate the ischial tuberosity, the nurse knows that it is best to have the patient:a.stand.b.flex his hip.c.flex his knee.d.lie in the supine position.

The knee joint is the articulation of the femur, the tibia, and the:a.fibula.b.radius.c.patella.d.humerus.

The ankle joint is the articulation of the tibia, the fibula, and the:a.talus.b.cuboid.c.calcaneus.d.talocalcaneonavicular joint.

The nurse is explaining the growth of long bones to a mother of a toddler. The nurse knows that bones increase in width or diameter by deposition of new bony tissue around the shafts. The nurse also knows that lengthening occurs at the:a.bursa.b.calcaneus.c.epiphyses.d.tuberosities.

A woman who is 8 months pregnant comments that she has noticed a change in her posture and that she is having low back pain. The nurse tells her that during pregnancy, women have a posture shift to compensate for the enlarging fetus. This shift in posture is known as:a.lordosis.b.scoliosis.c.ankylosis.d.kyphosis.

A patient has been diagnosed with osteoporosis and asks the nurse, “What is osteoporosis?” The nurse knows that osteoporosis can be defined as:a.the loss of bone matrix.b.the loss of bone density.c.new, weaker bone growth.d.increased phagocytic activity.

The nurse knows that the incidence of osteoporosis is greatest in which group?a.Males of African descentb.Females of African descentc.Females of European descentd.Aboriginal males

A teenage girl is complaining of pain in her left wrist. She had been playing basketball and fell, landing on her left hand. The nurse examines her hand, and would expect a fracture if the girl complains:

A patient is complaining of pain in his joints that is worse in the morning, is better after he has moved around for awhile, and then gets worse again if he sits for long periods of time. The nurse suspects that he may have:a.tendinitis.b.osteoarthritis.c.rheumatoid arthritis.d.intermittent claudication.

A patient states, “I can hear a crunching or grating sound when I kneel.” She also states, “it is very difficult to get out of bed in the morning because of stiffness and pain in my joints.” The nurse suspects that the sound the patient hears is:a.crepitation.b.a bone spur.c.a loose tendon.d.fluid in the knee joint.

A patient is able to flex his right arm forward without difficulty or pain, but is unable to abduct his arm because of pain and muscle spasms. The nurse suspects:a.crepitation.b.rotator cuff lesions.c.dislocated shoulder.d.rheumatoid arthritis.

An 80-year-old woman is visiting the clinic for a checkup. The nurse is observing for motor dysfunction in the woman’s hip, and would have her:a.internally rotate her hip while she is sitting.b.abduct her hip while she is lying on her back.c.adduct her hip while she is lying on her back.d.externally rotate her hip while she is standing.

The nurse has completed the musculoskeletal examination of a patient’s knee and has found a positive bulge sign. The nurse suspects:a.irregular bony margins.b.soft tissue swelling in the joint.c.swelling from fluid in the epicondyle.d.swelling from fluid in the suprapatellar pouch.

During an examination, the nurse asks a patient to bend forward from the waist and notes that the patient has lateral tilting. When one of his legs is raised straight up, he complains of a pain going down the buttock into the leg. The nurse suspects:a.scoliosis.b.a meniscus tear.c.a herniated nucleus pulposus.d.a spasm of paravertebral muscles.

The nurse is examining a 3-month-old infant. While holding her thumbs on the infant’s inner mid thighs and her fingers outside on the hips, touching the greater trochanter, the nurse adducts the infant’s legs until her thumbs touch and then abducts the legs until the knees touch the table. The nurse does not note any “clunking” sounds, and is confident to record a:a.positive Allis sign.b.negative Allis sign.c.positive Ortolani sign.d.negative Ortolani sign.

During a neonatal examination, the nurse notes that the newborn infant has six toes. This finding is documented as:a.unidactyly.b.syndactyly.c.polydactyly.d.multidactyly.

A mother brings her newborn baby boy in for a checkup. She tells the nurse that he doesn’t seem to be moving his right arm as much as his left, and that he seems to have pain when she lifts him up under the arms. The nurse suspects a fractured clavicle, and would observe for:a.a negative Allis sign.b.a positive Ortolani sign.c.limited range of motion during the Moro reflex.d.limited range of motion during LaSegue’s reflex.

A 40-year-old man has come into the clinic with complaints of “extreme tenderness in my toes.” The nurse notes that his toes are slightly swollen, reddened, and warm to the touch. His complaints would suggest:a.osteoporosis.b.acute gout.c.ankylosing spondylitis.d.degenerative joint disease.

A young swimmer comes to the sports clinic complaining of a very sore shoulder. He was running at the pool, slipped on some wet concrete, and tried to catch himself with his outstretched hand. He landed on this hand, and has not been able to move his shoulder since. The nurse suspects:a.joint effusion.b.tear of rotator cuff.c.adhesive capsulitis.d.dislocated shoulder.

A 68-year-old woman has come in for an assessment of her rheumatoid arthritis, and the nurse notes raised, firm, nontender nodules at the olecranon bursa and along the ulna. These nodules are most commonly diagnosed as:a.epicondylitis.b.gouty arthritis.c.olecranon bursitis.d.subcutaneous nodules.

A woman who has had rheumatoid arthritis for years is starting to notice that her fingers are drifting to the side. The nurse knows that this condition is commonly referred to as:a.radial drift.b.ulnar deviation.c.swan neck deformity.d.Dupuytren’s contracture.

A patient who has had rheumatoid arthritis for years comes to the clinic to ask about changes in her hands. The nurse knows that changes associated with rheumatoid arthritis include:a.Heberden’s nodes.b.Bouchard’s nodules.c.swan-neck deformities.d.Dupuytren’s contractures.

A patient’s annual physical examination reveals a lateral curvature of the thoracic and lumbar segments of his spine; however, this curvature disappears with forward bending. The nurse knows that this abnormality of the spine is called:a.structural scoliosis.b.functional scoliosis.c.herniated nucleus pulposus.d.dislocated nucleus pulposus.

A 14-year-old boy who has been diagnosed with Osgood-Schlatter disease reports painful swelling just below the knee for the past 5 months. Which response by the nurse is appropriate?a.“If these symptoms persist, you may need arthroscopic surgery.”b.“You are experiencing degeneration of your knee, which may not resolve.”c.“Your disease is due to repeated stress on the patellar tendon. It is usually self-limited, and your symptoms should resolve with rest.”d.“Increasing your activity and performing knee-strengthening exercises will help to decrease the inflammation and maintain mobility in the knee.”

When assessing muscle strength, the nurse observes that a patient has complete range of motion against gravity with full resistance. What should the nurse record using a 0 to 5+ scale?a.2+b.3+c.4+d.5+

The nurse is examining a 6-month-old baby and places the baby’s feet flat on the table and flexes his knees up. The nurse notes that the right knee is significantly lower than the left. Which of the following is true of this finding?a.This is a positive Allis sign and suggests hip dislocation.b.The infant probably has a dislocated patella on the right knee.c.This is a normal finding for the Allis test for an infant of this age.d.The infant should return to the clinic in 2 weeks to see if this has changed.

The nurse is assessing a 1-week-old infant and testing his muscle strength. The nurse lifts the infant with her hands under the axillae, and notes that the infant starts to “slip” between her hands. The nurse should:a.suspect a fractured clavicle.b.consider that the infant may have a deformity of the spine.c.suspect that the infant may have weakness of the shoulder muscles.d.consider this a normal finding because the musculature of an infant at this age is undeveloped.

The nurse is planning to measure a patient’s angles of joint flexion. What instrument will be used?a.Caliperb.Protracterc.Goniometerd.Measuring tape

Which test should the nurse use to check for large amounts of fluid around the patella?a.Ballottementb.Tinel’s signc.Phalen’s testd.McMurray’s test

A patient has been diagnosed with a ganglion cyst over the dorsum of his left wrist. He asks the nurse, “What is this thing?” The nurse’s best answer is:a.“It is a common benign tumour.”b.“It is a tumor that will have to be watched because it may turn malignant.”c.“It is caused by chronic repetitive motion injury.”d.“It is a skin infection that will need to be drained.”

A man who has had gout for several years comes to the clinic with a “problem with my toe.” On examination, the nurse notes the presence of hard, painless nodules over the great toe; one has burst open and released a chalky discharge. This finding is known as:a.a callus.b.a plantar wart.c.tenosynovitis.d.tophi.

When performing a musculoskeletal assessment, the nurse knows that the correct approach for the examination should be:a.proximal to distal.b.distal to proximal.c.posterior to anterior.d.anterior to posterior.

Which sound is normally elicited when percussing in the seventh right intercostal space at the midclavicular line over the liver?a.Dullnessb.Tympanyc.Resonanced.Hyperresonance

Which structure is located in the left lower quadrant of the abdomen?a.Liverb.Duodenumc.Gallbladderd.Sigmoid colon

A patient is having difficulty swallowing medications and food. The nurse would document that this patient has:a.aphasia.b.dysphasia.c.dysphagia.d.myophagia.

The nurse suspects that a patient has a distended bladder. How should the nurse assess for this condition?a.Percuss and palpate in the lumbar region.b.Inspect and palpate in the epigastric region.c.Auscultate and percuss in the inguinal region.d.Percuss and palpate in the hypogastric region.

Which of the following changes may occur in the gastrointestinal system of an aging adult?a.Increased salivationb.Increased esophageal emptyingc.Increased peristalsisd.Decreased gastric acid secretion

A 22-year-old man falls with his motorcycle, and lands on the handlebars on his left side. He comes to the clinic for an examination. The nurse suspects that he may have injured his spleen. Which of the following is true regarding assessment of the spleen in this situation?a.The spleen can be enlarged as a result of trauma.b.The spleen is normally felt upon routine palpation.c.If an enlarged spleen is noted, palpate it thoroughly to determine size.d.An enlarged spleen should not be palpated because it can rupture easily.

A patient’s abdomen appears bulging and stretched. The nurse would describe this finding as:a.obese.b.herniated.c.scaphoid.d.protuberant.

When viewed horizontally, a scaphoid contour of the abdomen displays:a.a flat profile.b.a convex profile.c.a bulging profile.d.a concave profile.

While examining a patient, the nurse observes abdominal pulsations between the xiphoid and umbilicus. The nurse would suspect that these are:a.pulsations of the renal arteries.b.pulsations of the inferior vena cava.c.normal abdominal aortic pulsations.d.increased peristalsis from a bowel obstruction.

The nurse knows that a potential cause of hypoactive bowel sounds is:a.diarrhea.b.peritonitis.c.laxative use.d.gastroenteritis.

The main reason auscultation precedes percussion and palpation of the abdomen is to:a.determine areas of tenderness.b.prevent distortion of bowel sounds.c.allow the patient more time to relax, and therefore be more comfortable with the physical examination.d.prevent distortion of vascular sounds, such as bruits and hums.

Which of the following is true of bowel sounds?a.They are usually loud, high-pitched, rushing, tinkling sounds.b.They are usually high-pitched, gurgling, irregular sounds.c.They sound like “two pieces of leather being rubbed together.”d.They originate from the movement of air and fluid through the large intestine.

Which of the following is a normal finding in an abdominal assessment?a.The presence of a bruit in the femoral areab.A tympanic percussion note in the umbilical regionc.A palpable spleen between the ninth and eleventh ribs in the left midaxillary lined.A dull percussion note in the left upper quadrant at the midclavicular line

The nurse is assessing the abdomen of a pregnant woman, who is complaining of having a continuous “stomach ache”. The nurse knows that esophageal reflux during pregnancy can cause:a.diarrhea.b.pyrosis.c.dysphagia.d.constipation.

Percussion notes heard during an abdominal assessment may include:a.flatness, resonance, and dullness.b.resonance, dullness, and tympany.c.tympany, hyperresonance, and dullness.d.resonance, hyperresonance, and flatness.

A patient is complaining of tenderness along the costovertebral angles. The nurse knows that this symptom is most often indicative of:a.ovary infection.b.liver enlargement.c.kidney inflammation.d.spleen enlargement.

Which of the following is present in a patient with ascites?a.Fluidb.Fecesc.Flatusd.Fibroid tumours

The nurse knows that during an abdominal assessment, deep palpation is used to determine:a.bowel motility.b.enlarged organs.c.superficial tenderness.d.overall impression of skin surface and superficial musculature.

Tenderness on light palpation in the right lower quadrant could indicate a disorder of which of the following structures?a.Spleenb.Sigmoidc.Appendixd.Gallbladder

Which of the following statements regarding abdominal assessment of the aging adult is true?a.The abdominal tone is increased.b.The abdominal musculature is thinner.c.The abdominal rigidity with acute abdominal conditions is more common.d.The aging person complains of more pain with an acute abdomen than a younger person would.

During an assessment of a newborn infant, the nurse recalls that pyloric stenosis would be manifested by:a.projectile vomiting.b.hypoactive bowel activity.c.a palpable, olive-sized mass in right lower quadrant.d.pronounced peristaltic waves crossing from right to left.

To detect diastasis recti, the nurse should have the patient perform which of the following manoeuvres?a.Relax in the supine position.b.Raise arms in the left lateral position.c.Raise arms over the head while supine.d.Raise the head while remaining supine.

Which of the following statements about aortic aneurysm is true?a.A bruit is absent.b.Femoral pulses are increased.c.A pulsating mass is usually present.d.Most are located below the umbilicus.

During an abdominal assessment, the nurse is unable to hear the patient’s bowel sounds. Before reporting this finding as “silent bowel sounds”, the nurse should listen for at least:a.1 minute.b.5 minutes.c.10 minutes.d.2 minutes in each quadrant.

A patient is suspected of having cholecystitis (inflammation of the gallbladder). The nurse will conduct which of the following to assess for this condition?a.Obturator testb.Murphy’s signc.Assessment for rebound tendernessd.Iliopsoas muscle test

Just before going home, a new mother asks the nurse about her baby’s umbilical cord. The nurse would tell her that:a.it should fall off by 10 to 14 days.b.at birth, the cord is a bluish colour.c.it contains two veins and one artery.d.skin will cover the area within 1 week.

Which of the following percussion findings would be found in a patient with a large amount of ascites?a.Dullness across the abdomenb.Flatness in the right upper quadrantc.Hyperresonance in the left upper quadrantd.Tympany in the right and left lower quadrants

A 40-year-old man states that his doctor told him that he has a hernia. He asks the nurse to explain what a hernia is. Which appropriate action can be taken by the nurse?a.Tell him not to worry, since most men his age develop hernias.b.Refer him to his physician for additional consultation, because he or she made the initial diagnosis.c.Explain that a hernia is a loop of bowel protruding through a weak spot in the abdominal muscles.d.Explain that hernias that occur in adulthood are often the result of prenatal growth abnormalities.

A 45-year-old man visits the clinic for a physical. During the abdominal assessment, the nurse percusses and notes a 10-cm area of dullness above the right costal margin. The nurse should:a.document the presence of hepatomegaly.b.ask additional history questions regarding the patient’s alcohol intake.c.describe this as an enlarged liver, and refer the patient to a physician.d.consider this a normal finding, and proceed with the examination.

When palpating the abdomen of a 20-year-old patient, the nurse notes the presence of tenderness with deep palpation in the left upper quadrant. Which of the following structures is most likely to be involved?a.Spleenb.Sigmoidc.Appendixd.Gallbladder

The incidence of lactose intolerance is higher in adults in which group?a.Canadian Aboriginalsb.Spanish descentc.European descentd.Canadians of northern European descent

The nurse is assessing a patient for possible peptic ulcer disease. Which condition often causes this problem?a.Hypertensionb.Streptococcus infectionsc.History of constipation and frequent laxative used.Frequent use of nonsteroidal anti-inflammatory drugs

While participating in patient assessment, the student nurse hears that a patient has “hepatomegaly”, and recognizes that this term refers to:a.an enlarged liver.b.an enlarged spleen.c.distended bowels.d.excessive diarrhea.

During an assessment, the nurse notes that a patient’s umbilicus is enlarged and everted. It is midline, and there is no change in skin color. The nurse recognizes that the patient may have which condition?a.Intra-abdominal bleedingb.Constipationc.Umbilical herniad.An abdominal tumor

During an abdominal assessment, the nurse tests for a fluid wave. A positive fluid wave test occurs with:a.splenomegaly.b.distended bladder.c.constipation.d.ascites.

The nurse is preparing to examine a patient who has been complaining of right lower quadrant pain. Which technique is correct during the assessment?a.The nurse should examine the tender area first.b.The nurse should examine the tender area last.c.The nurse should avoid palpating the tender area.d.The nurse should palpate the area first and then auscultate for bowel sounds.

Which of the following statements about the arterial system is true?a.Arteries are large-diameter vessels.b.The arterial system is a high-pressure system.c.The walls of arteries are thinner than those of veins.d.Arteries can expand greatly to accommodate a large blood volume increase.

The major artery supplying the arm is the:a.ulnar artery.b.radial artery.c.brachial artery.d.deep palmar artery.

A 65-year-old patient is experiencing pain in his left calf during exercise, which disappears after resting for a few minutes. The nurse recognizes that this description is most consistent with:a.venous obstruction of the left leg.b.claudication due to venous abnormalities in the left leg.c.ischemia caused by partial blockage of an artery supplying the left leg.d.ischemia caused by complete blockage of an artery supplying the left leg.

Which of the following statements best describes the mechanism(s) by which venous blood returns to the heart?a.Intraluminal valves ensure unidirectional flow toward the heart.b.Contracting skeletal muscles milk blood distally toward the veins.c.The high-pressure system of the heart helps to facilitate venous return.d.Increased thoracic pressure and decreased abdominal pressure facilitate venous return to the heart.

Which of the following veins are responsible for most of the venous return in the arm?a.Deep veinsb.Ulnar veinsc.Subclavian veinsd.Superficial veins

A 70-year-old patient is scheduled for open-heart surgery. The physicians plan to use the great saphenous vein for the coronary bypass grafts. The patient asks, “What happens to my circulation when the veins are removed?” The nurse should reply:a.“Venous insufficiency is a common problem after this type of surgery.”b.“Oh, there are lots of veins in your body—you won’t even notice that one has been removed.”c.“You will probably experience decreased circulation after the vein has been removed.”d.“Because the deeper veins in your leg are in good condition, this vein can be removed without harming your circulation.”

Which of the following situations best describes a person at risk for development of venous disease?a.A woman in her fifth month of pregnancyb.A person who has been in bed rest for 4 daysc.A person with a 30-year, 1-pack-per-day smoking historyd.An older adult taking anticoagulant medication (adsbygoogle = window.adsbygoogle || []).push({});

Which of the following statements about the lymphatic system is true?a.Lymph flow is propelled by the contraction of the heart.b.The flow of lymph is slow compared with that of blood.c.One of the functions of the lymph is to absorb lipids from the biliary tract.d.Lymph vessels have no valves, so there is a free flow of lymph from the tissue spaces into the bloodstream and back again.

During assessment of a patient, the nurse notes an enlarged right epitrochlear lymph node. What should the nurse do next?a.Palpate the patient’s abdomen, noting any tenderness.b.Carefully palpate the cervical lymph nodes, checking for any enlargement.c.Ask additional history questions regarding any recent ear infection or sore throat.d.Examine the patient’s lower arm and hand, checking for the presence of any infection or lesions.

The nurse is examining the lymphatic system of a healthy 3-year-old child. Which of the following would the nurse expect to find?a.Excessive swelling of the lymph nodesb.The presence of palpable lymph nodesc.No nodes palpable because of the immature immune system of a childd.Fewer numbers and decreased size of lymph nodes compared with an adult

Which of the following is a normal physiological change associated with the aging process?a.Hormonal changes causing vasodilation and a resulting drop in blood pressure.b.Progressive atrophy of the intramuscular calf veins, causing venous insufficiency.c.Peripheral blood vessels growing more rigid with age, producing a rise in systolic blood pressure.d.Narrowing of the inferior vena cava, causing low blood flow and increases in venous pressure, resulting in varicosities.

A 67-year-old patient states that he recently began experiencing pain in his left calf while climbing the 10 stairs to his apartment. This pain is relieved by sitting for about 2 minutes; then he is able to resume his activities. This patient is most likely experiencing:a.claudication.b.sore muscles.c.muscle cramps.d.venous insufficiency.

The nurse uses the “profile sign” to detect:a.pitting edema.b.early clubbing.c.symmetry of the fingers.d.insufficient capillary refill.

The nurse is performing well-child assessment on a 3-year-old child. The child’s vital signs are normal. Capillary refill time is 5 seconds. The nurse would:a.ask the parent if the child has had frostbite in the past.b.suspect that the child has a venous insufficiency problem.c.consider this a delayed capillary refill time and investigate further.d.consider this a normal capillary refill time that requires no further assessment.

When assessing a patient, the nurse documents the left femoral pulse as 0/0-4+. Which of the following findings would the nurse expect at the dorsalis pedis pulse?a.0/0-4+b.1+/0-4+c.2+/0-4+d.3+/0-4+

When performing peripheral vascular assessment on a patient, the nurse is unable to palpate the ulnar pulses. The patient’s skin is warm, and capillary refill time is normal. The nurse would next:a.check for the presence of claudication.b.refer the individual for further evaluation.c.consider this a normal finding and proceed with peripheral vascular evaluation.d.ask the patient if he or she has experienced any unusual cramping or tingling in the arm.

Which of the following pulses would most likely be a finding in an individual with untreated hyperthyroidism?a.A normal pulseb.An absent pulsec.A bounding pulsed.A weak, thready pulse

Why would the nurse would perform a modified Allen test?a.To measure the rate of lymphatic drainageb.To evaluate the adequacy of capillary patency before drawing venous bloodc.To evaluate the adequacy of collateral circulation before cannulating the radial arteryd.To evaluate the venous refill rate that occurs after the ulnar and radial arteries are temporarily occluded

The nurse is aware that a positive Homan’s sign in a patient:a.occurs with venous insufficiency.b.is indicative of possible thrombophlebitis.c.is seen in the presence of severe edema.d.indicates problems with arterial circulation.

The nurse is attempting to assess the femoral pulse in an obese patient. Which of the following actions would be most appropriate?a.Have the patient assume a prone position.b.Ask the patient to bend the knees to the side in a froglike position.c.With the patient in a semi-Fowler’s position, press firmly against the bone.d.Listen with a stethoscope for pulsations because it is very difficult to palpate the pulse.

How would the nurse document mild, slightly pitting edema of the ankles of a pregnant patient?a.1+/0-4+b.3+/0-4+c.4+/0-4+d.Edema present

A patient has hard, nonpitting edema of the left lower leg and ankle. The nurse is aware that:a.nonpitting, hard edema occurs with lymphatic obstruction.b.alterations in arterial function will cause this edema.c.phlebitis of a superficial vein will cause bilateral edema.d.longstanding arterial obstruction will cause pitting edema.

When assessing a patient’s pulse, the nurse notes that the amplitude is weaker during inspiration and stronger during expiration. When the nurse measures the blood pressure, the reading decreases by 20 mm Hg during inspiration and increases with expiration. This patient is experiencing:a.pulsus alternans.b.pulsus bisferiens.c.pulsus bigeminus.d.pulsus paradoxus.

The nurse is performing peripheral vascular assessment on a bedridden patient, and notes the following findings in the right leg: increased warmth, swelling, redness, tenderness to palpation, and a positive Homan’s sign. The nurse would:a.re-evaluate the patient in a few hours.b.consider this a normal finding for a bedridden patient.c.seek emergency referral because of the risk of pulmonary embolism.d.ask the patient to raise his leg off of the bed and check for pain on elevation.

During assessment, the nurse has elevated a patient’s legs 30 cm (1 ft) off the table and has had him wiggle his feet to drain off venous blood. After helping him to sit up and dangle his legs over the side of the table, the nurse would expect that a normal finding at this point would be:a.marked elevational pallor.b.venous filling within 15 seconds.c.pain in the feet and lower legs when assuming a sitting position.d.colour returning to the feet within 20 seconds of assuming a sitting position.

A patient has bilateral pitting edema of the feet. While assessing the peripheral vascular system, the nurse’s primary focus should be:a.oxygenation of the lower extremities.b.arterial function of the lower extremities.c.venous function of the lower extremities.d.possible thrombophlebitis of the lower extremities.

Which of the following statements about the manual compression test is true?a.Rapid filling of the veins indicates incompetent veins.b.Competent valves in the veins will transmit a wave to the distal fingers.c.A palpable wave transmission occurs when the valves are incompetent.d.The test assesses whether the valves of varicosity are competent when the person is in the supine position.

During assessment, the nurse notes that a patient’s left arm is swollen from the shoulder down to the fingers, with nonpitting edema. The right arm is normal. The patient had a mastectomy 1 year ago. Which of the following problems does the nurse suspect?a.Venous stasisb.Lymphedemac.Arteriosclerosisd.Deep vein thrombosis

Which of the following statements about assessment of the ankle-brachial index (ABI) is true?a.Normal ABI indices are from 0.50 to 1.0.b.The normal ankle pressure is slightly lower than the brachial pressure.c.The ABI is a reliable measurement of peripheral vascular disease in diabetic individuals.d.An ABI of 0.90 to 0.70 indicates the presence of peripheral vascular disease and mild claudication.

The nurse is performing a well-child assessment on a 5-year-old boy. He has no current history that would lead the nurse to suspect any illness. His past medical history is unremarkable, and he received immunizations 1 week ago. Which of the following findings would be considered normal in this situation?a.Enlarged, warm, tender nodesb.Lymphadenopathy of the cervical nodesc.Palpable firm, small, shotty, mobile, nontender lymph nodesd.Firm, rubbery, large nodes, somewhat fixed to the underlying tissue

Which of the following sounds is heard when using a Doppler ultrasonic stethoscope to recognize venous flow?a.A low humming soundb.A regular “lub, dub” patternc.A swishing, whooshing soundd.A steady, even, flowing sound

When describing a weak, thready pulse, the nurse should document:a.“Easily palpable, pounds under the fingertips.”b.“Greater than normal force, then collapses suddenly.”c.“Hard to palpate, may fade in and out, easily obliterated by pressure.”d.“Rhythm is regular, but force varies with alternating beats of large and small amplitude.”

During assessment, a patient tells the nurse that her fingers often change colour when she goes out in cold weather. She describes these episodes as her fingers first turning white, then blue, then red with a burning, throbbing pain. The nurse suspects that she is experiencing:a.lymphedema.b.Raynaud’s syndrome.c.deep vein thrombosis.d.chronic arterial insufficiency.

During a routine visit to the clinic, a patient takes off his shoes and shows the nurse “this awful sore that won’t heal.” On examination, the nurse notes on the left great toe a 3-cm round ulcer with a pale ischemic base, well-defined edges, and no drainage. The nurse will assess for other signs and symptoms of:a.varicosities.b.a venous stasis ulcer.c.an arterial ischemic ulcer.d.pitting edema.

The nurse is reviewing the assessment of a patient’s peripheral pulses and notes that the radial pulses are “2+.” What type of pulse does this reading indicate?a.Boundingb.Normalc.Weakd.Absent

The sac that surrounds and protects the heart is called the:a.pericardium.b.myocardium.c.endocardium.d.pleural space.

Which of the following best describes the direction of blood flow through the heart?a.Vena cava → right atrium → right ventricle → lungs → pulmonary artery → left atrium → left ventricleb.Right atrium → right ventricle → pulmonary artery → lungs → pulmonary vein → left atrium → left ventriclec.Aorta → right atrium → right ventricle → lungs → pulmonary vein → left atrium → left ventricle → vena cavad.Right atrium → right ventricle → pulmonary vein → lungs → pulmonary artery → left atrium → left ventricle

Which of the following best describes what is meant by “atrial kick”?a.The atria contract during systole and attempt to push against closed valves.b.The contraction of the atria at the beginning of diastole can be felt as a palpitation.c.This is the pressure exerted against the atria as the ventricles contract during systole.d.The atria contract toward the end of diastole and push the remaining blood into the ventricles.

When listening to heart sounds, the nurse is aware that the valve closures that can be heard best at the base of the heart are:a.mitral, tricuspid.b.tricuspid, aortic.c.aortic, pulmonic.d.mitral, pulmonic.

Which of the following describes the closure of the valves in a normal cardiac cycle?a.The aortic valve closes slightly before the tricuspid valve does.b.The pulmonic valve closes slightly before the aortic valve does.c.The tricuspid valve closes slightly later than the mitral valve does.d.Both the tricuspid and pulmonic valves close at the same time.

The component of the conduction system referred to as the pacemaker of the heart is the:a.atrioventricular (AV) node.b.sinoatrial (SA) node.c.bundle of His.d.bundle branches.

Which sequence does the electrical stimulus of the cardiac cycle follow?a.AV node → SA node → bundle of Hisb.Bundle of His → AV node → SA nodec.SA node → AV node → bundle of His → bundle branchesd.AV node → SA node → bundle of His → bundle branches

Assessment of a 70-year-old patient with swelling in his ankles reveals jugular venous pulsations 5 cm above the sternal angle when the head of his bed is elevated 45 degrees. The nurse is aware that this indicates:a.decreased fluid volume.b.increased cardiac output.c.narrowing of jugular veins.d.increased pressure on the right side of his heart.

A 25-year-old woman in her fifth month of pregnancy has a blood pressure of 100/70 mm Hg. In reviewing her previous examination findings, the nurse notes that her blood pressure in her second month was 124/80 mm Hg. In evaluating this change, what does the nurse know to be true?a.This is the result of peripheral vasodilatation and is an expected change.b.Because of increased cardiac output, the blood pressure should be higher this time.c.This is not an expected finding because it would mean a decreased cardiac output.d.This would mean a decrease in circulating blood volume, which is dangerous for the fetus.

A 45-year-old man is in the clinic for “a routine physical.” During history taking, the patient states he has been having difficulty sleeping. “I’ll be sleeping great and then I wake up and feel like I can’t get my breath.” The nurse’s best response to this would be:a.“When was your last electrocardiogram?”b.“It’s probably because it’s been so hot at night.”c.“Do you have any history of problems with your heart?”d.“Have you had a recent sinus infection or upper respiratory infection?”

In assessing a patient’s major risk factors for heart disease, which of the following would the nurse want to include in the history?a.Family history, hypertension, stress, ageb.Personality type, high cholesterol, diabetes, smokingc.Smoking, hypertension, obesity, diabetes, high cholesterold.Alcohol consumption, obesity, diabetes, stress, high cholesterol

The mother of a 3-month-old states that her daughter has not been gaining weight. With further questioning, the nurse finds that the infant falls asleep after nursing and wakes up after a short amount of time, hungry again. What other information would the nurse want to obtain?a.The position that the baby sleeps inb.Sibling history of eating disordersc.Amount of background noise when the baby is eatingd.Presence of dyspnea or diaphoresis when sucking

While assessing the carotid arteries of an older adult with cardiovascular disease, the nurse would:a.palpate the artery in the upper one third of the neck.b.listen with the bell of the stethoscope to auscultate for bruits.c.palpate both arteries simultaneously to compare amplitude.d.instruct the patient to take slow deep breaths during auscultation.

During the assessment of a 68-year-old man with a recent onset of right-sided weakness, the nurse hears a blowing, swishing sound with the bell of the stethoscope over the left carotid artery. This finding would indicate:a.a valvular disorder.b.blood flow turbulence.c.fluid volume overload.d.ventricular hypertrophy.

During inspection of the precordium of an adult patient, the nurse notices the chest moving in a forceful manner along the fourth-to-fifth left intercostal space at the midclavicular line. This finding most likely suggests:a.a normal heart.b.a systolic murmur.c.enlargement of the left ventricle.d.enlargement of the right ventricle.

During assessment of a healthy adult, where would the nurse palpate for the apical impulse?a.Third left intercostal space at the midclavicular lineb.Fourth left intercostal space at the sternal borderc.Fourth left intercostal space at the anterior axillary lined.Fifth left intercostal space at the midclavicular line

While auscultating for heart sounds, which sequence would the nurse use?a.Aortic area → pulmonic area → Erb’s point → tricuspid area → mitral areab.Pulmonic area → aortic area → Erb’s point → tricuspid area → mitral areac.Aortic area → tricuspid area → Erb’s point → mitral area → pulmonic aread.Pulmonic area → Erb’s point → tricuspid area → pulmonic area → mitral area

While counting the apical pulse in a 16-year-old patient, the nurse notes an irregular rhythm. His rate speeds up on inspiration and slows on expiration. What would the nurse do next?a.Talk to the patient about his intake of caffeine.b.Do an electrocardiogram after the examination.c.No further action is needed because this is normal.d.Refer the patient to a cardiologist for further testing.

Which of the following statements about S1 during cardiac auscultation is true?a.S1 is louder than S2 at the base.b.S1 indicates the beginning of diastole.c.S1 coincides with the carotid artery pulse.d.S1 is caused by closure of the semilunar valves.

Which of the following would the nurse expect to find during cardiac assessment of a 4-year-old child?a.S3 when sitting upb.Persistent tachycardia above 150c.Murmur at second left intercostal space when supined.Palpable apical impulse in fifth left intercostal space lateral to the midclavicular line

While auscultating for heart sounds in a 7-year-old during a “routine physical,” the nurse hears the following: an S3, a soft murmur at left midsternal border, and a venous hum when the child is standing. Which of the following about the findings is true?a.S3 is indicative of heart disease in children.b.These can all be normal findings in a child.c.These are indicative of congenital problems.d.The venous hum most likely indicates an aneurysm.

During precordial assessment of an 8-month pregnant patient, the nurse palpates the apical impulse at the fourth left intercostal space lateral to the midclavicular line. This would indicate:a.right ventricular hypertrophy.b.increased volume and size of the heart as a result of pregnancy.c.displacement of the heart from elevation of the diaphragm.d.increased blood flow through the internal mammary artery.

In auscultating for an S4 with a stethoscope, the nurse would listen with the:a.bell at the base, with the patient leaning forward.b.bell at the apex, with the patient in the left lateral position.c.diaphragm in the aortic area, with the patient sitting.d.diaphragm in the pulmonic area, with the patient in the supine position.

The vital signs of a 70-year-old patient with a history of hypertension are BP 180/100 and HR 90. The nurse hears an extra heart sound at the apex immediately before S1. The sound is heard only with the bell while patient is in the left lateral position. With these findings and the patient’s history, the nurse knows that this extra heart sound is most likely:a.split S1.b.atrial gallop.c.diastolic murmur.d.summation sound.

The nurse is performing cardiac assessment on a 65-year-old patient 3 days after the patient had a myocardial infarction. Heart sounds are normal when she is supine, but while the patient is sitting and leaning forward, the nurse hears a high-pitched, scratchy sound at the apex with the diaphragm of the stethoscope. The sound disappears on inspiration. The nurse suspects:a.increased cardiac output.b.another myocardial infarction.c.inflammation of the precordium.d.ventricular hypertrophy resulting from muscle damage.

The mother of a 10-month-old tells the nurse that she has noticed that her son turns blue whenever he is crying and this is happening more and more frequently. He is also not crawling yet. During examination, the nurse identifies a thrill at the left lower sternal border as well as a loud systolic murmur. What would be the most likely cause of these findings?a.Tetralogy of Fallotb.Atrial septal defectc.Patent ductus arteriosusd.Ventricular septal defect

A 30-year-old woman with a history of mitral valve problems states that lately she has been “very tired.” She has started waking up at night and feels like her “heart is pounding.” During assessment, the nurse identifies a thrill and lift at the fifth left intercostal space midclavicular line. In the same area, the nurse also identifies a blowing, swishing sound right after S1. These findings would be most consistent with:a.heart failure.b.aortic stenosis.c.pulmonary edema.d.mitral regurgitation.

During cardiac assessment on an adult patient in the hospital for “chest pain,” the nurse finds the following: jugular vein pulsations 4 cm above sternal angle when he is elevated at 45 degrees, BP 98/60, HR 130; ankle edema; difficulty breathing when supine; and S3. Which of the following best explains the cause of these findings?a.Fluid overloadb.Atrial septal defectc.Myocardial infarctiond.Heart failure

Normal splitting of the second heart sound is associated with:a.expiration.b.inspiration.c.exercise state.d.low resting heart rate.

During cardiovascular assessment, the nurse is aware that a “thrill” is:a.a vibration that is palpable.b.palpated in the right epigastric area.c.associated with ventricular hypertrophy.d.a murmur auscultated at the third intercostal space.

During cardiovascular assessment, the nurse is aware that S4 is:a.heard at the onset of atrial diastole.b.usually a normal finding in the older adult.c.heard at the end of ventricular diastole.d.heard best over the second left intercostal space with the individual sitting upright.

Which of the following statements about the apical impulse is true?a.It is palpable in all adults.b.It occurs with the onset of diastole.c.Its location may be indicative of heart size.d.It should normally be palpable in the anterior axillary line.

A heart sound heard during the interval between the second heart sound (S2) and the next first sound (S1) is a(n):a.systolic sound.b.diastolic sound.c.atrial filling sound.d.ventricular contraction sound.

During assessment of a patient, the nurse notes that the apical impulse is displaced laterally and is palpable over a wide area. This indicates:a.systemic hypertension.b.pulmonic hypertension.c.pressure overload, as in aortic stenosis.d.volume overload, as in mitral regurgitation.

When the nurse is auscultating the carotid artery for bruits, which of the following reflects correct technique?a.While listening with the bell of the stethoscope, have the patient take a deep breath and hold it.b.While auscultating one side with the bell of the stethoscope, palpate the carotid artery on the other side to check pulsations.c.Lightly apply the bell of the stethoscope over the carotid artery; have the patient take a breath, exhale, and hold briefly after exhaling while you listen to it.d.Firmly place the bell of the stethoscope over the carotid artery, and have the patient take a breath, exhale, and hold briefly after exhaling while you listen to it.

Which racial group in Canada has the highest prevalence of heart disease and stroke?a.People of African descentb.People of European descentc.Aboriginal peopled.People of Asian descent

The nurse is assessing a patient suspected to have cardiomyopathy and is evaluating the hepatojugular reflux. If heart failure is present, what would the nurse find while pushing on the right upper quadrant of the patient’s abdomen, just below the rib cage?a.The jugular veins will rise for a few seconds and then recede back to the previous level if the heart is working properly.b.The jugular veins will remain elevated as long as pressure on the abdomen is maintained.c.An impulse will be visible at the fourth or fifth intercostal space, at or inside the midclavicular line.d.The jugular veins will not be detected during this manoeuvre.

The nurse is assessing the apical pulse of a 3-month-old infant and finds the rate to be 135 beats per minute. The nurse interprets this result as:a.normal for this age.b.lower than expected.c.higher than expected, probably caused by crying.d.higher than expected, reflecting persistent tachycardia.

Which of the following about the vertebra prominens is true? The vertebra prominens is:a.the spinous process of C7.b.usually not palpable in most individuals.c.opposite the interior border of the scapula.d.located next to the manubrium of the sternum.

When performing respiratory assessment on a patient, the nurse notes a costal angle of approximately 90 degrees. This characteristic is:a.seen in patients with kyphosis.b.indicative of pectus excavatum.c.a normal finding in a healthy adult.d.an expected finding in a patient with a “barrel chest.”

When assessing a patient’s lungs, the nurse is aware that the left lung:a.consists of two lobes.b.is divided by a horizontal fissure.c.consists primarily of an upper lobe on the posterior chest.d.is shorter than the right lung because of the underlying stomach.

Which statement about the apices of the lungs is true? The apices of the lungs:a.are at the level of the second rib anteriorly.b.extend 3 to 4 cm above the inner third of the clavicles.c.are located at the sixth rib anteriorly and the eighth rib laterally.d.rest on the diaphragm at the fifth intercostal space in the midclavicular line.

During examination of the anterior thorax, the nurse is aware that the trachea bifurcates anteriorly at the:a.costal angle.b.sternal angle.c.xiphoid process.d.suprasternal notch.

The nurse knows that expected assessment findings in the normal adult lung include:a.adventitious sounds and limited chest expansion.b.increased tactile fremitus and dull percussion tones.c.muffled voice sounds and symmetrical tactile fremitus.d.absent voice sounds and hyperresonant percussion tones.

The primary muscles of respiration include the:a.diaphragm and intercostals.b.sternomastoids and scaleni.c.trapezius and rectus abdominis.d.external obliques and pectoralis major.

A 65-year-old patient with a history of heart failure comes to the clinic with complaints of “being awakened from sleep with shortness of breath.” Which of the following actions by the nurse is most appropriate?a.Obtain a detailed history of the patient’s allergies and asthma.b.Tell the patient to sleep on his or her right side to facilitate ease of respiration.c.Assess for other signs and symptoms of paroxysmal nocturnal dyspnea.d.Reassure the patient that this is normal and will probably resolve within the next week.

When assessing for tactile fremitus, over which location is it normal to feel tactile fremitus most intensely?a.Between the scapulaeb.Third intercostal space, midclavicular line (MCL)c.Fifth intercostal space, midaxillary line (MAL)d.Over the lower lobes, posterior side

The nurse is aware that tactile fremitus is produced by:a.moisture in the alveoli.b.air in the subcutaneous tissues.c.sounds generated from the larynx.d.blood flow through the pulmonary arteries.

During percussion, the nurse knows that a dull percussion note elicited over a lung lobe most likely results from:a.shallow breathing.b.normal lung tissue.c.decreased adipose tissue.d.increased density of lung tissue.

The most important technique when progressing from one auscultatory site on the thorax to another is:a.side-to-side comparison.b.top-to-bottom comparison.c.posterior-to-anterior comparison.d.interspace-by-interspace comparison.

When auscultating the lungs of an adult patient, the nurse notes that over the posterior lower lobes, low-pitched, soft breath sounds are heard, with inspiration being longer than expiration. The nurse is aware that these are:a.sounds normally auscultated over the trachea.b.bronchial breath sounds and are normal in that location.c.vesicular breath sounds and are normal in that location.d.bronchovesicular breath sounds and are normal in that location.

When auscultating the chest in an adult, the nurse would:a.instruct the patient to take deep, rapid breaths.b.instruct the patient to breathe in and out through the nose.c.hold the diaphragm of the stethoscope firmly against the chest.d.hold the bell of the stethoscope lightly against the chest to avoid friction.

Percussion over an area of atelectasis in the lungs reveals:a.dullness.b.tympany.c.resonance.d.hyperresonance.

During auscultation of the lungs, the nurse is aware that decreased breath sounds would most likely be heard:a.when the bronchial tree is obstructed.b.when adventitious sounds are present.c.in conjunction with whispered pectoriloquy.d.in conditions of consolidation, such as pneumonia.

The nurse notes hyperresonant tones when percussing the thorax of an infant. The nurse’s best action would be to:a.notify the physician.b.suspect a pneumothorax.c.consider this a normal finding.d.monitor the infant’s respiratory rate and rhythm.

A normal finding when assessing the respiratory system of an older adult is:a.increased thoracic expansion.b.decreased mobility of the thorax.c.a decreased anteroposterior diameter.d.bronchovesicular breath sounds throughout the lungs.

When assessing the respiratory system of a 4-year-old child, which of the following findings would the nurse expect?a.Crepitus palpated at the costochondral junctionsb.No diaphragmatic excursion as a result of decreased inspiratory volumec.The presence of bronchovesicular breath sounds in the peripheral lung fieldsd.An irregular respiratory pattern and a respiratory rate of 40 breaths per minute at rest

When inspecting the anterior chest of an adult, the nurse should assess for:a.diaphragmatic excursion.b.symmetric chest expansion.c.the presence of breath sounds.d.the shape and configuration of the chest wall.

The nurse has noted unequal chest expansion and is aware that this occurs when:a.the patient is obese.b.part of the lung is obstructed or collapsed.c.bulging of the intercostal spaces is present.d.accessory muscles are used to augment respiratory effort.

Bronchophony heard upon auscultation is associated with:a.pneumothorax.b.hyperresonance.c.pulmonary consolidation.d.decreased breath sounds.

Bronchovesicular breath sounds are:a.musical in quality.b.usually pathological.c.expected near the major airways.d.similar to bronchial sounds, except that they are shorter in duration.

Which of the following adventitious sounds would be produced by air passing through narrowed bronchioles?a.Wheezesb.Bronchial soundsc.Bronchophonyd.Whispered pectoriloquy

A patient has a long history of chronic obstructive pulmonary disease (COPD). During assessment, the nurse is most likely to observe:a.unequal chest expansion.b.increased tactile fremitus.c.atrophied neck and trapezius muscles.d.an anteroposterior-to-transverse diameter ratio of 1:1.

A teenage patient comes to the emergency department with complaining of inability to “breathe and a sharp pain in my left chest.” Assessment findings include cyanosis, tachypnea, tracheal deviation to the right, decreased tactile fremitus on the left, hyperresonance on the left, and decreased breath sounds on the left. This description is consistent with:a.bronchitis.b.a pneumothorax.c.acute pneumonia.d.an asthma attack.

An adult patient with a history of allergies comes to the clinic complaining of wheezing and difficulty breathing while working in his yard. Assessment findings include tachypnea, use of accessory neck muscles, prolonged expiration, intercostal retractions, decreased breath sounds, and expiratory wheezes. This description is consistent with:a.asthma.b.atelectasis.c.lobar pneumonia.d.congestive heart failure.

A woman in her 26th week of pregnancy states that she is “not really short of breath” but feels that she is always aware of her breathing and the need to breathe. Which of the following would be the nurse’s best reply?a.“The diaphragm becomes fixed during pregnancy, making it difficult to take in deep breaths.”b.“The increase in estrogen levels during pregnancy often causes a decrease in the diameter of the rib cage and makes it difficult to breathe.”c.“What you are experiencing is normal. Some women may interpret this as shortness of breath, but nothing is wrong.”d.“This is normal as the fetus grows because of the increased oxygen demand on the mother’s body, which results in an increased respiratory rate.”

A 35-year-old recent immigrant complains of having a cough, expectoration of rust-coloured sputum, low-grade afternoon fevers, and night sweats for the past 2 months. The nurse’s preliminary analysis, based on this history, is that this patient may be suffering from:a.bronchitis.b.pneumonia.c.tuberculosis.d.pulmonary edema.

A 70-year-old patient is being seen in the clinic for severe exacerbation of his heart failure. Which of the following is the nurse most likely to observe in this patient?a.Shortness of breath, orthopnea, paroxysmal nocturnal dyspnea, ankle edemab.Rasping cough, thick mucoid sputum, wheezingc.Productive cough, dyspnea, weight loss, anorexiad.Fever, dry nonproductive cough, bronchial breath sounds

A patient complains of a cough that is worse at night. The nurse recognizes that this may indicate:a.pneumonia.b.postnasal drip or sinusitis.c.exposure to irritants at work.d.chronic bronchial irritation due to smoking.

During morning assessment, the nurse notes that the patient’s sputum is frothy and pink. Which of the following conditions could this finding indicate?a.Croupb.Tuberculosisc.Viral infectiond.Pulmonary edema

While auscultating for breath sounds, the nurse will use the stethoscope correctly by:a.listening to at least one full respiration in each location.b.listening as the patient inhales and then go to the next site during exhalation.c.having the patient breathe in and out rapidly while listening to the breath sounds.d.listening to sounds through clothing or the hospital gown if the patient is shy.

During palpation of the anterior chest wall, the nurse notes a coarse, crackling sensation over the skin surface and suspects:a.tactile fremitus.b.crepitus.c.friction rub.d.adventitious sounds.

The nurse is auscultating the lungs of a sleeping patient and notes short, popping, crackling sounds that stop after a few breaths. The nurse recognizes that these breath sounds are:a.atalectatic crackles, and not pathological.b.fine crackles, which may be a sign of pneumonia.c.vesicular breath sounds.d.fine wheezes.

During breast examination, it is especially important for the nurse to examine the upper outer quadrant of the breast because it is:a.the largest quadrant of the breast.b.the area where most breast tumours occur.c.where most of the suspensory ligaments are attached.d.more prone to injury and calcifications than are other areas in the breast.

During assessment of a woman’s axillary lymph system, the nurse will assess the:a.central, axillary, lateral, and sternal nodes.b.pectoral, lateral, anterior, and sternal nodes.c.central, lateral, pectoral, and subscapular nodes.d.lateral, pectoral, axillary, and suprascapular nodes.

If a patient reports a recent infection in the breast, the nurse would expect to find:a.nonspecific node enlargement.b.ipsilateral axillary node enlargement.c.contralateral axillary node enlargement.d.inguinal and cervical node enlargement.

A 9-year-old girl is in the clinic for a sports physical. Getting over her initial shyness, she asks, “Am I normal? I don’t seem to need a bra yet but I have some friends who do. What if I never get breasts?” The nurse’s best response would be:a.“Don’t worry, you still have plenty of time to develop.”b.“I know just how you feel; I was a late bloomer myself. Just be patient; they will develop.”c.“You will probably get your periods before you notice any significant development of your breasts.”d.“I understand that it is hard when you feel different from your friends. Breasts usually develop between 8 and 10 years of age.”

A 14-year-old girl is anxious about not having reached menarche yet. When taking history, which of the following should the nurse ascertain?a.The age she began to develop breastsb.The age her mother developed breastsc.The age she began to develop pubic haird.The age she began to develop axillary hair

A woman is in the family planning clinic to obtain birth control information. She states that her breasts “change all through the month” and that she is worried that this may be unusual. Which of the following would be the nurse’s best response?a.Tell her that it is unusual. Breasts of nonpregnant females usually remain unchanged throughout the month.b.Tell her that it is very common for breasts to change in response to stress and assess her life for stressful events.c.Tell her that because of hormonal changes during the monthly menstrual cycle, cyclical breast changes are common.d.Tell her that breast changes normally occur only during pregnancy and that a pregnancy test is needed at this time.

The nurse is teaching a pregnant woman about breast milk. Which of the following statements would be true?a.Breast milk is secreted immediately after delivery of the baby.b.Breast milk is rich in protein and sugars (lactose) but has very little fat.c.Colostrum, a thin, watery fluid, is present on days 4 and 5 post partum (after the birth).d.Colostrum is a fluid that contains antibodies and is usually present after the fourth month of pregnancy.

A 65-year-old patient remarks that she just cannot believe that her breasts sag so much. She thinks it could be due to lack of exercise. What explanation should the nurse offer her?a.Only women with large breasts experience sagging after menopause.b.Sagging of breasts after menopause is usually due to decreasing muscle mass within the breast.c.After menopause, a high-protein diet will help maintain muscle mass, which would prevent sagging of breasts.d.After menopause, the glandular and fat tissues atrophy, causing breast size and elasticity to decrease, resulting in sagging.

While examining a 70-year-old male patient, the nurse notices that he has bilateral gynecomastia. Which of the following describes the nurse’s best course of action?a.Recommend that he make an appointment with his physician for a mammogram.b.Ignore it; it is not unusual for men to have benign breast enlargement.c.Explain that this condition may be the result of hormonal changes and recommend that he consult his physician.d.Tell him that gynecomastia in men is usually associated with prostate enlargement and recommend that he undergo thorough screening.

During a breast health interview, a patient states that she occasionally experiences pain in her left breast. The nurse’s most appropriate response to this would be:a.“Don’t worry about the pain; breast cancer is not painful.”b.“I would like some more information about the pain in your left breast.”c.“Oh, I had pain like that after my son was born; it turned out to be a blocked milk duct.”d.“Breast pain is almost always the result of benign breast disease, so let’s just ignore it.”

During history taking, a female patient reports that she has noticed a few drops of clear discharge from her right nipple. What should the nurse do next?a.Immediately report this to the physician.b.Discontinue the interview and refer the patient for a mammogram.c.Question the patient about medications she is taking.d.Make a note about the discharge but ask no further questions because clear discharge is often found in healthy women.

During physical examination, a 45-year-old woman states that she has had a crusty, itchy rash on her breast for about 2 weeks. In trying to find the cause of the rash, which of the following questions would be important for the nurse to ask?a.Is the rash raised and red?b.Does the rash appear cyclically?c.Where did it first appear—on the nipple, the areola, or the surrounding skin?d.What was she doing when she first noticed the rash, and do her actions make it worse?

A patient is newly diagnosed with benign breast disease. Which of the following statements about benign breast disease is true? The presence of benign breast disease:a.makes it more difficult to examine the breasts.b.frequently turns into cancer in a woman’s later years.c.is easily resolved with hormone replacement therapy.d.is usually diagnosed before a woman reaches the childbearing age.

During the annual physical examination, a 43-year-old patient states that she does not perform monthly breast self-examinations. She tells the nurse that she believes that mammograms “do a much better job than I ever could to find a lump.” The nurse should explain to her that:a.mammography may not detect all palpable lumps.b.breast self-examination is unnecessary until the age of 50 years.c.she is correct, mammography is a good replacement for breast self-examination.d.she does not need to perform breast self-examination as long as a physician examines her breasts yearly.

During assessment, a patient reveals that she is pregnant. She states that she is not sure whether she will breastfeed her baby and asks for some information on breastfeeding. Which of the following statements about breastfeeding is accurate?a.Breastfed babies tend to be more colicky.b.Breastfeeding may reduce the risk of breast cancer.c.Breastfed babies feed more frequently than do infants on formula.d.Breastfeeding is natural to women and every woman can do it.

Which of the following women have risk factors that place them at a higher risk for breast cancer?a.A 37-year-old who is slightly overweightb.A 42-year-old who had a diagnosis of ovarian cancerc.A 45-year-old who has never been pregnantd.A 65-year-old whose mother had breast cancer

During examination, the nurse notices that a woman’s left breast is slightly larger than her right breast. Which of the following is true?a.Breasts should always be symmetrical.b.This probably occurred due to breastfeeding and is nothing to worry about.c.This finding is not unusual, but the nurse should verify that this change is not new.d.This finding is very unusual and means that the woman may have an inflammation or growth.

Which of the following are abnormal findings during the inspection phase of breast examination?a.Nipples in different planes (deviated)b.The left breast slightly larger than the rightc.Skin of a nonpregnant woman marked with linear striaed.Breasts of a pregnant woman having a fine blue network of veins visible under the skin

During physical examination, the nurse notes that a female patient has an inverted left nipple. Which of the following statements about inverted nipples is most accurate?a.Normal nipple inversion is usually bilateral.b.Unilateral nipple inversion is always a serious sign.c.It should be determined whether the inversion is a recent change.d.Nipple inversion is not significant unless accompanied by an underlying palpable mass.

During breast examination, which of the following is the correct procedure to screen for nipple or skin retraction?a.Have the woman bend and touch her toes with the tips of her fingers.b.Have the woman lie down on her left side, and note any retraction.c.Have the woman change from a supine position to a standing position, and note any lag or retraction.d.Have the woman slowly lift her arms above her head, and note any retraction or lag in movement.

During breast palpation, which of the following positions is most likely to clearly delineate significant lumps?a.Supine, with arms raised over her headb.Sitting, with arms relaxed at the sidesc.Supine, with arms relaxed at the sidesd.Sitting, with arms flexed and fingertips touching shoulders

Which of the following clinical situations would the nurse consider as being outside normal limits?a.A patient has had one pregnancy. She states that she believes she may be entering menopause. Her breast examination reveals breasts that are soft and sag slightly.b.A patient has never been pregnant. Her breast examination reveals large pendulous breasts that have a firm, transverse ridge along the lower quadrant in both breasts.c.A patient has never been pregnant. She reports that she should begin her period tomorrow. Her breast examination reveals breast tissue that is nodular and somewhat engorged. She states that the examination was slightly painful.d.A patient has had two pregnancies and she breastfed both her children. Her youngest child is now 10 years old. Her breast examination reveals breast tissue that is somewhat soft, and she has a small amount of thick yellow discharge from both nipples.

A patient states during the assessment that she noticed a new lump in her left breast near her axilla while in the shower a few days ago. The nurse should plan to:a.palpate the lump first.b.palpate the unaffected breast first.c.avoid palpating the lump because it could be a cyst, which might rupture.d.palpate the breast with the lump first and the axilla last.

While palpating, the nurse has found a lump in a female patient’s right breast. The nurse documents this as a small, round, firm, distinct lump located at 2 o’clock, 2 cm from the nipple. It is nontender and fixed. There is no associated retraction of skin or nipple, no erythema, and no axillary lymphadenopathy. Which of the following statements about missing information in the documentation is true?a.It is missing information about the shape of the lump.b.It is missing information about the lump’s consistency.c.It is missing information about the exact size of the lump.d.It is missing information about whether the lump is solitary or multiple.

The nurse is conducting a class about breast self-examination (BSE). Which of the following statements indicates proper BSE?a.The best time to perform BSE is in the middle of the menstrual cycle.b.The woman needs to do BSE only bimonthly unless she has fibrocystic breast tissue.c.The best time to perform BSE is 4 to 7 days after the first day of the menstrual period.d.If she suspects that she is pregnant, the woman should not perform BSE until her baby is born.

Which of the following statements reflects the best approach to teaching a woman about BSE?a.“BSE is more important for you, since you have never had any children.”b.“BSE is very important because one out of nine women will develop breast cancer.”c.“Monthly BSE will help you feel familiar with your own breasts and their normal variations.”d.“BSE will save your life because there is a likelihood of finding a cancerous lump between mammograms.”

A 55-year-old postmenopausal woman is being seen in the clinic for her annual physical examination. She is concerned about the changes she has noticed in her breasts in the past 5 years. She states that her breasts have decreased in size and that the elasticity has decreased so much that her breasts seem “flat and flabby.” The nurse’s best reply would be:a.“This change occurs most often because of long-term use of bras that do not provide enough support to the breast tissues.”b.“This is a normal change that occurs as women get older. It is due to the increased levels of progesterone during the aging process.”c.“Decreases in progesterone and estrogen after menopause causes atrophy of the glandular tissue in the breast. This is a normal process of aging.”d.“Postural changes in the spine make it appear that your breasts have changed in shape. Exercises to strengthen the muscles of the upper back and chest wall will help prevent the decreases in elasticity and size.”

The nurse is discussing BSE with a postmenopausal woman. The best time for postmenopausal women to perform BSE is:a.the same day every month.b.daily, during the shower or bath.c.1 week after the menstrual period.d.every year during the annual gynecological examination.

While examining a patient’s breasts, the nurse finds the left breast slightly larger than the right, as well as the presence of Montgomery’s glands bilaterally and a fine venous pattern in both breasts. The nurse would:a.palpate over the venous patterns, checking for drainage.b.consider these normal findings and proceed with the examination.c.ask extensive history questions regarding the woman’s breast asymmetry.d.continue with examination and then refer the patient for further evaluation of the Montgomery’s glands.

During examination, the nurse notes a supernumerary nipple just under the patient’s left breast. The patient tells the nurse that she always thought it was a mole. Which of the following statements about this finding is correct?a.It is a normal variation and not a significant finding.b.It is a significant finding and needs further investigation.c.It also contains glandular tissue and may leak milk during pregnancy and lactation.d.The patient is correct—it is actually a mole that happens to be located under the breast.

While examining a 75-year-old woman, the nurse notes that the skin over her right breast is thickened and the hair follicles appear exaggerated. This condition is known as:a.dimpling.b.retraction.c.peau d’orange.d.benign breast disease.

A new mother, who has been breastfeeding for 1 month, calls the clinic to report that an area of her left breast is red, swollen, tender, very hot, and hard. She has a fever of 38.3ºC (101ºF). She has also had symptoms of flu, such as chills, sweating, and tiredness. From this description, what condition does the nurse suspect?a.Mastitisb.Paget’s diseasec.Plugged milk ductd.Mammary duct ectasia

During breast examination, the nurse notes that the woman’s nipple is flat, broad, and fixed. The patient states that this “started happening a few months ago.” This finding suggests:a.dimpling.b.a retracted nipple.c.nipple inversion.d.deviation in nipple pointing.

A 54-year-old man comes to the clinic with a “horrible problem.” He tells the nurse that he has just discovered a lump in his breast and is fearful of cancer. Which statement about breast cancer in men is true?a.Breast masses in men are difficult to detect because of minimal breast tissue.b.Breast masses in men are less noticeable because of minimal breast tissue.c.Fewer than 1% of all breast cancers occur in men.d.Most breast masses in men are diagnosed as gynecomastia.

A 54-year-old man comes to the clinic with a “horrible problem.” He tells the nurse that he has just discovered a lump in his breast and is fearful of cancer. Which statement about breast cancer in men is true?a.Breast masses in men are difficult to detect because of minimal breast tissue.b.Breast masses in men are less noticeable because of minimal breast tissue.c.Fewer than 1% of all breast cancers occur in men.d.Most breast masses in men are diagnosed as gynecomastia.

The primary purpose of the ciliated mucous membrane in the nose is to:a.warm the inhaled air.b.filter out dust and bacteria.c.filter coarse particles from inhaled air.d.facilitate movement of air through the nares.

The projections in the nasal cavity that increase the surface area are called the:a.meatus.b.septum.c.turbinates.d.Kiesselbach’s plexus.

Which of the following about a newborn infant is true?a.The sphenoid sinuses are at full size at birth.b.The maxillary sinuses reach full size after puberty.c.The frontal sinuses are fairly well developed at birth.d.The maxillary and ethmoid sinuses are the only ones present at birth.

The tissue that connects the tongue to the floor of the mouth is the:a.uvula.b.palate.c.papillae.d.frenulum.

The salivary gland that is located in the cheek in front of the ear is the:a.parotid gland.b.Stenson’s gland.c.sublingual gland.d.submandibular gland.

While assessing the tonsils of a 30-year-old, the nurse notes that they look involuted and granular and appear to have deep crypts. What is the correct follow up to these findings?a.Refer the patient to a throat specialist.b.Nothing, this is the appearance of normal tonsils.c.Continue with the assessment to look for any other abnormal findings.d.Obtain a throat specimen to culture for possible strep infection.

The nurse is obtaining history for a 3-month-old infant. During the interview, the mother states, “I think she is getting her first tooth because she has started drooling a lot.” The nurse’s best response would be:a.“You’re right, drooling is usually a sign of the eruption of the first tooth.”b.“It would be unusual for a 3-month-old to be getting her first tooth.”c.“This could be the sign of a problem with the salivary glands.”d.“She is just starting to salivate and hasn’t learned to swallow the saliva.”

During an assessment of an 80-year-old patient, the nurse would expect to find:a.hypertrophy of the gums.b.increased production of saliva.c.decreased ability to identify odours.d.finer and less prominent nasal hair.

During history taking, a patient tells the nurse that he has frequent nosebleeds and asks about the best way to prevent them. What would be the nurse’s best response?a.“Sit straight, and place a cold compress over your nose.”b.“Sit straight with your head tilted forward, and pinch your nose.”c.“Just let the bleeding stop on its own, but don’t blow your nose.”d.“Lie on your back with your head tilted back, and pinch your nose.”

A 92-year-old patient has had a stroke, and the right side of his face is drooping. What else would the nurse suspect?a.Epistaxisb.Agenesisc.Dysphagiad.Xerostomia

While obtaining history for a 1-year-old from the mother, the nurse notices that the baby has had a bottle in his mouth the entire time. The mother states that “it makes a great pacifier.” The best response by the nurse would be:a.“You’re right, bottles make very good pacifiers.”b.“Use of a bottle is better for the teeth than thumb sucking.”c.“It’s okay to do this as long as the bottle contains milk and not juice.”d.“Prolonged use of a bottle can increase the risk for tooth decay and ear infections.”

A 72-year-old patient has a history of hypertension and chronic lung disease. An important question for the nurse to include in history taking would be:a.“Do you use a fluoride supplement?”b.“Have you had tonsillitis in the last year?”c.“At what age did you get your first tooth?”d.“Have you noticed any dryness in your mouth?”

The nurse is performing an assessment on a 21-year-old patient and notes that his nasal mucosa appears pale, grey, and swollen. What would be the most appropriate question to ask the patient?a.“Are you aware of having any allergies?”b.“Do you have an elevated temperature?”c.“Have you had any symptoms of a cold?”d.“Have you been having frequent nosebleeds?”

The nurse is palpating the sinus areas. If they are normal, which of the following would the patient report?a.No sensationb.Firm pressurec.Pain during palpationd.Pain sensation behind eyes

During the oral assessment of a 30-year-old patient of African descent, the nurse notes bluish lips and a dark line along the gingival margin. What would the nurse do in response to this finding?a.Check the patient’s hemoglobin for anemia.b.Assess for other signs of insufficient oxygen supply.c.Proceed with the assessment, knowing that this is a normal finding.d.Ask the patient if he had been exposed to an excessive amount of carbon monoxide.

During the assessment of a 20-year-old patient with a 3-day history of nausea and vomiting, the nurse notes the following: dry mucosa and deep fissures in the tongue. This finding is indicative of:a.dehydration.b.irritation by gastric juices.c.a normal oral condition.d.side effects of nausea medication.

A 32-year-old woman is at the clinic for a checkup of “little white bumps in my mouth.” During the assessment, the nurse notes that she has a 0.5 cm white, nontender papule under her tongue and one on the mucosa of her right cheek. Which of the following would the nurse tell the patient?a.“These spots are seen with infections such as strep throat.”b.“These could be indicative of a serious lesion, so I will refer you to a specialist.”c.“This is called leukoplakia and can be caused by chronic irritation due to, for example, smoking.”d.“These bumps are Fordyce’s granules, which are sebaceous cysts and not a serious condition.”

Which of the following best describes the test to assess the function of cranial nerve X?a.Observe the patient’s ability to articulate specific words.b.Assess movement of the hard palate and uvula with the gag reflex.c.Have the patient stick out the tongue, and observe for tremors or pulling to one side.d.Ask the patient to say “ahhh,” and watch for movement of the soft palate and uvula.

A 10-year-old is at the clinic for “a sore throat lasting 6 days.” Which of the following would be consistent with an acute infection?a.Tonsils 1+/1–4+ and pinkb.Tonsils 2+/1–4+ with small plugs of white debrisc.Tonsils 3+/1–4+ with large white spotsd.Tonsils 3+/1–4+ with yellowish exudate

The nurse notices that the mother of a 2-year-old boy brings him to the clinic quite frequently for various injuries and suspects there may be some child abuse involved. The nurse should inspect the young child for:a.swollen, red tonsils.b.ulcerations on the hard palate.c.bruising on the buccal mucosa or gums.d.small yellow papules along the hard palate.

The nurse is assessing a 3-year-old who has been brought to the clinic for “drainage from the nose.” On assessment, it is found that there is a purulent, malodorous drainage from the left naris but no drainage from the right naris. The child is afebrile and has no other symptoms. What should the nurse do next?a.Refer the child to a physician for an antibiotic prescription.b.Have the mother bring the child back in 1 week.c.Perform an otoscopic examination of the left naris.d.Tell the mother that this is normal for children of this age.

During the assessment of a 26-year-old for “a spot on my lip I think is cancer,” the nurse notes the following: a cluster of clear vesicles with an erythematous base around them located at the lip–skin border. The patient mentions that she just returned from Hawaii. What would be the most appropriate response by the nurse?a.Tell the patient she will need to see a skin specialist.b.Discuss the benefits of doing a biopsy of any unusual lesion.c.Tell the patient this is herpes simplex virus I infection, which will heal in 4 to 10 days.d.Tell the patient this is most likely the result of a riboflavin deficiency and discuss nutrition.

While performing a mouth assessment on a patient, the nurse notices a 1-cm ulceration that is crusted and has an elevated border. It is located on the outer third of the lower lip. What other information would be most important for the nurse to obtain?a.Nutritional statusb.When the patient first noticed the lesionc.Whether the patient has had a recent coldd.Whether the patient has had any recent exposure to sick animals

A pregnant woman states that she is concerned about her gums because she has noticed they are swollen and have started bleeding. What would be an appropriate response by the nurse?a.“This is probably due to a vitamin C deficiency.”b.“I’m not sure what causes it, but let me know if it doesn’t get better in a few weeks.”c.“You need to make an appointment with your dentist as soon as possible to have this checked.”d.“This can be caused by hormonal changes during pregnancy.”

A 40-year-old patient who has just finished chemotherapy for breast cancer tells the nurse that she is concerned about her mouth. During assessment, the nurse finds the following: areas of buccal mucosa that are raw and red with some bleeding, as well as other areas that have a white, cheesy coating. The nurse recognizes that this abnormality is:a.carcinoma.b.candidiasis.c.leukoplakia.d.Koplik’s spots.

A hospitalized patient who has received numerous antibiotics is being assessed by the nurse, who notes that his tongue appears black and hairy. Which of the following would the nurse say to the patient?a.“We need to get a biopsy done and find out what the cause is.”b.“This is an overgrowth of hair and will go away in a few days.”c.“This is a fungal infection caused by all the antibiotics you’ve received.”d.“This is probably caused by the same bacteria you had in your lungs.”

The nurse is assessing a patient with a history of intravenous drug abuse. While assessing his mouth, the nurse notices a dark red confluent macule on the hard palate. This could be an early sign of:a.AIDS.b.measles.c.leukemia.d.carcinoma.

A mother brings her 4-month-old to the clinic with concerns about a small pad-like growth on the mid-upper lip that has been present since the baby was 1 month old. The infant has no health problems. On physical examination, the nurse notes a 0.5-cm, fleshy, elevated area on the mid-upper lip. There is no evidence of inflammation or drainage. What would the nurse tell this mother?a.“This is an area of irritation caused by teething and is nothing to worry about.”b.“This is an abnormal finding and should be evaluated by a specialist.”c.“This is the result of chronic drooling and should resolve within the next month or two.”d.“This is a sucking tubercle caused by friction due to breast- or bottlefeeding and is normal.”

A mother is concerned that her 18-month-old already has 12 teeth. She is wondering if this is normal for a child of this age. The nurse’s best response would be:a.“How many teeth did you have at this age?”b.“All 20 deciduous teeth are expected to have erupted by 4 years.”c.“This is a normal number of teeth for an 18-month-old.”d.“Normally, 16 deciduous teeth are expected to have erupted between 2 and 3 years.”

When examining the nares of a 45-year-old patient who complains of rhinorrhea, itching of the nose and eyes, and sneezing, the nurse notes the following: pale turbinates, swelling of the turbinates, and clear rhinorrhea. Which of the following is most likely the cause?a.Nasal polypsb.Acute sinusitisc.Allergic rhinitisd.Nasal carcinoma

When assessing the tongue of an adult, an abnormal finding would be:a.a smooth glossy dorsal surface.b.a thin white coating over the tongue.c.raised papillae on the dorsal surface.d.visible venous patterns on the ventral surface.

Which of the following assessment findings would the nurse be most concerned about?a.A painful vesicle inside the cheek for 2 daysb.The presence of moist, nontender Stenson’s ductsc.Stippled gingival margins that adhere snugly to the teethd.An ulceration with rolled edges on the side of the tongue

A patient has been diagnosed with “strep throat.” Which complication may occur without treatment?a.Rubellab.Leukoplakiac.Rheumatic feverd.Scarlet fever

During a checkup, a 22-year-old woman tells the nurse that she uses an over-the-counter nasal spray for her allergies. She also reports that it does not work as well as it used to. The best response by the nurse would be:a.“You should never use over-the-counter nasal sprays because of the risk of addiction.”b.“You should try switching to another brand of medication to prevent this problem.”c.“It is important to keep using this spray to keep your allergies under control.”d.“Using these nasal medications irritates the lining of the nose and may cause rebound swelling.”

During examination of a 4-year-old Aboriginal child, the nurse notices that her uvula is partially split. Which of the following statements about this condition is accurate?a.This is cleft palate, which is commonly found in Aboriginal people.b.This is a bifid uvula, which is a common occurrence in some Aboriginal groups.c.This occurs due to an injury and should be reported to the authorities.d.This is torus palatinus, common among Aboriginal people.

A patient comes into the clinic complaining of facial pain, fever, and malaise. On examination, the nurse notes swollen turbinates and a purulent discharge from the nose. The patient also complains of a dull, throbbing pain in his cheeks and teeth on the right side and pain when the nurse palpates the areas. The nurse recognizes that this patient has:a.posterior epistaxis.b.frontal sinusitis.c.maxillary sinusitis.d.nasal polyps.

A woman in her second trimester of pregnancy mentions that since becoming pregnant she has had “more nosebleeds than ever.” The nurse recognizes that this is due to:a.coagulation problems in the patient.b.increased vascularity in the upper respiratory tract as a result of the pregnancy.c.increased susceptibility to colds and nasal irritation.d.inappropriate use of nasal sprays.

The portion of the ear that consists of movable cartilage and skin is called the:a.auricle.b.concha.c.outer meatus.d.mastoid process.

The nurse is examining a patient’s ears and notices cerumen in the external canal. Which of the following statements about cerumen is correct?a.Sticky honey-coloured cerumen is a sign of infection.b.The presence of cerumen is indicative of poor hygiene.c.The purpose of cerumen is to protect and lubricate the ear.d.Cerumen is necessary for transmitting sound through the auditory canal.

When examining the ear with an otoscope, the nurse would expect to find that the tympanic membrane is:a.light pink with a slight bulge.b.pearly grey and slightly concave.c.pulled in at the base of the cone of light.d.whitish, with a small fleck of light in the superior portion.

Which of the following statements about the Eustachian tube is true?a.It is responsible for the production of cerumen.b.It remains open except during swallowing or yawning.c.It allows passage of air between the middle and outer ear.d.It helps equalize air pressure on both sides of the tympanic membrane.

A patient with a middle ear infection asks the nurse, “What does the middle ear do?” The nurse says that the function of the middle ear is to:a.maintain balance.b.interpret sounds as they enter the ear.c.conduct vibrations of sounds to the inner ear.d.increase amplitude of sound to enable the inner ear to function.

Which of the following cranial nerves is responsible for conducting nerve impulses to the brain from the organ of Corti?a.CN Ib.CN IIIc.CN VIIId.CN XI

Which of the following statements about air conduction is true?a.It is the most efficient pathway for hearing.b.It is caused by the vibrations of bones in the skull.c.The amplitude of sound determines the pitch that is heard.d.A loss of air conduction is called a conductive hearing loss.

Tests have shown that a patient has sensorineural hearing loss. During the assessment, it would be important for the nurse to:a.speak loudly so he can hear the questions.b.assess for middle ear infection as a possible cause.c.ask the patient what medications he is currently taking.d.look for the source of the obstruction in the external ear.

During an interview, the patient says that he experiences a sensation as if “everything around him is spinning.” The nurse recognizes that the portion of the ear responsible for this sensation is:a.the cochlea.b.cranial nerve VIII.c.the organ of Corti.d.the bony labyrinth.

A patient in her first trimester of pregnancy is diagnosed with rubella. Which of the following indicates the significance of this in relation to the infant’s hearing?a.Rubella may affect the mother’s hearing but not the child’s.b.Rubella can damage the organ of Corti in the fetus, which will result in impaired hearing.c.Rubella is dangerous to the fetus only when it occurs in the second trimester of pregnancy.d.Rubella can impair the development of CN VIII and thus affect hearing.

The mother of a 2-year-old is concerned because her son has had three ear infections in the past year. Which of the following would be an appropriate response by the nurse?a.“It is unusual for a small child to have frequent ear infections unless there is something else wrong.”b.“We need to check the immune system of your son to see why he is having so many ear infections.”c.“Ear infections are not uncommon in infants and toddlers because they tend to have more cerumen in the external ear.”d.“Your child’s Eustachian tube is shorter and wider than that in an adult, which allows infections to develop more easily.”

A 31-year-old patient tells the nurse that he is experiencing a progressive loss of hearing. He says that it does seem to help when people speak more loudly or if he turns up the volume. The most likely cause of his hearing loss is:a.otosclerosis.b.presbycusis.c.trauma to the bones.d.frequent ear infections.

A 70-year-old patient reports to the nurse that he is having trouble hearing, especially when he is in large groups. He says he “can’t always tell where the sound is coming from” and the words often sound “mixed up.” What might the nurse suspect as the cause for this?a.Atrophy of the apocrine glandsb.Cilia becoming coarse and stiffc.Degeneration of nerves in the inner eard.Scarring of the tympanic membrane

During an assessment of a 20-year-old patient of Asian descent, the nurse notices that he has dry, flaky cerumen in his ear canal. What is the significance of this finding?a.This is probably the result of eczema lesions in his ear.b.This indicates poor hygiene.c.This is a normal finding, and no further follow-up is necessary.d.This could be indicative of change in cilia; the nurse should assess for conductive hearing loss.

The nurse is taking history from a patient who may have a perforated eardrum. What would be an important question in this situation?a.“Do you ever notice ringing or crackling in your ears?”b.“When was the last time you had your hearing checked?”c.“Have you ever been told you have any type of hearing loss?”d.“Was there any relationship between the ear pain and the discharge you mentioned?”

A 31-year-old patient tells the nurse that he has noticed pain in his left ear when people speak loudly to him. The nurse knows that this finding:a.is normal for people of that age.b.is a characteristic of recruitment.c.may indicate a middle ear infection.d.indicates that the patient has a cerumen impaction.

While discussing the history of a 6-month-old infant, the mother tells the nurse that she took a great deal of aspirin while she was pregnant. What question would the nurse want to include in the history?a.“Does your baby startle at loud noises?”b.“Has the baby had any surgeries on her ears?”c.“Have you noticed any drainage from her ears?”d.“How many ear infections has your baby had since birth?”

The nurse is performing an otoscopic examination on an adult. Which of the following is true?a.Tilt the person’s head forward during the examination.b.Once the speculum is in the ear, release the traction.c.Pull the pinna up and back before inserting the speculum.d.Use the smallest speculum to decrease the amount of discomfort.

The nurse is assessing a 16-year-old patient with head injuries from a recent motor vehicle accident. Which of the following statements indicates the most important reason for assessing for any drainage from the canal?a.If the drum has ruptured, there will be purulent drainage.b.Bloody or clear watery drainage can indicate a basal skull fracture.c.The auditory canal many be occluded from increased cerumen.d.There may be occlusion of the canal caused by foreign bodies from the accident.

While performing a voice test to assess hearing in a patient, which of the following would the nurse do?a.Shield the lips while speaking so that the sound is muffled to the patient.b.Whisper two-syllable words and ask the patient to repeat them.c.Ask the patient to place his finger in his ear to occlude outside noise.d.Stand about 120 cm away to ensure that the patient can really hear at this distance.

While performing an examination of a 3-year-old with a suspected ear infection, the nurse would:a.omit the otoscopic exam if the child has a fever.b.pull the ear up and back before inserting the speculum.c.ask the mother to leave the room while examining the child.d.perform the otoscopic examination at the end of the assessment.

Which of the following statements about otoscopic examination of a newborn would be true?a.Immobility of the drum is a normal finding.b.An “injected” membrane would indicate infection.c.The normal membrane may appear thick and opaque.d.The appearance of the membrane is identical to that of an adult.

During the ear examination of an 80-year-old patient, which of the following would be a normal finding?a.Loss of high-tone frequency hearingb.Increased elasticity of the pinnac.A thin, translucent membraned.A shiny, pink tympanic membrane

An assessment of a 23-year-old patient reveals the following: an auricle that is tender and reddish-blue in colour and has small vesicles. What additional information would the nurse need to know?a.Any change in the ability to hearb.Any recent drainage from the earc.Recent history of trauma to the eard.Any prolonged exposure to extreme cold

The mother of a 2-year-old is concerned about tympanostomy tubes that are going to be inserted in her son’s ears. Which of the following would the nurse include in the teaching plan?a.The tubes are placed in the inner ear.b.The tubes are used in children with sensorineural loss.c.The tubes are permanently inserted during a surgical procedure.d.The purpose of the tubes is to decrease the pressure and allow for drainage.

During hearing assessment using the Weber test, the nurse finds that sound lateralizes to the patient’s left ear. What can the nurse conclude from this?a.The patient has a conductive hearing loss in the right ear.b.Lateralization is a normal finding from the Weber test.c.The patient could have either a sensorineural loss or a conductive loss.d.A mistake has occurred; the test must be repeated.

A patient states that she is unable to hear well with her left ear. The Weber test shows lateralization to the right ear. The patient has AC>BC with ratio of 2:1 in both ears, left-AC 10 sec and BC 5 sec, right-AC 30 sec, and BC 15 sec. What would be the interpretation of these results?a.The patient may have sensorineural loss.b.The test results are reflective of normal hearing.c.Conduction of sound through bones is impaired.d.These results make no sense, so further tests should be done.

Which of the following signs would the nurse expect to find on assessment of an individual with otitis externa?a.Rhinorrheab.Periorbital edemac.Pain over the maxillary sinusesd.Enlarged superficial cervical nodes

When performing an otoscopic examination on a 5-year-old child with a history of chronic ear infections, the nurse sees that his right tympanic membrane is amber-yellow in colour and there are air bubbles behind the tympanic membrane. The child reports occasional hearing loss and a popping sound with swallowing. The preliminary analysis based on this information would be that:a.this is most likely serous otitis media.b.the child has acute purulent otitis media.c.there is evidence of a resolving cholesteatoma.d.the child is experiencing the early stages of perforation.

A 65-year-old male has reported a crusty nodule behind the pinna. Over the past 6 months, the nodule has been bleeding intermittently and has not healed. On physical assessment, the nurse finds an ulcerated crusted nodule with an indurated base. The preliminary analysis in this situation would be that this:a.is most likely a benign sebaceous cyst.b.is most likely a Darwin’s tubercle and is not significant.c.could be potential carcinoma and warrants referral.d.is a tophus, which is common in older adults, and is a sign of gout.

The nurse is performing middle ear assessment on a 15-year-old patient who has a history of chronic ear infections. When examining the right tympanic membrane, the nurse sees dense white patches. The tympanic membrane is otherwise unremarkable. It is pearly, with the light reflex at 5 o’clock and visible landmarks. The nurse should:a.refer the patient for investigating the possibility of a fungal infection.b.know that these are scars caused from frequent ear infections.c.consider that these findings may be an indication of blood in the middle ear.d.be concerned about the patient’s ability to hear because of this abnormality of the tympanic membrane.

The nurse is preparing to do an otoscopic examination on a 2-year-old child. Which of the following reflects correct procedure?a.Pull the pinna down.b.Pull the pinna up and back.c.Tilt the child’s head slightly toward the examiner.d.Have the child touch his chin to his chest.

Which of the following is a risk factor for ear infections in young children?a.Family historyb.Air conditioningc.Excessive cerumend.Secondhand cigarette smoke

During an otoscopic examination, the nurse notes an area of black and white dots on the tympanic membrane and ear canal wall. What does this finding suggest?a.Malignancyb.Viral infectionc.Blood in the middle eard.Yeast or fungal infection

A 17-year-old student is a swimmer on her high school’s swim team. She has had three bouts of otitis externa so far this season and wants to know how to prevent it. The nurse instructs her to:a.use a cotton-tipped swab to dry the ear canals thoroughly after each swim.b.use rubbing alcohol or 2% acetic acid eardrops after every swim.c.irrigate the ears with warm water and suction with a bulb syringe after each swim.d.rinse the ears with a warmed solution of mineral oil and hydrogen peroxide.

During ear examination, the patient reports that he is hears a buzzing sound that is “driving me crazy!” The nurse recognizes that this symptom is:a.vertigo.b.pruritus.c.tinnitus.d.cholesteatoma.

What is a focused history and physical examination?

In documenting a focused history and performing a focused physical examination, you need to explore the chief complaint, the history of the present illness, the past medical history, medications and allergies, the family history and social history, the occupational history, and the sexual history that are relevant to ...

Which technique is used during the history taking and the physical exam process?

Which technique is used during both the history taking and the physical examination process? Inspection is the technique that is used while gathering and validating data during both the history taking and the actual hands-on physical examination.

What type of data is found during a physical examination?

Information pertinent to the physical examination can be learned from observation of speech, gestures, habits, gait, and manipulation of features and extremities. Interactions with relatives and staff are often revealing. Pigmentary changes such as cyanosis, jaundice, and pallor may be noted.

What is the purpose of health history and physical assessment?

A complete history and physical examination can identify important health issues that may be solved at the domestic medical examination visit or more chronic conditions that need further evaluation or management.